Anatomy Exam Prep for Med Students
Anatomy Exam Prep for Med Students
8. Malignancy from the palatine tonsils will metastasize first in what lymph node level?
❏ I → submental/submandibular
❏ II → upper jugular
❏ III → oral cavity, oropharynx, nasopharynx
❏ IV → hypopharynx, larynx
9. A maneuver used to test presence of hypocalcemia, in which the parotid gland over
the facial nerve is tapped to elicit ipsilateral facial twitching:
❏ Chvostek’s
❏ Trousseau’s → carpopedal spasm induced by ischemia secondary to the inflation of
a sphygmomanometer cuff, to 20 mmHg over their systolic blood pressure for 3
minutes
❏ Babinski’s → stroking the lateral sole of foot towards the big toe resulting to
dorsiflexion of big toe, and fanning of other toes; Upper motor neuron injury
❏ Pemberton’s → evaluate patients with goiter elicited by bilaterally elevating arm →
facial plethora from thyroid obstructing thoracic inlet due to inc venous return
10. The thyroid gland is enveloped by which cervical fascial layer?
❏ Superficial → subcutaneous tissue and
platysma muscles
❏ Investing → encloses parotid
and submandibular glands
❏ Visceral → superior: Hyoid and
thyroid cartilage, Inferior: great vessels of
neck to pericardium; encloses strap
muscles and visceral muscles of
pharynx, larynx, trachea and esophagus
❏ Pre-vertebral → forms floor of posterior
triangle, located anterior to vertebra
*Deep fascia comprises investing, visceral
and prevertebral
15. Which of the following movements prevent food from entering the larynx?
❏ Closing of the false vocal cord
❏ Epiglottis moves posteroinferiorly
❏ Arytenoid moves close to each other
❏ Adduction of the true vocal cord
*Function of Larynx:
1) Respiration 2) Protect from food going to airway 3) Voice production
*other answers more on for voice production
16. Which of the muscles of the larynx acts during respiration?
❏ Lateral cricoarytenoid → adduction
❏ Posterior cricoarytenoid → sole abductor
❏ Thyroarytenoid → relaxes
0
❏ Transverse arytenoid → narrows laryngeal inlet by pulling the epiglottis posteriorly
cjhernandez/djhorneja/ccorpuz
*during respiration → vocal cords are open or abducted
*talking → vocal cords are closed post woo
-
arytenoid
17. The patient was examined and revealed left sided pleurisy. Where would you expect
the pleural fluid to gravitate?
❏ Cardiac notch → located above
❏ Costomediastinal recess → at hilum; medial
❏ Costodiaphragmatic recess → most dependent portion
❏ Oblique fissure → located slightly up
18. A 34 male, obese, presented in the ER with difficulty of breathing. On examination,
there is an abnormal sound every time he breathes in and out. Where is the most
likely obstruction located?
❏ bronchus
❏ glottic
❏ pharynx
❏ thoracic trachea
*inspiratory stridor → pharynx and supraglottis
*expiratory stridor → trachea and bronchi may be blocked
19. An 18-year-old girl is thrust into the steering wheel while driving and experiences
difficulty in expiration. Which of the following muscles is most likely damaged?
❏ Innermost intercostal → inspiration
❏ External intercostal → inspiration
❏ Levator costarum→ inspiration
❏ muscles of abdominal wall
20. A 33-year-old patient is suffering from a sudden occlusion at the origin of the
descending (thoracic) aorta. This condition would most likely decrease blood flow in
which of the following intercostal arteries?
❏ Upper six posterior
❏ All of the posterior
cjhernandez/djhorneja/ccorpuz
❏ Upper two posterior → from superior intercostal artery (from costocervical trunk)
❏ Lower six posterior → originate from thoracic aorta along with subcostal artery
21. Fracture of this rib will lead to liver or splenic damage?
❏ 11 & 12th rib
❏ 9th & 10th rib
❏ 8th & 9th rib
❏ 6th & 7th rib
* 1st rib - difficult to fracture
22. Brain metastasis from Ca of the breast occurs via:
❏ Anterior intercostal
❏ lateral thoracic
❏ inferior phrenic
❏ Posterior intercostal → azygous/hemiazygous system of veins → Batson plexus
→ internal vertebral venous plexus surrounding spinal cord
*Batson plexus - network of valveless veins that connect deep pelvic veins and thoracic
veins to the internal vertebral venous plexuses; can be for LUNG CA as well
23. A 56 year old female presented with a hard breast mass accompanied by a retracted
nipple on the left breast which she claims occurs 3 months ago. Which part of the
breast is affected?
❏ lactiferous duct
❏ ligament of cooper → skin dimpling
❏ pectoralis fascia → sudden upward movement of entire breast
❏ subcutaneous lymphatics → peau D’ orange
24. Which nerve is responsible for increasing the nasal discharge in allergy?
(Parasympathetic)
❏ Ophthalmic division of V5
❏ Maxillary division of the V5 → goes along with facial nerve only
❏ Facial nerve - cranial nerve responsible
❏ Cervical sympathetic ganglion
*Parotid gland → Glossopharyngeal nerve
25. The main cause of death in COVID 19 is:
❏ Attachment of virus to alveolus type B
❏ Cytokine storm
❏ Hemorrhage
❏ Pulmonary embolism
26. What lymphatic pathway/node is responsible for breast metastasis to the opposite
side?
❏ Apical
❏ Central
❏ Internal thoracic → drains medial side
❏ Pectoral
27. Which of the following chambers is commonly injured in stab wounds on the anterior
chest?
❏ Right atrium
❏ Right ventricle → occupies majority of costal space
cjhernandez/djhorneja/ccorpuz
❏ Left ventricle
❏ Left atrium
28. Patient suffered myocardial infarction and was noted to have new onset arrhythmias,
this is probably due to occlusion of the:
❏ right coronary → supplies the right atrium which contains SA and AV node;
blockage results to rhythm disorders
* ❏ left coronary →leads to pump failure
❏ anterior interventricular → branch of left coronary
❏ Circumflex → branch of left coronary
29. Patient informed you she has mitral valve stenosis, you will place your stethoscope in
which of the ff:
❏ Right 2nd intercostals space close to the sternal border → aortic valve
❏ Left 2nd intercostals space close to the sternal border → pulmonic valve
❏ Right 6th chondrosternal junction → tricuspid valve
❏ Left 5th intercostals space along the midclavicular line
30. Presence of which of the following component of the conducting system allows
impulses to travel between the atria and ventricles even in the presence of the cardiac
skeleton?
❏ atrioventricular node → Lower part of atrial septum
❏ sino-atrial node → right atrium near sulcus terminalis
❏ atrioventricular bundle → “”, connects atria to ventricle and to cardiac skeleton
❏ Bundle branch → moderator band (right); bundle branch in the left side
31. The right lymphatic duct drains the following:
❏ Right side of thorax
❏ Right side abdominal cavity
❏ Pelvic cavity
❏ both lower extremity
32. A 70 yo male had severe substernal chest pain running down the left upper extremity.
Work up showed myocardial infarction. The pain on the left upper extremity is relayed
by which nerve on the left?
❏ vagus
❏ axillary
❏ Intercostobrachial → communicate with medial cutaneous nerve of the arm;
referred pain
❏ Phrenic - pain referred to epigastric area
33. Which of the following structures forms the basis of electrical discontinuity between
the myocardia of the atria and the ventricles?
❏ AV Bundle of His
❏ AV valves
❏ Cardiac skeleton
❏ Purkinje fibers
cjhernandez/djhorneja/ccorpuz
34. Components of the cardiac plexus:
❏ Vagus → longest cranial nerve
❏ Sympathetic
❏ Vagus and sympathetic
❏ Vagus, sympathetic and phrenic
35. Which of the following chamber is closely related to the esophagus?
❏ Right ventricle
❏ Right atrium
❏ Left ventricle
❏ Left atrium
36. The moderator band is located in which of the following chamber?
❏ Right atrium
❏ Left atrium
*❏ Right ventricle → moderator band is also called septomarginal band, facilitates
conduction time, allows coordinated contraction of papillary muscle
❏ Left Ventricle
37. Apex beat is located in the same auscultatory area of which of following heart valve:
❏ Aortic
❏ Pulmonic
❏ Mitral
❏ Tricuspid
38. Muscles found in the internal surface of the auricles:
❏ Pectinate
❏ Trabeculae carnae → ventricles
❏ Auricular
❏ Papillary → ventricles
39. The pain coming from gallbladder wall inflammation is relayed to which of the
following spinal cord levels?
❏ TV7 - TV12
❏ TV8 - TV10
❏ TV 7 - TV 9 → fundus of GB at TV 9
❏ TV 7 - LV1
40. W hich among the following abdominal wall veins is involved in portosystemic
anastomosis?
❏ Inferior epigastric → drains to femoral vein
❏ Superior epigastric → drains to internal thoracic vein
❏ Superficial epigastric→ Connects to paraumbilical vein
❏ Subcostal
41. What surface of the spleen can be palpated in a patient with splenomegaly?
❏ Superior
❏ Inferior
cjhernandez/djhorneja/ccorpuz
❏ Posterior
❏ Anterior
42. What is the medial border of the triangle of calot?
❏ Cystic duct → lateral border
❏ Liver → inferior edge of liver: superior border
❏ Common hepatic duct
❏ Common bile duct
*Contents:- Cystic artery,
- right hepatic artery and
-
cystic lymph nodes
ink
❏ Abdominal aorta →branches are choices A, B, C
45. Which part of the alimentary canal has the
most number of intestinal arcades?
! ❏ Duodenum
outrm ❏ Jejunum → Few but large AA
❏ Ileum → Numerous and small AA
❏ Duodeno-jejunal flexure
❏ CN IX
❏ CNO O
X → until proximal ⅔ of transverse colon
❏ CN XI
47. Which of the following structures may be found in the porta hepatis?
❏ Gallbladder
❏ Portal vein
❏ Hepatic vein
❏ Inferior vena cava
*Porta hepatis - is the central intraperitoneal fissure of the liver that separates the caudate
and the quadrate lobes. It is the entrance and the exit for several important vessels.
cjhernandez/djhorneja/ccorpuz
48. Which of the following vessels carry blood out of the liver?
❏ Portal vein → 80% blood supply
❏ Hepatic artery → 20% blood supply
❏ Hepatic veins → drains toO IVC
❏ Inferior vena cava
49. Pancreatic endocrine secretions are transported through which structures?
❏ Pancreatic veins
❏ Hepatic duct → conveys bile
❏ Pancreatic duct of Wirsung → exocrine secretions
❏ Portal vein
50. In blunt abdominal injuries, which part of the duodenum is usually crushed against
the lumbar vertebra?
❏ 1st → small portion only at level of L1
❏ 2nd → level of L1-L3
❏ 3rd → crosses the vertebra, inferior vena cava and abdominal aorta
❏ 4th
51. W
opening?
O
hich of the following connects the medial borders of 2 crura to form the aortic
O
❏ Median Arcuate Ligament
❏ Superior Arcuate Ligament
phreniccjhernandez/djhorneja/ccorpuz
!
52. Aortic aneurysms usually below the origins of which vessels?
❏ Celiac Artery
❏ Superior Mesenteric Artery
❏ Renal Artery
❏ Inferior Mesenteric Artery
53. The abdominal part of the Aorta extends from the its diaphragmatic opening at the
level of
❏ 8th thoracic vertebra to its bifurcation at the level of Lumbar vertebra 4.
❏ 8th thoracic vertebra to its bifurcation at the level of Lumbar vertebra 5.
O
❏ 12th thoracic vertebra to its bifurcation at the level of Lumbar vertebra 4.
❏ 12th thoracic vertebra to its bifurcation at the level of Lumbar vertebra 5.
①*TV8 - Inferior Vena Cava; bifurcate at LV 5
54. The ureter is best described by which statement hg.rs#
IMA →
side from the aorta, the aortic opening of the diaphragm transmits which of the
57. A
following structures?
cjhernandez/djhorneja/ccorpuz
❏ Vagus Nerve → esophageal opening
172 ❏ Thoracic duct → with azygos vein
60. Prostate cancer usually develops at which specific part of the prostate gland?
❏ Apex
❏ Base
❏ Peripheral Zone
❏ Transitional Zone → usual site for BPH
61. Which of the following is true regarding Renal Transplantation?
❏ The iliac fossa is usual site chosen for kidney transplant
❏ renal artery is anastomosed to the external Iliac artery → Internal Iliac Artery
❏ renal vein is anastomosed to the internal iliac vein → External Iliac Vein
❏ The ureter of the transplanted kidney is anastomosed to the contralateral ureter
62. When catheterization has failed, it is possible to pass a needle into the bladder
through the anterior abdominal wall approximately?
❏ 1 cm below the umbilicus → will hit other structures and create more bleeding
❏ 3cm below the umbilicus → way above
❏ 1cm above symphysis pubis
❏ 3cm above symphysis pubis
cjhernandez/djhorneja/ccorpuz
63. I n the gynecoid variety of pelvis, which two bony landmarks are used to determine
the clinically palpable AP diameter?
❏ Sacral promontory and ischial spine
❏ Sacral promontory and symphysis pubis
❏ Symphysis pubis and coccyx
❏ Symphysis pubis and ischial tuberosity
64. Which is responsible for the narrower configuration of the male pelvis?
❏ Curved sacrum
❏ Indented sacral promontory
❏ Coccyx directed forwards
❏ Fist sized distance between the ischial tuberosities
65. What is the most likely source if the ischiorectal fossa is infected?
❏ Alcock’s canal → also called Pudendal Canal where Internal Pudendal vessels are
found
❏ Anal canal → close proximity to Ischiorectal fossa
❏ Urethra
❏ Vagina
68. An infected Bartholin’s cyst will cause enlargement of which lymph nodes?
❏ External iliac
❏ Internal iliac
❏ Inguinal → located in vestibule
❏ Femoral
o
69. Which is true regarding Skene’s glands?
❏ Massaging the glands will produce urethral discharge → will produce substance that
will lubricate urethra
❏ These greater vestibular glands are found in the deep perineal pouch → lesser
vestibular glands; Greater vestibular glands are Bartholin’s glands
❏ Swelling of these glands is accompanied by severe pain → asymptomatic, dysuria,
dyspareunia, urinary retention, (+) palpable mass, recurrent UTI
❏ These serve as reservoir for Trichomonas ✓
*Skene’s gland located around urethra; homolog of prostate gland
70. Proper pudendal nerve block will produce which effect?
❏ Inability for penile erection
❏ Absence of cremasteric reflex → Genitofemoral N.
❏ Fecal incontinence
❏ Loss of sensation over the perineum
71. During abdominal hysterectomy before ligation of the uterine artery the surgeon
looks for the ureter at the:
❏ Middle of the broad ligament
❏ Base of the broad ligament → ureter is below uterine artery
❏ Middle of the round ligament
❏ Base of the round ligament
cjhernandez/djhorneja/ccorpuz
72. What is the significance of the isthmus in a nonpregnant uterus?
❏ Demarcates the endocervix from the ectocervix
❏ Area where the ureter crosses over the uterine artery → at the broad ligament
❏ Site of anastomosis of its dual blood supply
❏ Signifies level of the internal os → isthmus signifies opening of cervical canal
73. In which fascial condensation can one feel tender nodularities due to endometriosis?
❏ Cardinal
❏ Pubocervical
❏ Round
❏ Uterosacral
74. Vaginal carcinoma in the lower third will metastasize into which node?
❏ Aortic
❏ External iliac
❏ Internal iliac
❏ Inguinal
75. A patient was involved in a car accident and dislocated his left hip, damaging a nerve
and had difficulty extending his left thigh over the hip joint. Which of the following
nerves most likely was affected?
❏ Femoral nerve → innervates anterior compartment of thigh; flexion of hip
❏ Obturator nerve→ adductor of hip joint; innervates medial compartment of thigh
❏ Tibial division of sciatic nerve → innervates muscles of posterior thigh,
semitendinosus and semimembranosus, thigh extensors at hip joint
❏ Common peroneal division of sciatic nerve → no function on hip joint
76. The woman undergoes hip surgery. If all of the arteries that are part of the cruciate
anastomosis of the upper thigh are ligated, which of the following arteries maintains
blood flow?
❏ Medial femoral circumflex
❏ Lateral femoral circumflex
❏ Superior gluteal
❏ Inferior gluteal
*CRUCIATE ANASTAMOSIS - “FILM” (First perforating, Inferior gluteal, Lateral circumflex
femoral, Medial circumflex femoral)
cjhernandez/djhorneja/ccorpuz
24 year old tri-athlete has intermittent numbness from the heel radiating to the
82. A
plantar aspect of the toes. He has tingling sensation on tapping the area behind the
medial malleolus. Which nerve is probably involved?
❏ Deep peroneal nerve
❏ Superficial peroneal nerve
❏ Sural nerve → lateral malleolus
❏ Tibial nerve
atellar fractures by indirect trauma causes a transverse fracture pattern due to the
83. P
forcible contraction of which muscle?
❏ Gracilis
❏ Hamstrings
❏ Quadriceps - Rectus femoris, vastus lateralis, vastus medialis, vastus intermedius
❏ Sartorius
thoracic surgeon is going to collect a portion of the greater saphenous vein for
84. A
coronary bypass surgery. He has observed that this vein runs:
❏ Posterior to the medial malleolus → anterior to medial malleolus
❏ Into the popliteal vein → does not pass into popliteal vein bc it drains to femoral
vein
❏ Anterior to the medial condyles of the tibia and femur → located posteriorly
❏ Superficial to the fascia lata of the thigh
cjhernandez/djhorneja/ccorpuz
85. A 47-year-old woman is unable to invert her foot after she stumbled on her driveway.
Which of the following nerves are most likely injured?
❏ Superficial and deep peroneal
❏ Deep peroneal and tibial
❏ Superficial peroneal and tibial
❏ Medial and lateral plantar
*Foot invertors: Anterior and posterior tibialis, extensor digitorum longus are
*Deep peroneal nerve - anterior compartment of leg
*Foot Evertors: Peroneus longus brevis and tertius - nerve supply: common peroneal nerve
92. In what part of the respiratory system are the Clara cells seen?
❏ Trachea
❏ Bronchus
❏ Primary bronchiole
❏ Respiratory bronchiole - progenitor cells for ciliated and secretory epithelial cells
93. Which cells of the fundic gland secrete pepsinogen?
❏ Parietal cells → HCl and Intrinsic factor
❏ Chief cells
❏ Mucus neck cells
❏ Basal cells
cjhernandez/djhorneja/ccorpuz
94. Which zone in the hepatic lobule is most resistant to hypoxia?
❏ Zone 1→ near portal vein, oxidative liver function
❏ Zone 2
❏ Zone 3 → near the central vein; oxygenation is poor
❏ Zone 4 → does not exist
100. Which cell in the thyroid produces a hormone that opposes the action of
parathormone?
❏ Squamous cells
❏ Cuboidal cells → secrete T3 and T4; cuboidal=follicular cells
❏ Follicular cells → secrete T3 and T4
❏ Parafollicular cell → secrete calcitonin
cjhernandez/djhorneja/ccorpuz
MICROBIOLOGY MOCK BOARD EXAM
UST MBR 2020
cjhernandez/djhorneja/ccorpuz
7. Alpha hemolysis, Optochin positive, bile soluble and positive for Quellung test are
properties of which organism?
❏ Streptococcus pyogenes
❏ Streptococcus agalactiae
❏ Streptococcus pneumoniae
❏ Streptococcus mutans
8. Lactose fermentation differentiates:
❏ E. coli from Klebsiella → rapid lactose fermenters
❏ E. coli from Salmonella → Salmonella and Shigella are non-lactose fermenters
❏ Salmonella from Shigella → Non-lactose fermenters
❏ Neisseria meningitidis from Neisseria gonorrhea → Non-lactose fermenters; N.
meningitidis utilizes glucose and maltose; N. gonorrhea utilizes glucose only
9. Which of the following is an endotoxin?
❏ Botulinum toxin → exotoxin
❏ Diphtheria toxin → exotoxin
❏ Peptidoglycan → part of cell wall; structural support
❏ Lipopolysaccharide → cell wall of gram neg bacteria
10. Bacteria possessing this structure usually grow as mucoid colonies on media:
❏ Flagella
❏ Pili
❏ Capsule → prototype is Klebsiella pneumoniae
❏ Spore
11. Which of the following culture media is used specifically for the isolation of Neisseria
gonorrhea?
❏ Modified Thayer Martin
❏ Buffered charcoal yeast extract agar → Legionella
❏ Regan-Lowe agar → Bordatella pertussis
❏ Thiosulfate-citrate-bile salt-sucrose agar → V. Cholerae
12. Triple sugar iron contains lactose, sucrose and glucose in what proportions?
❏ 0.0486226851851852 (1:10:1)
❏ 0.417372685185185 (10:1:1)
❏ 0.423622685185185 (10:10:1)
❏ 0.0487268518518519 1:10:10
13. This organism exhibits tumbling motility at 25 deg C?
❏ Escherichia coli → also motile but no pattern
❏ Klebsiella pneumonia
❏ Campylobacter jejuni → darting motility
cjhernandez/djhorneja/ccorpuz
❏ Listeria monocytogenes
14. This test differentiates Staphylococcus aureus from Staphylococcus epidermidis:
❏ Catalase
❏ Coagulase
❏ Oxidase
❏ DNAse
15. Which organism exhibits “satellite phenomenon” on sheep blood agar in the presence of
Staphylococcus aureus?
❏ Streptococcus pyogenes
❏ Bacillus anthracis
❏ Haemophilus influenzae → S. aureus is Beta hemolytic on sheep blood agar,
releases factor V and X for H. influenzae to grow → satellite phenomenon on
sheep blood agar
❏ Corynebacterium diphtheriae
16. A 26 year old female who is on her 35th week of pregnancy went for her routine prenatal
check-up. The patient should be screened for this organism:
❏ Streptococcus pyogenes
❏ Streptococcus agalactiae → grp B hemolytic; causes neonatal pneumonia
❏ Viridans streptococci
❏ Enterococci
17. A gastric tissue biopsy was obtained from a 38 year old male patient diagnosed with
gastric ulcers was sent for culture. On the 5th day, the culture produced colonies on
Skirrow’s medium of Gram-negative curved bacilli. The colonies also test positive for urease.
The most likely organism is:
❏ Klebsiella pneumoniae → urease positive, grows on MacConkey
❏ Campylobacter jejuni → may also grow on Skirrow’s medium but is urease
negative; gram negative curved bacilli
❏ Helicobacter pylori
❏ Vibrio cholerae → comma shaped bacilli, grows on TCBS
18. An emergency physician is suspecting Corynebacterium diphtheriae when examining the
sore throat of an exchange student from Africa. The specific culture media to use for the
nasopharyngeal swab of the patient is:
❏ Chocolate agar → enriched medium specific for fastidious organisms
❏ Cystine tellurite blood agar → brown black colonies
❏ Thayer-Martin agar → Neisseria
❏ Regan-Lowe agar → Bordetella
19. A 25 year old pregnant patient complained of vaginal irritation. A Gram-stained vaginal
smear revealed many epithelial cells covered at its margins by Gram-variable coccobacilli.
The most likely cause of the vaginitis is:
❏ Neisseria gonorrhoeae → gram negative diplococci
cjhernandez/djhorneja/ccorpuz
❏ Streptococcus agalactiae
❏ Gardnerella vaginalis → “clue cells”
❏ Staphylococcus epidermidis
20. Which of the following organisms causes opportunistic infections and produces red
pigmented colonies on culture?
❏ Escherichia coli → pink in MacConkey agar
❏ Klebsiella pneumoniae → pink in MacConkey agar
❏ Proteus mirabilis → non lactose fermenter; colorless
❏ Serratia marcescens → produces red pigment called prodigiosin, late lactose
fermenter, colorless colonies on MacConkey agar and later becomes pink
colonies
21. A 45 year old male, had hamburgers and fries from a fast food restaurant in a city mall. 2
days after, he began to experience abdominal cramps and watery diarrhea, becoming
‘bloody’ in the last 24 hours. He also felt feverish and weak. He self-medicated with
antidiarrheal and antipyretic medicines but with slight to no relief. He had no history of
travel in the past 6 months. He sought consult at the emergency department for his
condition. Stool culture was done and grew E. coli. Given the history of the patient, which E.
coli type could have caused the patient’s illness?
❏ Enterotoxigenic E. coli → “Traveler’s Diarrhea”, Childhood diarrhea, profuse
watery diarrhea ETEE
cjhernandez/djhorneja/ccorpuz
24. A 57 year old male consulted for his non healing wound on his forearm, initially red and
fluid filled eventually becoming black and necrotic. There was no recalled trauma on the site
of the wound. The patient works in a farm tending to goats, sheep and other farm animals.
Culture of his wound sample showed nonhemolytic colonies with edge like matted hair,
resembling a medusa head. Gram-stain of the colonies showed large, Gram-positive,
spore-forming bacilli. The most likely isolate is:
❏ Bacillus anthracis → eschar; cutaneous anthrax
❏ Bacillus cereus → hemolytic on blood agar, food poisoning
❏ Clostridium perfringens → spore forming, anaerobic, sporulation is rarely
produced on culture; gas gangrene and necrotizing fasciitis
❏ Corynebacterium diphtheriae → gram positive non sporming bacilli, arrange in
palisades, Chinese characters, causative agent for Diphtheria
25. Which sugar/s are used by Neisseria meningitidis that differentiates it from Neisseria
gonorrhoea?
❏ Glucose and sucrose
❏ Glucose and lactose
❏ Glucose and maltose → MeninGitidis
❏ Glucose only → Neisseria gonorrhoea
26. Which zoonotic organism is the causative agent of Undulant fever?
❏
- Borrelia burgdoferi → Lyme Disease amaurosis
❏ Rickettsia rickettsia→ Rocky mountain spotted fever
❏ Pasteurella multocida → Pastuerellosis
❏ Brucella melitensis → initially present as mild flu, fever will rise and fall, also
called Brucellosis, Malta Fever or Meditteranean Fever
27. Which spirochete is transmitted to its human host by contact with infected animal urine?
❏ Leptospira interrogans
❏ Treponema pallidum → sexually transmitted
-
❏ Borrelia burgdoferi → tick bite (Ixodes tick)
❏ Helicobacter pylori → fecal oral route
28. A 5 year male was brought for consult with an inflamed wound at his right foot noted a
few days after playing with their pet cat. Gram staining of the wound showed,
Gram-negative coccobacilli with bipolar staining. Which of the following is suggestive to be
the organism in question?
❏ Pasteurella multocida
=
❏ Yersinia pestis → gram neg coccobacilli; also with bipolar staining and transmitted
by flea bite
❏ Chlamydia psittaci → exposure with birds
-
❏ Francisella tularensis → exposure to wild animals, transmitted via arthropod bite
29. A 31 year old male consulted a urologist for genital lesion that initially were small red
pustular bumps, becoming ulcerated and painful. Culture of the lesion showed
Gram-negative organisms, grew on Chocolate agar, with growth around on the X factor,
none on the V factor. The most likely organism causing the infection is:
cjhernandez/djhorneja/ccorpuz
woe
O
31. Acid fastness of Mycobacterium tuberculosis is due to the presence of:
❏ Peptidoglycan → thicker in gram positive organism that is why they retain
primary stain
❏ Teichoic acid → only found in gram positive organisms; structural cell wall
support
❏ Lipopolysaccharide → found in the cell wall of gram negative bacteria
❏ Mycolic acid → on cell wall envelope of MTB
32. Virulence of Mycobacterium tuberculosis is attributed to:
❏ Cord factor
❏ Capsule → gram negative and positive organisms if encapsulated
❏ Lipopolysaccharide → for Gram negative organisms
❏ Exotoxin → seen in gram positive organisms
33. True of Primary TB:
❏ Lesions can show tubercles with caseation and fibrosis → seen in reactivation TB
❏ Involves the lung apex → seen in reactivation TB
❏ Contains Ghon complex → presence of enlarged LN in tracheobronchial tree
❏ All of the above
34. Mycobacterium leprae is best isolated using which of the following?
❏ Lowenstein-Jensen medium → M. tuberculosis
❏ Armadillos → footpad of armadillos or mice
❏ Fletcher medium → Leptospira
❏ McCoy cells → Chlamydia
35. Lepromatous leprosy differs from the Tuberculoid leprosy wherein:
❏ There is abundant acid fast bacilli in skin lesions
❏ There is asymmetric nerve involvement → symmetric nerve involvement
❏ Shows positive for lepromin test → LL is negative on lepromin test
❏ Cell mediated immunity is intact
cjhernandez/djhorneja/ccorpuz
*Other choices are characteristics of Tuberculoid leprosy
36. Two siblings, 7 and 10 years old, presented to their pediatrician with a 2-day history of
fever, facial swelling, and muscle pain. Two weeks earlier, they attended a county fair and
consumed pork sandwiches, potato salad, and ice cream. Based on a blood smear, which
demonstrated an eosinophilia of 20%, a biopsy was obtained which showed striated muscles
with an encapsulated larva. Which one of the following is the most likely etiologic agent?
❏ Taenia saginata
❏ Wuchereria bancrofti
0
❏ Trichinella spiralis → Trichinosis; “porkworm” and is transmitted via ingestion of
undercooked pork
❏ Trichuria trichuria
37. A 56-year-old female patient presents complaining of fatigue 11 months after receiving a
heart transplant. The organ donor emigrated from South America 15 years previously. An
endomyocardial biopsy was performed, and the histologic results show amastigotes and
kinetoplasts. Which one of the following pathogens is shown?
❏ Leishmania braziliensis
❏ Listeria monocytogenes
❏ Toxoplasma gondii
❏ Trypanosoma cruzi → trypomastigotes (blood) and amastigotes (muscle biopsy)
38. Which one of the following is the insect vector responsible for transmitting the organism
showing a peripheral smear with banana shaped gametocytes?
❏ Tsetse fly → Trypanosoma brucei (African sleeping sickness)
❏ Anopheline mosquito
❏ Reduviid bug → Trypanosoma cruzi (Chagas disease/American sleeping sickness)
❏ Black fly → Onchocerca volvulus
39. A 12-year-old bone marrow transplant recipient presented to the emergency department
with nonspecific influenza-like symptoms, including fever, chills, and myalgia. Findings on
the smear showed RBCs infected with multiple ring forms which sometimes appear as a
tetrad or “Maltese cross”. Relevant past medical history includes a blood transfusion 1
month before admission and no recent travel, although his parents confirmed a trip to the
Ivory Coast 3 years ago. Which one of the following is the etiologic agent?
❏ Plasmodium falciparum
❏ Toxoplasma gondii
❏ Plasmodium ovale
❏ Babesia microti
cjhernandez/djhorneja/ccorpuz
40. A 50-year-old pig farmer, a recent immigrant to the United States from Indonesia,
presented to his local emergency department with a 3-week history of diarrhea.
Examination of hisO stool demonstrated a large, ciliated pathogen. Which one of the following
is the etiologic agent?
❏ Chilomastix mesnili
❏ Enterocytozoon species
❏ Balantidium coli → only ciliated protozoan; reservoir: swine
❏ Iodamoeba butschlii
*Rest of the organisms are non-ciliated
41. A 40-year-old man, recently returned from China, has experienced abdominal discomfort
of 1 month’s duration. He was subsequently found to have biliary duct obstruction. Stool
examination demonstrated the ovum showing thick yellow-brown shell, and has an
operculum with opercular shoulders. Which one of the following trematodes is the etiologic
agent of his illness?
❏ Schistosoma haematobium → Blood fluke; has terminal spine
❏ Fasciola hepatica → Sheep liver fluke
❏ Fasciolopsis buski → Giant intestinal fluke; eggs are broadly ellipsoid
❏ Clonorchis sinensis → Chinese liver fluke, causes Cholangiocarcinoma
42. Which one of the following parasites is a liver fluke and infects the biliary ducts of
humans?
❏ Clonorchis species
gigantean
❏ Diphyllobothrium latum → flatworn "
cjhernandez/djhorneja/ccorpuz
❏ The genus Echinococcus contains four species for which humans are an accidental
host → Echinococcus is also called dog tapeworm, dog is final host and humans
are accidental host only
❏ Immunodiagnostic tests are not useful in diagnosing hydatid disease because of
poor sensitivity resulting from the inability of antigen to be released from the cysts.
45. A 4-year-old girl presented with complaints of severe perianal itching. Other children
attending her day care setting had similar symptoms. Stool examination showed a D shaped
egg. Which one of the following is the cause of this patient’s infection?
❏ Enterobius vermicularis
❏ Ascaris lumbricoides → mamillated eggs
❏ Ancylostoma duodenale
❏ Necator americanus
46. A 21-year-old female college student presented to the student clinic complaining of a
1-week history of diarrhea and stomach cramps. She had traveled to rural Central America
for humanitarian purposes 1 month before presentation. Direct examinations of stained
fecal material show trophozoites with ingested RBCs in the cytoplasm and thin-walled cysts
containing two nuclei and a chromatoidal bar. Which one of the following is the etiologic
agent responsible for this patient’s gastrointestinal disease?
❏ Entamoeba coli →nonpathogenic, cyst has 8 or morenuclei; chromatoidal bodies
have splintered or pointed ends
❏ Blastocystis hominis → commensal, non-pathogenic, but in immunocompromised
may cause diarrhea
❏ Iodamoeba butschlii → cyst and trophozoite has large glycogen vacuole
❏ Entamoeba histolytica → cyst have 4 nuclei and chromatoidal bodies are
blunted; pathogenic
47. Many blood-borne pathogens are transmitted by an insect (vector). Which of the
following parasites is not vector transmitted?
❏ Toxoplasma gondii → transmitted via ingestion of oocyst of feces of cats; or
ingestion of raw meat; or transplacental
❏ Plasmodium species → via female Anopheles mosquito
❏ Leishmania species → via sandfly
❏ Trypanosoma cruzi → Reduviid bug/kissing bug
48. A 50-year-old woman presented to her primary care physician with complaints of a 2-
week history of chronic cough and hemoptysis. She had traveled through rural parts of China
during the previous month. Sputum examinations demonstrated 80 to 120 _m ovum with
thick brown-yellow shell, and an operculum. Which one of the following parasites is the
most likely etiologic agent of this patient’s illness?
❏ Fasciolopsis buski → Giant intestinal fluke
❏ Fasciola hepatica → Common/Sheep liver fluke
❏ Paragonimus westermani → Lung fluke
❏ Clonorchis sinensis → Chinese liver fluke, affects biliary tree also
49. A 25-year-old woman presented to her physician with a 2-week history of fever, chills,
and headache. Physical examination was significant for splenomegaly. Peripheral smear
cjhernandez/djhorneja/ccorpuz
showed delicate ring forms with stippling in the cytoplasm (Schuffner dots) and ameboid
trophozoites. Which one of the following is the most likely etiologic agent in this case?
❏ Plasmodium vivax → along with P. ovale have Schuffner dots, causes relapse
due to hypnozoites
❏ Plasmodium falciparum → ring forms seen in RBC
❏ Ehrlichia chaffeensis → obligate intracellular gram negative causes Rickettsia
❏ Babesia microti → maltese cross appearance
50. Which of the following trematodes has an ovum with a lateral spine?
❏ Schistosoma hematobium → terminal spine
❏ Schistosoma mansoni
❏ Fasciola hepatica
❏ Schistosoma japonicum→ lateral knob
51. Some parasites are causative agents in transfusion-related infections. Which one of the
following parasites is not transmitted by blood transfusion?
❏ Babesia microti
❏ Trypanosoma cruzi
❏ Toxoplasma gondii
❏ Strongyloides stercoralis → penetration of filariform larvae in skin of human host
52. A patient with HIV/AIDS presented to the hospital with a 2-week history of headaches,
weakness, and slurred speech. The results of a computed tomography scan with contrast
and a magnetic resonance imaging revealed multiple ring-enhancing lesions. Tissue from a
brain biopsy was negative on an acid-fast stain. Serodiagnostic tests were positive for the
etiologic agent. Which one of the following is the most likely infectious cause of the brain
lesions in this patient?
❏ Toxoplasma gondii → immunocompromised patients; presents with cerebral
mass lesions, encephalopathy; dx: Toxoplasma IgG, IgM and Double sandwich
ELISA test
❏ Cryptosporidium parvum → acid fast, causes GI symptoms
❏ Mycobacterium tuberculosis → acid fast
❏ Entamoeba histolytica → causes extra-intestinal symptoms but the trophozoite
must be seen in the lesion
53. Which of the following is a key characteristic of the thick blood film?
❏ It allows the parasite to be seen in the RBCs
❏ It allows identification of parasites to the species level
❏ Less blood must be examined than with a thin blood film
❏ RBCs must be lysed for more efficient detection of parasite → drop of water
causes lysis of rbc releasing parasite
54. Entamoeba dispar is most easily confused morphologically with:
❏ Entamoeba coli
❏ Entamoeba hartmanni
cjhernandez/djhorneja/ccorpuz
❏ Dientamoeba fragilis
❏ Entamoeba histolytica
*E. dispar is nonpathogenic amoeba
55. Diseases caused by Trypanosoma brucei gambiense and Trypanosoma brucei rhodesiense
have distinct differences, which include:
❏ The morphology of the trypomastigotes, which looks very different
❏ The symptoms and severity of the overall illness → T. brucei rhodiense have
rapidly progressive acute febrile illness with lymphadenopathy may die
before CNS symptoms are prominent; T. brucei gambiense - present as
classical African sleeping sickness and are more chronic
❏ The presence or absence of the undulating membrane in the trypomastigote
❏ The totally different continents in which they appear
56. A 12-year-old girl is brought to the emergency department with meningitis and a history
of swimming in a warm-water spring. Motile amebae that measure 10 μm in size are seen in
the CSF and are most likely:
❏ Iodamoeba butschlii trophozoites
❏ Endolimax nana trophozoites !
57. Eye infections with Acanthamoeba spp. have most commonly been traced to:
❏ Use of soft contact lenses
❏ Use of hard contact lenses
❏ Use of contaminated lens care solutions → corneal infection
❏ Failure to remove lenses while swimming
58. Which of the following roundworms is capable of hyperinfection, and a heavy worm
burden with characteristic larval migration required in the life cycle?
❏ Ancylostoma duodenale
❏ Enterobius vermicularis
❏ Strongyloides stercoralis
❏ Ascaris lumbricoides
59. The filariform larva is the infective stage for:
❏ Ascaris lumbricoides, Trichostrongylus spp → Ascaris: embryonated egg
❏ Ascaris lumbricoides, Strongyloides stercoralis
❏ Ancylostoma duodenale, Strongyloides stercoralis → hookworms
❏ Trichuris trichiura, Necator americanus → infective stage is egg for Trichuris
trichiura
60. Removal and gradual retraction of the adult gravid female worm is recommended in
infections with:
❏ Ancylostoma braziliense
cjhernandez/djhorneja/ccorpuz
❏ Dracunculus medinensis → female worm causes eruption/blister in LE →
contact with water releases egg; worm is wound in a stick however complete
surgical removal is definitive
❏ Trichinella spiralis
❏ Toxocara cati
61. Lymphatic vessel involvement within the genital region is associated with infection with:
❏ O. volvulus → worms deposit within skin or subcutaneous tissues
❏ B. malayi → usually involve extremities
❏ W. bancrofti → involve genital area
❏ Loa loa → eye infection/ calabar swelling
62. Which tapeworm cannot be identified to the species level based on its egg morphology;
instead, proglottids must be examined?
❏ Diphyllobothrium
❏ Dipylidium
❏ Hymenolepis
❏ Taenia → Taenia solium and saginata eggs are similar; Taenia solium - 13 gravid
proglottids or lateral uterine branches, cup shape suckers with arm rostellum;
Taenia saginata without rostellum and with 20 or more lateral uterine branches
63. Which of the following is not an appropriate prevention measure for cestodes?
❏ Controlling the flea/rat population
❏ Avoiding the consumption of raw meat or fish
❏ Immunization
❏ Practicing good hygiene
64. What intermediate host is required in the life cycle of all trematodes?
❏ Freshwater snail - 1st intermediate host
❏ Crayfish
❏ Aquatic vegetation
❏ Freshwater crab
65 .The drug of choice for an intestinal trematode infection is:
❏ Niclosamide
❏ Praziquantel
❏ Albendazole → roundworm infection
❏ Triabendazole
66. What laboratory finding may be observed in Paragonimus infections?
❏ Increased number of eosinophils
❏ Decreased serum bilirubin level
❏ Decreased vitamin B12 level → Diphyllobotrium latum
cjhernandez/djhorneja/ccorpuz
❏ Increased number of red blood cells → no adult worm causes
this
67. The mode of transmission of schistosomal infection is by:
❏ Ingestion of contaminated aquatic vegetation → liver flukes
-
❏ Direct penetration of the skin by cercariae
❏ Ingestion of raw fish → Capillaria philippinensis
❏ Mosquito bite → Wuchereria and Plasmodium
68. Pathologic effects of filariae in humans are caused by the adult worms in all but one
species. In this case, the principal damage is caused by the microfilariae of:
❏ Brugia malayi
❏ Dracunculus medinensis
❏ Wuchereria bancrofti
❏ Onchocerca volvulus → migration of microfilariae into eye → blindness
69. The principal reservoir for the antigenic shift variants of influenza virus:
❏ Rodents
❏ Pigs, Fowl
❏ Chronic human carriers of the virus
❏ Mosquitoes → not reservoir, only vector
70. This virus is the most common cause of bronchiolitis in infants:
❏ Measles virus
❏ Influenza virus
❏ Respiratory syncytial virus
❏ Adenovirus
71. SARS is caused by which of the following virus?
❏ Influenza virus
❏ Adenovirus
❏ Coronavirus → SARS belong to beta-coronaviruses, cause pandemic for last 2
decades
❏ Parainfluenza virus → Croup
72. Which of the following is NOT associated with prions?
❏ Creutzfeldt-Jakob disease
❏ Kuru
❏ Progressive multifocal encephalopathy --> caused by JC Virus
❏ Bovine spongiform encephalopathy
73. This virus is responsible for Kaposi sarcoma and vascular tumors of mixed cellular
composition:
❏ Human Herpesvirus 6 → Roseola infantum
cjhernandez/djhorneja/ccorpuz
❏ Human Herpesvirus 8→ Kaposi is usually seen in immunocompromised (HIV)
❏ Human Herpesvirus 10
❏ Human Herpesvirus 9
E
❏ Rdrp protein → largest protein detected within viral capsule
❏ S protein → spike protein; detected in rapid antibody kits
❏ M protein → membrane protein
❏ All of the above → target in testing
76. Which of the following is a DNA virus?
❏ Poliovirus
❏ Calicivirus
❏ Retrovirus
❏ Hepadnavirus → Hepa B virus; the only DNA hepatitis virus
77. Which of the following statement/s is/are TRUE?
❏ Viruses have both DNA and RNA as genomes → either DNA or RNA only
❏ All RNA viruses are infectious
❏ All viruses are enveloped
❏ DNA viruses replicate in the nucleus → generally all will replicate in nucleus
EXCEPT Poxvirus that replicate in cytoplasm
78. The viral envelope is:
❏ Transcribed from the envelope gene of the viral genome
❏ Translated from the viral messenger RNA
❏ Obtained from the host cell → bud off a part of membrane host cell from Golgi
apparatus
❏ Acquired from the in-vitro culture medium
79. The viral capsid is:
cjhernandez/djhorneja/ccorpuz
❏ Used as receptor for the entry of the enveloped virus into the host cell
❏ Protective coat of the genome
❏ Composed of glycoproteins
❏ Seen in the helical viruses only
80. The most prevalent type of viral hepatitis is:
❏A
❏B
❏C
❏D
81. A chronic hepatitis B carrier is serum HBsAg positive on 2 occasions that are how many
months apart?
❏2
❏4
❏ 6 → chronic hepatitis is persistent elevation of HBsAg
❏ 10
82. During the “window phase” of acute hepatitis B infection, which one of the following
markers could be reactive?
❏ HBsAg
❏ Anti- HBs
❏ HBeAg
❏ Anti-HBc → 2 periods that are window periods: 1) conversion from HbsAg ot
Hbs Antibody
1) Antigens to form from earliest to last: HbsAg, HbeAg
2) Antibodies to form from earliest to last: anti-HBc → anti- Hbe → anti-HBs
83. Hepatitis D virus is dependent on this hepatitis virus for replication:
❏A
❏B
❏C
❏ D → defective virus
84. Which of the following is MOST CORRECT regarding delta agent?
❏ It is a defective mutant of HBV
❏ It is dependent on HBV surface antigen for virion formation
❏ It is related to HCV
❏ It contains a circular DNA genome
85. A 36 year old nurse is positive for both HBsAg and HBeAg. The nurse most likely
❏ Has acute hepatitis and is infectious
❏ Has both HBV and HEV infections
cjhernandez/djhorneja/ccorpuz
❏ Has a chronic HBV infection
❏ Has cleared a past HBV infection → (+) anti-HBs
*(+) HBsAg may mean an acute hepatitis infection or patient is immunized, so look into
the full hepatitis profile of patient
86. Which of the following is the most reasonable explanation for the ability of HBV to cause
chronic infection?
❏ Infection does not elicit the production of antibody
❏ The liver is an “immunologically sheltered” site
❏ Viral DNA can persist within the host cell → more than 6months (HBV infects
hepatocytes)
❏ Many humans are immunologically tolerant to HBsAg
87. The most common cause of bronchiolitis and pneumonia in infants less than 1 year old
is?
❏ Parainfluenza virus 1
❏ Human metapneumovirus
❏ Respiratory syncytial virus
❏ Rubella virus
88. Mumps occurs endemically in these areas or localities:
❏ Tropical countries
❏ Subtropical countries
❏ Urban areas
❏ Worldwide → No endemic area, late winter/early spring; member of
Paramyxovirus family; via respi droplets
89. Atypical measles is characterized by this/these types/s of lesion/s:
❏ Vesicles
❏ Petechiae
❏ Urticaria
❏ All of the above → Atypical measles is composed fever, unusual rash and
pneumonia
90. One of the following is a common sequel of congenital rubella:
❏ Jaundice
❏ Hearing loss → most common; other encephalitis, splenomegaly, miscarriage,
TTP; TRIAD OF CONGENITAL RUBELLA: Cataract, PDA, Deafness
❏ Hepatitis
❏ Cloudy cornea
91. Which of the following is NOT characteristic of Paramyxovirus?
❏ Envelope contains a glycoprotein with fusion activity
❏ They do NOT undergo genetic reassortment
cjhernandez/djhorneja/ccorpuz
❏ Replication occurs in the cytoplasm of susceptible cells
❏ Genome is segmented
*RNA viruses; Mumps, Measles, RSV, Parainfluenza virus and undergoes genetic
reassortment
92. Several paramyxovirus can cause pneumonia in infants and children. Which of these has
an effective vaccine to prevent pneumonia?
❏ Respiratory syncytial virus
❏ Measles virus → vaccine is not used to prevent measles but prevent severe lesions
outside respiratory tract
❏ Metapneumonia virus
❏ Parainfluenza virus
93. Mycetoma commonly involves which part of the human body?
❏ Nose/nasal passages
❏ Ears
0
❏ Feet → also in upper extremities and back; formation of sinus tracts
❏ Trunk
94. Which of the following statements regarding dematiaceous fungi is TRUE?
❏ They come from the soil→ brown pigment fungi, assoc with plants; causes
chromoblastomycosis, mycetoma
❏ They are easily treated
❏ They are dimorphic
❏ They are aseptate and clear
95. Two basic growth forms of fungi:
❏ Yeasts and pseudomolds
❏ Molds and pseudoyeasts
❏ Yeasts and molds → yeast in heat, mold in cold
❏ Yeasts and pseudoyeasts
96. Crosswalls that divide hyphae into cells:
❏ Conidia
❏ Septa
❏ Thallus
❏ Germ tube
97. Which of the following fungi produce only yeast colonies both at room temperature and
incubation temperature?
❏ Cryptococcus neoformans → exist in yeast only
❏ Sporothrix schenkii
❏ Histoplasma capsulatum
❏ Blastomyces dermatitidis
cjhernandez/djhorneja/ccorpuz
*Rest of the choices are dimorphic, convert from mold to yeast by incubating culture media
at 25-37C
98. Which of the following causes superficial mycosis?
❏ Coccidioides immitis
❏ Histoplasma capsulatum
❏ Malassezia furfur → commensal fungi causing dermal mycosis; spaghetti and
meatball apppearance
❏ Cryptococcus neoformans
99. The route of transmission to man of systemic mycosis:
❏ Skin
❏ GIT
❏ Respiratory
❏ Hematogenous
100. A 40 year old complained of gritty hair without accompanying alopecia. Direct
microscopic examination of the hair shaft revealed blackish nodule. What is the causative
agent?
❏ Microsporum spp
❏ Hortae werneckii → Tinea nigra
❏ Piedraia hortae → Black piedra
❏ Trichosporon spp
cjhernandez/djhorneja/ccorpuz
Drive Mag-sign in
FC
Mga File
1. A baby girl was born by emergency CS due to fetal distress. At 1 minute of life she was
limp & cyanotic all over, with weak unsustained cry & grimace on stimulation. CR=
90/min, What is her APGAR score?
❏2
❏ 3 → severely depressed
❏4
❏5
3. The components & proper sequence for essential newborn care (EINC) are:
❏ skin to skin contact → drying of the infant→ cord clamping→ early
breastfeeding
❏ bath in lukewarm water→ drying of the infant→ skin to skin contact→ cord
clamping
cjhernandez/djhorneja/ccorpuz
❏ drying of the infant→ skin to skin contact → cord clamping →
breastfeeding
❏ cord clamping→ skin to skin contact→ drying of infant → Hepa B vaccination
cjhernandez/djhorneja/ccorpuz
❏ Only HC is below p10 → microcephaly
❏ Only Wt is below p10
9. A 1 year old infant is expected to have the ff anthropometric data if his birth weight
was 3 Kg, length (L) was 50 cm and head circumference (HC) was 35 cm:
❏ 6 kg - 70 cm - 50 cm
❏ 7.5 Kg - 75 cm - 45 cm
❏ 7.5 Kg - 80 cm - 50 cm
❏ 9 Kg - 75 cm - 45 cm → 1 yr old: weight is tripled, Length increases by 50%
and HC increases by 10cm from birth
10. Among female adolescents, areola & the papillae form a secondary mound on the
breast at which SMR stage?
❏ 2 → breast and areola form small mound
❏ 3 → breast and areola enlarged, no contour separation
❏4
❏ 5 → mature nipple projects, areola part of general breast contour
11. Peak height velocity among female adolescents occurs during which SMR stage?
❏ 1-2
❏ 2-3 → 8-9cm /yr, 6 months before menarche
❏ 3-4 → for boys, accdg the adolescence chapter
❏ 4-5→ for boys, the puberty chapter
12. Calculate for the mid-parental height in cm for a son whose Mom stands 160 cm and
Father 173 cm.
❏ 160
❏ 167
❏ 170
❏ 173
13. Emergence of abstract thought or formal operations start from this period?
❏ Preadolescent
❏ Early → concrete operation
❏ Middle → abstract or logical thinking
❏ Late → absolutism, idealism
cjhernandez/djhorneja/ccorpuz
14. What is the main manifestation of Pickwickian syndrome seen in markedly obese
individuals?
❏ Cor pulmonale → pulmonary pathology → pulmonary hypertension → RV
failure, ie. Bronchiectasis, BPD
❏ Hypoventilation → Hypoxemia+hypercapnia → pulmonary hypertension
❏ Eisenmenger syndrome → right to left shunt in chronic VSD with Pulmonary
hyperventilation
❏ Narcolepsy → chronic sleep disorder characterized by daytime drowsiness and
sudden attacks of sleep even at daytime
*Pickwickian → there is hypoventilation syndrome
15. A 9 month old male developed epileptic spasms especially when drowsy or on waking
up, developmental regression since 6 mo old. EEG shows high voltage, slow chaotic
background with multifocal spikes. Most likely diagnosis is:
❏ West syndrome → Hypsarrhythmia; idiopathic/cryptogenic: with normal
development before onset; symptomatic: with developmental delay before
onset
❏ Lennox Gastaut syndrome → onset is 2-10 y/o; TRIAD: dev regression, multiple
seizure types; 1-2 hz spike-and-slow waves, polyspike bursts in sleep and a slow
background in wakefulness
❏ Dravet’s syndrome → Start as focal febrile status epilepticus or focal febrile
seizures → myoclonic or other seizure types
❏ Progressive myoclonic epilepsies → progressive dementia and worsening
myoclonic and other seizures
16. A mother developed varicella lesions 3 days before the delivery of her baby. The best
approach for the newborn infant is to:
❏ Start Acyclovir
❏ Give Varicella vaccine
❏ Give Varicella zoster immune globulin
❏ Isolate from Mom & observe only
*Earlier than 5 days → mother have enough antibodies to be passed to the baby
17. The single most common finding in infants with congenital rubella syndrome:
❏ PDA
❏ Cataract
❏ Nerve deafness
❏ Neonatal purpura
cjhernandez/djhorneja/ccorpuz
18. A 10 month old male had persistent moderate to high fever for 3 days. After
defervescence, there appear faint pink 2-3 mm morbilliform rash on the trunk. What is
the most likely etiologic agent?
❏ Rubella → German measles (3rd disease); headache, malaise, sore throat, red
eyes, lymphadenopathy → rash on face, neck, torso, extremities & Forschheimer
spots at uvulopalatoglossal (oropharynx)
❏ Rubeola → Measles (1st Disease); Koplik’s spots → exanthem from hairline to
the neck then extremities & during recovery leads to branny desquamation; High
fever and 3 Cs (cough, coryza, conjunctivitis)
❏ Human Herpesvirus 6 → (HHV6/7) Roseola infantum/ Exanthem subitum/ 6th
disease, ulcers at uvulopalatoglossal junction (Nagayama spots)
❏ Parvo B19 → Erythema Infectiosum (5th Disease); low fever, coryza, facial flush
(slapped cheek appearance) → spread lacy reticulated appearance; transient
aplastic crisis in patients with hemolytic diseases
19. What is the drug of choice for staphylococcal toxin mediated illnesses because it can
also inhibit toxin production?
❏ Cefazolin
❏ Oxacillin → for MSSA
❏ Vancomycin → for MRSA
❏ Clindamycin → MRSA that does not present with meningitis, sepsis
20. An unimmunized 3 month old infant presents with fever of 39-41C of 1 day duration
without localizing sign. His urinalysis is normal. If he has occult bacteremia, what is the
most likely etiologic agent?
❏ H. influenza B
❏ S. pneumoniae→ pneumonia, otitis media, sinusitis, meningitis in children; At
risk: sickle cell disease, asplenia, deficiencies in B cell and complement, HIV,
malignancies, chronic heart, lung and renal disease, cochlear implants
❏ N. meningitides
❏ S. aureus
21. A 15 y/o presents with sudden onset of right chest pain on inspiration, low to
moderate grade fever, shortness of breath, decreased breath sounds & dullness on the
right lung base. Chest X-ray shows pleural effusion. Thoracentesis yielded
serosanguineous fluid with sp. gr. 1.020, protein 4 mg/dL, glucose 20 mg/dL, wbc 200
pus cells/hpf with lymphocytic predominance. The most likely etiology is:
❏ S. pyogenes
❏ S. pneumoniae
❏ M. pneumoniae
❏ M. tuberculosis → low glucose, high protein and lymphocytic
predomenance
cjhernandez/djhorneja/ccorpuz
22. The presence of this finding in a patient with invasive meningococcal disease carries a
better prognosis:
❏ Hyperpyrexia
❏ Elevated CRP
❏ Leukocytosis
❏ Meningitis
23. Enterococci and Listeria are usually resistant to:
❏ Ampicillin → Drug of choice for Listeria
❏ Penicillin
❏ Imipenem
❏ Cephalosporins → Ampicillin is added to cover for Listeria
24. An unimmunized 4 y/o child developed sore throat the past few days with dysphagia,
hoarseness, malaise & headache. PE: ToC = 37, CR = 130, RR 40, markedly swollen neck
with markedly enlarged cervical & submandibular lymph nodes. There were thick
membranes covering the tonsils and uvula. What should be the treatment regimen to
prevent fatality and transmission of this condition to
others?
❏ Penicillin
❏ Erythromycin
❏ Clindamycin
❏ Specific antitoxin + Penicillin
*Diagnosis: Diphtheria which is mediated by diphtheria toxin
0
25.When is the best time to give Tdap to pregnant women to prevent neonatal pertussis?
❏ Before 8 weeks of gestation
❏ 8/12/20
❏ 13 - 26
❏ 27 - 36 → anytime will do but BEST time is at 27-36 weeks AOG
cjhernandez/djhorneja/ccorpuz
26. A malnourished 5 y child is screened for PTB by TST. He has BCG scar on his right
deltoid. His TST is considered (+) if the reaction is at least how many mm?
❏ 5 → in immunocompromised individuals
www.w.int
❏8
❏ 10
❏ 12
27. A 2 y child with FTT had 4 wks history of recurrent afternoon low to moderate grade
fever and occasional dyspnea especially at night for the past 2 wks. Chest x-ray shows
multiple stippling small shadows over both lung fields. He most likely has:
❏ Miliary TB → charac X-ray: multiple stippling small shadows over both lung
fields
❏ Viral pneumonia
❏ Pneumocystis jiroveci
❏ Streptococcal pneumonia
28. A 1 y/o asymptomatic boy had routine tuberculin skin test which yielded a 12 mm
induration at 48 hours. Chest X-ray is clear. Family history is (-). What is the treatment
of choice?
O
❏ Give INH for 9 months → Latent TB/TB infection [(+)TST, asymptomatic]
❏ Observe and repeat CxR in 3 months
❏ Give HRZ first 2 months then HR 4 months more
❏ Give HRZE first 2 months then HR 4 more months → TB disease
29. A 2 y/o presents with respiratory distress & inspiratory stridor preceded by coryza.
X-ray of the neck reveals (+) Steeple sign. What is the most likely etiology for his
condition?
❏ RSV → bronchiolitis
❏ S. aureus
❏ Adenovirus → pharyngoconjunctival fever
❏ Parainfluenza virus → Croup or LTB
30. Peak incidence of pneumonia due Mycoplasma pneumoniae occurs in this age group:
❏ Infancy
❏ Toddlers
❏ School age
❏ Adolescents
31. Which inhalational drug shortens the course of illness in children with croup?
❏ Salbutamol
❏ Salbutamol + Ipratropium
cjhernandez/djhorneja/ccorpuz
❏ Budesonide → O
steroids shorten the course (Dexamethasone pO/IV may also
be given)
-
:
32. This congenital lung anomaly is often mistaken for diaphragmatic hernia:
❏ Bronchogenic cyst → arise from abnormal budding other tracheal diverticulum
of the foregut before the 16th week of gestation, more commonly found on the
right and near a midline structure
❏ Congenital pulmonary airway malformation (CPAM) → hamartomatous or
dysplastic lung tissue mixed with normal lung, generally in 1 lobe
❏ Pulmonary sequestration → congenital anomaly located intra (>extra)
pulmonary with blood supply from systemic arteries; space occupying lesion
and does not connect bronchi and does not function in gas exchange
❏ Pulmonary hypoplasia → decrease in numbers of alveoli and airway
generations; usually secondary to intrauterine complications like
oligohydramnios and CPAM
33. A 14 y/o student developed chills, high grade fever, productive cough expectorating
rusty sputum, chest pain and vomiting. PE reveals decreased breath sounds in right
lung base with bronchial breath sounds on right mid chest, dullness on percussion &
increased fremitus. There is splinting of respiratory movement on the right. The most
likely etiologic agent for pneumonia in this patient is:
❏ GABHS
❏ Klebsiella pneumoniae
❏ Staphylococcus aureus
❏ Streptococcus pneumoniae
hat is the drug of choice for an intubated PICU patient with severe pneumonia,
34. W
pneumatoceles and empyema covering 3/4 of R lung on CxR?
❏ Vancomycin + oxacillin → used when patient is high risk for MRSA with life
threatening condition
❏ Clindamycin → community acquired MRSA that is not life threatening
❏ Chloramphenicol
❏ Oxacillin + Gentamicin → for MSSA
35. The gold standard for diagnosing bronchiectasis:
❏ MRI
❏ Chest X-ray
❏ High resolution CT scan
❏ Bronchoscopy & Bronchography
lex, 5 y/o, male had 2 days persistent high fever with anorexia & malaise. If he has
36. A
Dengue, what will be the usual sequence of events that leads to severe Dengue?
❏ Low platelet → low wbc → high Hct → plasma leak → narrow pulse pressure
O
❏ Low wbc → low platelet → plasma leak → high Hct → narrow pulse pressure
cjhernandez/djhorneja/ccorpuz
❏ Low platelet → plasma leak → low wbc → narrow pulse pressure → high Hct
❏ High Hct → low platelet → low wbc → narrow pulse pressure → plasma leak
lex eventually presents to the ER afebrile with platelet count of 50,000/mm3, wbc
37. A
2,000/mm3, BP 80/60. What should be your initial treatment for Alex if he weighs 20
kg?
❏ Plasma transfusion at 200 mL over 30 min
❏ Platelet transfusion
❏ D5LRS 1,500 mL/24 hr
❏ Plain NSS 400 mL over 30 min → 20ml/kg PNSS over 20-30mins
*SBP 2x5 + 70 = 80 (SBP not hypotensive but patient presents with narrowed pulse
pressure and impending shock
38. A middle child aged 5 y/o drinks a lot of juice and water and frequents the bathroom
many times at home and in your clinic. Urinalysis reveals sp. gr. of 1.001 with no other
abnormalities. What can you do to determine if his polydipsia and polyuria are
physiologic responses?
❏ Serum Na+ level → or serum osmolality
❏ Aldosterone level
❏ DDAVP challenge
❏ Atrial natriuretic peptide level
39. A male is noted to have early onset hypotonia, cryptorchidism, & micropenis, mental
retardation, hyperglycemia & obesity. He most likely has this syndrome:
❏ Turner → 45XO, short stature, webbed neck, coarctation of aorta, ovarian
hypofunction
❏ Edward’s → Trisomy 18; SGA, small head and jaws; clenched fist, overlapping
fingers, rocker bottom feet, CHDs and early death
❏ Klinefelter → 47XXY, with infertility and small testes, usually taller, hypotonia,
poor coordination and learning disability
0
❏ Willi-Prader → H4O: hypotonia, hypogonadism, hypomentia, hyperglycemia
and obesity
13 y/o girl presents with enlarged thyroid gland graded 1b. She had menarche 2
40. A
months before and has no other complaints. The rest of the PE is (-). Labs showed
Elevated TSH, Normal FT3 & FT4. She most likely has:
❏ Graves disease → Thyrotropin receptor stimulating antibody → receptor for
TSH causing Thyroid hyperplasia and unregulated production of thyroid
hormone; emotional lability, inc sympathetic activity, ophthalmopathy; low
TSH, high FT3 and FT4; ± anti-TPO
❏ Riedel thyroiditis → cause is unknown; pathology: dense fibrosis, usually (+)
TPO but euthyroid
cjhernandez/djhorneja/ccorpuz
❏ Chronic lymphocytic thyroiditis → Hashimoto’s/autoimmune thyroiditis, most
common cause of thyroid disease in children, most common cause of acquired
hypothyroidism, anti-TPO and anti-Tg Abs (+) in > 90%
❏ Subacute granulomatous thyroiditis → De Quervains disease; rare in pedia;
URTI → tenderness over thyroid with low fever → hormone leak →
hyperthyroid → hypo -> euthyroid
*Inc TSH with normal FT3 and FT4 → Subclinical hypothyroidism
41. This increases the risk for cerebral edema when given to patients with T1DM in DKA:
❏ Na+
❏ K+
❏ NAHCO3 → causes hypokalemia
❏ Phosphates
9 y/o boy admitted at PICU due to traumatic brain injury developed polyuria
42. A
associated with hyponatremia & elevated urinary Na+ excretion. The most likely
diagnosis is:
❏ SIADH
❏ Neurogenic DI → absence of ADH
❏ Nephrogenic DI → resistance to ADH receptors even in high levels of ADH
❏ Cerebral salt wasting → increased secretion of ANP seen in primarily with CNS
disorders including brain tumors, head trauma, hydrocephalus, neurosurgery
cjhernandez/djhorneja/ccorpuz
43. The most common cause of congenital primary hypothyroidism is:
❏ Dyshormogenesis
❏ Thyroid dysgenesis
❏ Defect in iodine transport
❏ Defects in organification and coupling
44. I n the absence of newborn screen, the most notable subtle sign of hypothyroidism
soon after birth is:
❏ Lethargy
❏ Constipation
❏ Abdominal distention
❏ Widely open fontanels
45. Corticosteroids may be considered an adjunctive therapy in the ff pulmonary TB cases
EXCEPT:
❏ Miliary TB
❏ Cavitary TB
❏ Endobronchial TB
❏ Pleural effusion causing respiratory distress
*TB meningitis also indicated with steroids
46.The most common cause of Extrapyramidal CP:
❏ Infection
❏ Asphyxia
❏ prematurity
❏ Kernicterus → 2nd most common cause of CP
47. A previously well 6 y/o boy developed behavioral changes and language regression
with moderate to high fever for the past few days. A few hours prior to admission, he
developed generalized tonic clonic seizure with upward rolling of eyeballs for about 5
minutes. Opening pressure on lumbar tap was 230 mm H2O. CSF analysis showed 80
lymphocytes/hpf, protein of 35 mg/dL, CSF sugar 70% of RBS. The most likely etiologic
agent is:
❏ Viral meningitis
❏ Viral encephalitis → Japanese B encephalitis; Enterovirus
❏ TB meningoencephalitis
❏ Bacterial meningoencephalitis
*protein and sugar are normal with lymphocytic predominance
48. Bartter syndrome is characterized by:
❏ Hypokalemia
❏ Metabolic acidosis
cjhernandez/djhorneja/ccorpuz
❏ Low urinary chloride
❏ Hypocalciuria & hypomagnesemia
49. A child who presents with a left peripheral facial palsy and right sided hemiparesis
with UMN signs most likely has a lesion at this level of the neuraxis:
❏ Cerebrum
❏ Brainstem
❏ Spinal cord
❏ Peripheral nerves
50. A 10 y/o girl has declining grades which is out of her character. Her teacher noted her
having staring spells associated with chewing movements a nd picking on her clothes
lasting for a minute. She most likely has:
❏ Atonic
❏ Absence seizure → last seconds only; blank stares
❏ Myoclonic seizure
❏ Focal seizures with impaired awareness → >30 seconds with automatism
51. A 2 y/o boy developed progressive ataxia and increasing head size in the last 4
months. On PE, HC z-score is above +2, wt and length are below zero, (+) titubation,
inability to ambulate but MMTs were 4/5 on all extremities. MRI showed a solid
nodule on the cerebellar vermis with a cystic component. What is the most likely
diagnosis?
❏ Ependymoma → intraventricular
❏ Brainstem glioma → involve cranial nerves
❏ Medulloblastoma → solid tumor
❏ Cerebellar astrocytoma→ cystic; most common
*Titubation is a type of involuntary tremor that occurs in the head, neck and trunk
area that causes uncontrollable rhythmic shaking.
52. A 4 y/o boy is delayed in speech. Which of the ff features DOES NOT fit the diagnosis
of Autism Spectrum disorder?
❏ Poor attention to details → patients have detailed attention
❏ Stereotyped motor movements
❏ Lack of protoimperative pointing
❏ Deficits in emotional reciprocity
cjhernandez/djhorneja/ccorpuz
53. Sue, a 1 month old infant developed erythematous maculopapular rash on both
cheeks, chin, neck and upper extremities. She was born term by CS & was breastfed for
2 wks then shifted to infant cow milk formula. Father smokes. Mom has active AR. The
family has cats & dogs. If Sue has Atopic Dermatitis, what is the most likely trigger?
❏ Dust mites from beddings → may sensitize to dust mites
❏ Cigarette smoke → irritant
❏ Cat & dog allergens → early exposure may protect against atopy later in life;
but later in life may aggravate atopy
❏ Cow milk formula → up to 40% AD has associated food allergy
*Atopic March: Food allergy → Atopic dermatitis → Asthma → Allergic rhinitis
54. At 2 mo old Sue had coryza, fever, cough, dyspnea & chest retractions. PE: RR= 60s, CR =
150s, alar flaring, wheezes and fine crackles. Chest X-ray: hyperaeration and bilateral streaky
infiltrates. CBC: Hgb = 10 gm/dL, Hct = 28%, RBC = 3.4 M, MCV = 82, MCH = 32, RDW = 12,
WBC = 10,000, segmenters = 25%, lymphocytes = 65%, eosinophils = 7%, monocytes = 3%.
Which of the ff clinical or laboratory data is the most important risk factor for Sue’s risk of
developing asthma?
❏ Eosinophilia
cjhernandez/djhorneja/ccorpuz
❏ Birth by CS
❏ Atopic dermatitis
❏ Maternal history of AR
*CBC shows anemia with eosinophilia
55. Sue’s RBC, Hgb, Hct and red cell indices are consistent with:
❏ Thalassemia
❏ Normal count for age
❏ Anemia due to iron deficiency
❏ Normocytic normochromic anemia due to atopy
56. As Sue grows older, her AD remitted and she began to have recurrent episodes of cough
& wheezing, shortness of breath even without coryza leading to frequent ER visits. What is
the best approach for Sue?
❏ Start Montelukast → alternative or add on
❏ Maintain Sue on inhaled steroid via spacer → patient has frequent
exacerbations and inhaled ICS are the best controller
❏ Nebulize with Salbutamol as needed during episodes
❏ Give Salbutamol MDI, 2 puffs via spacer every 4 to 6 hours daily
57. At 20 year old, Sue was diagnosed to have cavitary PTB and was started on HRZE fixed
dose combination which triggered a severe anaphylactic reaction. What should be the
course of action after treating the severe reaction?
❏ Do intradermal skin test to I, R, Z, E individually → not practical, difficult to do
❏ Do incremental dose challenge using fractionated fixed dose combination of
IRZE → dangerous, may deliver an anaphylactic dose
❏ Desensitize individually IRZE
❏ Premedicate with H1 blocker each time the fixed dose combination of IRZE is
taken
58. An 18 month old male infant presents with recurrent otitis media, sinusitis, 2 separate
episodes of pneumonia with chest x-ray finding of consolidation. The most ideal screening
test for this infant in the work up of immunodeficiency is:
❏ ESR, CRP
❏ C3, C4 level
❏ IgG, IgA, IgM level
❏ Serial CBC & differential count
*Recurrent respiratory tract infection → think of B cell deficiency and complement
deficiency
cjhernandez/djhorneja/ccorpuz
59. Children with recurrent infections caused by intracellular organisms often have
deficiency to which component of the immune system?
❏ T cell → intracellular organisms: TB, viral infections
❏ B cell
❏ Phagocytes
❏ Complements
60. A male infant develops severe pneumonia few weeks after birth associated with eczema
and diarrhea. CBC showed WBC of 7,000/mm3, segmenters of 70%, monocytes of 25% and
lymphocytes of 5%. PE also reveals absence of tonsils and lymph nodes. You should strongly
consider:
❏ Omenn syndrome
❏ SCID → early onset, eczema, diarrhea, absence of lymphoid tissue,
lymphopenia; one of deadliest disease, needs bone marrow transplant
❏ Wiskott Aldrich syndrome
❏ DiGeorge’s syndrome
*infants usually have lymphocytosis and this patient only has 5% lymphocytes;
male has predilection for X-linked immunodeficiencies
61. A 6 month old male has history of eczema from birth, recurrent otitis media and
thrombocytopenia on CBC. On peripheral smear, his platelets are small. The most likely
diagnosis is:
❏ SCID → early onset recurrent infections with opportunistic infections, diarrhea,
eczema, pneumonia, OM, FTT, live vaccine → disease, GVHD; lymphopenia,
absent thymus, tonsils, lymph nodes; immediate immune reconstitution vital
❏ Job syndrome → AD, type of Hyper IgE, “buckley syndrome’ stat-3 mutation,
coarse facial features, eczema, suscept to S. aureus, Candida, Pneumatoceles,
abscesses, eosinophilia
❏ Omenn syndrome → form of SCID with RAG 1 and RAG 2 mutations → failure to
form Ag receptors on T and B cells → inc NK, TH2, EO, leukocytosis,
lymphocytosis, IgE → exfoliative erythroderma
❏ Wiskott-Aldrich syndrome
62. Anne, a 12 mo old infant presents acutely with pallor and passage of darkly colored urine
staining the diaper after 3 days of URTI and fever. CBC shows Hgb = 7 g/dL, Hct=21%, RBC 3 x
1012/L, MCV = 90, MCH = 33, reticulocyte count = 12%, RDW = 16, WBC = 12 x 109/L,
segmenters= 60%, lymphocytes 40%, platelet = 800 x 109/L, Coomb’s test is (-). Which of the
ff features is most suggestive of an ongoing hemolytic process causing anemia?
❏ Low RBC
❏ High MCV
❏ Thrombocytosis
❏ High reticulocyte count
cjhernandez/djhorneja/ccorpuz
*increased RDW and reticulocyte count; WBC normal; ↑platelet → sign of reactive
thrombocytosis
*Dx: G6PD; peripheral smear: Heinz bodies - precipitated Hgb after oxidation
64. A 5 y/o male develops petechiae, ecchymoses & massive epistaxis 2 wks after developing
URTI. Platelet count is 8,000/mm3. This treatment can quickly raise the platelet level most
rapidly.
❏ Pulse therapy with methylprednisolone
❏ Prednisone p.o. → most cost effective
❏ Anti-D+
❏ IVIG → MOA: Binds Fc receptors on macrophages in spleen preventing
uptake of sensitized platelets; causes rapid increase in platelet count
*Below 10, 000/mm3 platelet count is dangerous
o
65. A 2 y male has progressive pallor. PE: + frontal bossing, icterisia, splenomegaly. CBC: Hgb
6 gm/dL, Hct 19%, RBC 3.8 M/_m, MCV 50 fL, WBC = 10,000/mm3, segmenters 45%, lymphos
55%, platelets 200,000. /_L, reticulocyte count is 15%. He most likely has:
❏ IDA
❏ G6PD deficiency
❏ Thalassemia major
❏ Diamond-Blackfan syndrome
*Frontal bossing meaning hyperactive bone marrow trying to compensate
66. A 16 y/o male presents with left supraclavicular firm rubbery nontender mass measuring
5x5 cm. He had been losing weight for the last 6 months. For the past several weeks, he had
night time sweating and & on & off fever. CBC & platelet count were normal. Chest x-ray
reveals an anterior mediastinal mass and What is the most likely diagnosis?
❏ ALL → ruled out bc patient has lymphadenopathy
❏ Hodgkin lymphoma → adolescents and young adults
❏ Non-Hodgkin lymphoma
❏ AML
cjhernandez/djhorneja/ccorpuz
*The diagnosis of acute leukemia requires the identification of more than 20-25% blasts in the
bone marrow, i.e. a bone marrow aspirate is essential to confirm the diagnosis
*A cut-off of 20% blasts is used for the diagnosis of AML with a cut-off of 25% used to
distinguish ALL from lymphoma
67. In patients with Hemophilia with mild factor VIII deficiency & type 1 von Willebrand (vW)
disease, which of the ff drugs can be used to induce release of endogenously produced
factors VIII & vW?
❏ Danazol
❏ Estrogens
❏ Desmopressin
❏ Betamethasone
68. Which of the ff statements is TRUE regarding relative incidence of childhood cancers?
❏ Embryonal tumors & sarcomas > CNS cancers > lymphohematopoietic
cancers
cjhernandez/djhorneja/ccorpuz
❏ CNS cancers > lymphohematopoietic cancers > Embryonal tumors &
sarcomas
❏ Lymphohematopoietic cancers > CNS cancers > Embryonal tumors &
sarcomas
❏ All 3 major types equally distributed
69. Children with Down syndrome are at increased risk for developing this type malignancy:
❏ Brain cancers → Risk Factors: NF-1, Tuberous sclerosis
❏ Retinoblastoma → genetics
❏ Non-Hodgkin’s lymphoma → EBV, Primary immunodeficiencies
❏ Acute myeloblastic leukemia → 400x increased risk in 1st 3 yr of life; ALL
AML
10-20x increased risk
70. Which of the ff malignancies is most likely to present with diabetes insipidus,
galactorrhea & poor growth?
❏ Neuroblastoma → Most common extracranial solid tumor in children; most
common malignancy in infants; arising from adrenals and paraspinal
sympathetic ganglia; accidental diagnosis by palpation of spinal mass
❏ Medulloblastoma → cerebellar tumor 90% of embryonal CNS tumors, on the
midline cerebellar vermis, mass in posterior fossa causing 4th ventricular
obstruction and hydrocephalus
❏ Rhabdomyosarcoma → most common soft tissue sarcoma in children; head &
neck, GUT, extremities, orbit; represents as mass in vagina: sarcoma
botryoides
❏ Langerhans cell histiocytosis → from pituitary involvement; other tumors
causing DI: craniopharyngiomas
71. All the ff are common finding in tumor lysis syndrome EXCEPT?
❏ Hyperphosphatemia
❏ Hyperuricemia
❏ Hypercalcemia → should be hypocalcemia
❏ Hyperkalemia
72. A 3 y/o male presents with anorexia, intermittent low grade fever & right knee and shin
pain of 1 month duration, pallor & hepatosplenomegaly. CBC reveals anemia, leucocytosis
and lymphocytosis. The most likely diagnosis is:
❏ SLE
Au
❏ Acute lymphoblastic leukemia → bone pain is due to marrow infiltration;
Diagnosis would be a bone marrow biopsy
❏ Systemic Juvenile Idiopathic Arthritis
❏ Growing pain
cjhernandez/djhorneja/ccorpuz
73. Ace, an overweight 1 y/o known asthmatic maintained on moderate dose ICS and
montelukast continued to have cough and post-tussive vomiting of previously ingested milk.
Patient still bottle feeds 8 x a day. He now has respiratory distress. Chest X-ray reveals RUL
consolidation. You should strongly consider this empiric treatment on top of the antibiotic &
anti-asthma therapy.
❏ PPI → empiric treatment since patient has GERD
❏ Prokinetic
❏ N-acetylcysteine by inhalation
❏ Extensively hydrolyzed cow milk formula
*At 1 year old patient must be complementarily fed; only 2-3 bottles of milk/day
74. This E. coli infection is the most common cause of acute, prolonged & persistent diarrhea
in children <2 y/o in developing countries characterized by profuse, non-bloody diarrhea
with mucus, vomiting & low grade fever.
❏ ETEC
❏ EPEC → Enteropathogenic E. Coli and Enteroaggressive E.coli cause
persistent non bloody diarrhea
❏ EIEC
❏ STEC
75. Young infants with malrotation usually present in the first year of life with:
❏ Bilious vomiting
❏ Fecaloid vomitus→ lower GIT obstruction
❏ Recurrent abdominal pain
❏ Recurrent non-bilious vomiting
76. The main reason for massive painless bleeding in Meckel’s diverticulum is:
❏ Intussusception → may occur but usually PAINFUL; meckel’s serves as lead
point
❏ Secondary tissue invasive colitis
❏ Presence of hemorrhoidal tissue
❏ Presence of acid secreting mucosa
77. The marker of seroconversion that correlates with improvement in chronically infected
hepatitis B patients:
❏ Anti-HbsAg → after immunization
❏ Anti-HBcAg
❏ Anti-HBe IgG
❏ All of the above
cjhernandez/djhorneja/ccorpuz
78. The most common complication of Hirschsprung’s disease that contributes to morbidity
& mortality:
❏ GERD
❏ Enterocolitis → HAEC: Hirschsprung-associated Enterocolitis
❏ Gastric volvulus
❏ Intussusception
80. Calcinosis is a complication of this connective tissue disease when treatment is delayed:
❏ SLE
❏ SoJIA
❏ Behcet’s disease
❏ Juvenile dermatomyositis
*Scleroderma also had calcinosis
81. Fever lasting at least how many weeks in a child with evanescent rash, arthritis and
hepatomegaly qualifies for the diagnosis of systemic onset juvenile idiopathic arthritis?
❏2
❏4
❏ 6 → minimum duration of arthritis to qualify for JIA
❏8
*Quotidian fever of at least 3 days qualifies for SoJIA
82. Enthesitis & axial arthritis are MOST prominent in which of the ff spondyloarthritides?
❏ Reactive arthritis
❏ Juvenile psoriatic arthritis
❏ Arthritis associated with IBD
❏ Juvenile ankylosing spondylitis → sacroiliitis, inflammatory back pain, limited
ROM of lumbar spine; HLA B27; <16 y/o; left untreated develops
bamboo-spine deformity
83. The highest risk for sudden death among Kawasaki patients who have developed
coronary aneurysm occurs in this stage:
❏ Acute
❏ Subacute
cjhernandez/djhorneja/ccorpuz
❏ Convalescent
❏ Any time
84. A 6 month old infant presents with high fever & irritability for the past 5 days. All of the
ff findings are consistent with Kawasaki EXCEPT:
❏ Markedly erythematous bulbar conjunctivae without purulent discharge
❏ Multiple 0.5 to 1 cm posterior cervical lymph nodes → unilateral ≥ 1.5cm
❏ Multiple erythematous macules
❏ Swollen hands and feet
87. Steroid resistant nephrotic syndrome is most often seen in this condition:
❏ Mesangial proliferation → response rate 50%
❏ Minimal change nephrosis → remission rate 90%
cjhernandez/djhorneja/ccorpuz
❏ Membranous nephropathy
❏ Focal segmental glomerulosclerosis → remission rate 15-20%
88. A child with chronic kidney disease complains of frequent muscle cramps. She also has
failure to thrive. She most likely has renal osteodystrophy caused by:
❏ Hyperkalemia
❏ Hypercalcemia
❏ Metabolic alkalosis
❏ Increased parathyroid hormone secretion
91. A 16 y/o male for sport clearance is thin & tall for his age, with long fingers, arm span
longer than his height, decreased upper body: lower body segment ratio. Most of his family
members are tall & thin. You should consider:
❏ Familial Tall stature → no organic pathology found
❏ Klinefelter syndrome → common sex chromosomal ploidy in males, 47XXY
syndrome, tall stature, gynecomastia, low upper:lower body ratio; learning
disability
❏ Marfan syndrome → autosomal dominant with lens subluxation, hypotonia,
cardiac valvular deformities, aortic root dilation
❏ XYY syndrome → tall stature, problems in motor and language, antisocial
behavior
cjhernandez/djhorneja/ccorpuz
92. A newborn is noted to have poor suck, vomiting, lethargy & convulsion. The first sibling
died in the neonatal period from sepsis. If a genetic metabolic disorder is suspected, what
should be the initial lab test?
❏ ABG
❏ Glucose
❏ Electrolytes
❏ Serum ammonia level
93. Children with severe cases of this condition have lighter complexion, end up with
profound intellectual disability & seizures a nd often smell musty or mousey.
❏ Tyrosinemia
❏ Galactosemia
❏ Phenylketonuria
❏ Urea cycle disorder
94. Defects in breakdown of branched chain amino acids like leucine, valine & isoleucine
lead to:
❏ Tyrosinemia
❏ Galactosemia
❏ Urea cycle defect
❏ Maple syrup urine disease
95. An apparently healthy couple who are first cousins have 2 affected children, 1 boy and 1
girl, with the same disease. The inheritance pattern is most likely:
❏ Autosomal dominant
cjhernandez/djhorneja/ccorpuz
❏ Autosomal recessive → parents are carriers; common in consanguinity
❏ X-linked disease
❏ Y-linked disease
96. A 5 y/o girl diagnosed to have large VSD with pulmonary hypertension frequently had
cyanosis of the lips and nailbeds & easy fatigability. What complication has set in?
❏ Arrhythmias
❏ Cerebral thrombosis
❏ Infective endocarditis
❏ Eisenmenger syndrome
97. A 10 y/o male has high fever of 5 days , then exquisitely tender & swollen right wrist,
with recent history of sore throat. The rest of the PE is unremarkable. Which data below will
qualify child as having Rheumatic fever?
❏ Lymphocytosis
❏ Leucocytosis & neutrophilia
❏ Low C3
❏ Elevated ASO titer & increased CRP
98. A newborn male became cyanotic few hours after birth with no response to 100% O2. S2
is single with holosystolic harsh murmur on the left lower sternal border. CXR shows boot
shaped heart, with decreased pulmonary blood flow. What drug should be infused to
improve oxygenation while waiting for corrective intervention?:
❏ Furosemide
❏ Dopamine
❏ Digoxin
cjhernandez/djhorneja/ccorpuz
❏ PGE1 → to maintain patencypatency of ductus arteriosus
99. A newborn LGA male infant of diabetic mother is tachypneic & cyanotic soon after birth.
There was no response to 100% O2. S2 is single & loud. Chest X-ray shows mild
cardiomegaly, narrow mediastinum, and increased pulmonary blood flow. What is the
surgical treatment of choice?
❏ Blalock-Taussig shunt → Tetralogy of Fallot
❏ Rashkind balloon atrial septostomy → for TGA
❏ Jatene procedure → arterial switches for TGA
❏ Mustard procedure → atrial switch for TGA
*Dx: TGA; CXR: Egg on side appearance
100. PDA in this age group can be medically closed by administration of Paracetamol:
❏ Preterm
❏ Full term
❏ Infants
❏ All of the above
EXTRA QUESTIONS
101. COVID infection in the early stage leads to impaired production of this cytokine in
patients with poor outcome:
❏ IFN-1
❏ TNF-∝
❏ IFN-ɣ
❏ IL-6
102. Dysfunctional activation of the immune response by COVID-19 leads to pyroptosis of
which cell component of leucocytes:
❏ Neutrophils
❏ Lymphocytes
❏ Eosinophils
❏ Monocytes
103. COVID cytokine storm usually begins on which day of the illness?
❏ 5th
❏ 7th
❏ 10th
❏ 14th
104. Which of the following enhances expression of ACE2:
❏ Vit D
❏ Vit C
❏ Zinc
❏ Chloroquine
105. Which of the following inhibits binding of SARS-COV2 to ACE2 receptors:
❏ Vit D
❏ Vit C
❏ Zinc
❏ Chloroquine
cjhernandez/djhorneja/ccorpuz
106. Congenital CMV → as chorioretinitis, microcephaly, hepatosplenomegaly, SGA,
petechiae, purpura, jaundice
107. Children below this age may not have meningeal signs even in the presence of
meningitis:
❏ 6 mos
❏ 12
❏ 18
❏ 24
108. Anticonvulsant should be given to a child when acute febrile seizure lasts beyond how
many minutes?
❏5
❏ 10
❏ 15
❏ 20
109. After a young child has untreated primary TB disease, which of the ff extrapulmonary
TB infections is most likely to occur first?
❏ Scrofula
❏ Meningitis
❏ Bones
❏ Kidneys
* Meningitis → scrofula → bones → kidneys
110. The most common cause of mortality in patients with Coarctation of aorta:
❏ CHF
❏ Acute MI
❏ Endocarditis/Endarteritis
❏ Arrhythmia
111. Patients with ITP whose platelet count is below this is at high risk for serious bleed:
❏ 40, 000
❏ 30,000
❏ 20, 000
❏ 10, 000
112. Peak incidence of ITP in childhood:
❏ <1 y/o
❏ 1-4 y/o
❏ >4-10 y/o
❏ >10-18 y/o
114. If a newborn infant is found to have (-) ROR, hypoglycemia and subsequently develops
cholestasis you should think of:
❏ Tyrosinemia
❏ Galactosemia
❏ PKU
❏ Hypothyroidism
115. Systemic steroids are indicated for all the ff glomerular diseases except:
❏ PSGN → acute
❏ MCNS
❏ HSP Nephritis
cjhernandez/djhorneja/ccorpuz
❏ MPGN
116. James, a 10 yo boy has 2 mo history of deteriorating school grades, odd behavior,
frequent grimacing and emotional lability. If sydenham’s chorea is suspected, which of the ff
statements is TRUE?
❏ ASO Titer is expected to be >1:333
❏ James should be started on Benzathine Penicillin IM every 3 weeks
❏ James should first undergo psych evaluation to rule out depression or psychosis
❏ EEG should be performed to rule out seizure disorder before considering
Sydenham;s chorea
117. In the treatment of T1DM patients with DKA who is on insulin drip, glucose containing
IVF should be started once serum glucose (mg/dL) has decreased to about this level:
❏ 200
❏ 250 → D10 containing solution is added
❏ 300 → D5 containing solution is added
❏ 350
118. What is the drug of choice for children with grave’s disease?:
❏ PTU → toxic to liver in children
❏ Methimazole
❏ Thyroidectomy
❏ RAI
120. Which of the ff sets of blood types increases the risk for ABO incompatibility in the
baby?
Mother Baby Father
A A O B
B O B B
C B O B
D O O O
121. An 8 y/o child came in after 3 days of diarrhea and vomiting. Last urine output was
more than 12 hours ago. Serum creatinine is 4mg/dL. If he is 120 cm tall, his estimated CrCl
(ml/min/.73m2) with a proportionality constant of 0.55 will be:
❏ 11
❏ 16.5 → CrCl = 120 x 0.55)/4
❏ 35.5
❏ 66
123. Premature rupture of membrane beyond how many hours increases the risk for
neonatal infections.
❏8
❏ 12
❏ 18
❏ 24
124. “Tet” spell of ToF is primarily due to:
❏ Complicating myocarditis
❏ Markedly decreased LV outflow
❏ Increased L to R shun through VSD
cjhernandez/djhorneja/ccorpuz
❏ Markedly decreased RV outflow
125. A 6 month old girl presents with small head, developmental delay and intractable
seizures. Cranial MRI showed bilateral clefts along bilateral frontal lobes. The most likely
diagnosis is:
❏ Lissencephaly
❏ Schizencephaly
❏ Porencephaly
❏ Holoprosencephaly
126. Alex, 5 y/o had 5 days persistent high fever, anorexia, vomiting, diarrhea, on and off
abdominal pain. At the ER, Temp = 37.5/axilla, he was flushed, lethargic, with sunken
eyeballs, dry cracks lips and scanty urine. BP = 80/60, CR = 120, RR = 30/min, with weak
pulses. CBC showed Hgb = 16gm/dL, Hct = 48, WBC = 1,500/mm3, platelet 80, 000/mm3. UA:
spec gravity 1.030, WBC: 6-8/HPF, rbc 0-2/HPF. What is the most likely diagnosis?
❏ Acute Viral Illness
❏ Dengue Fever
❏ Dengue with Warning Sign
❏ Severe Dengue
127. A 16 y/o asymptomatic male had trauma to epigastric area for which ultrasound of
abdomen was done which showed marked hepatosplenomegaly. CBC showed Hgb 79gm/L,
Hct 0.19, platelet 220, 000, WBC 505, 000/mm3, segmenters 43%, bands 12%,
metamyelocytes 3%, myelocytes 40%, lympho 1%, EO 1%. PE: liver span 14cm, spleen tip
22cm from LSCM. What is the most likely diagnosis?
❏ Hodgkin’s lymphoma
❏ Non-Hodgkin’s lymphoma
❏ AML
❏ CML
128. Aq 3 y/o picky eater is in severe colicky periumbilical abdominal pain. He has BM 2 to 3x
a week. Rectal exam is (+) for hard stools. Scout film shows mottled lucencies throughout
the colon. After successfully evaluating the hardened stools, what should be given to help
reestablish regular BM?
❏ Senna daily
❏ Lactulose per orem BID
❏ Domperidone per orem TID
❏ Glycerine suppository as often as needed
129. True about ANA positivity:
❏ Has high specificity as screening test for SLE
❏ Can be used to gauge disease activity in lupus nephritis → use Anti-Dsdna
❏ Indicates high risk for development of uveitis in children with JIA
❏ Indicates high risk for developing autoimmune disease in the future
130. Prenatal infection with this organism is associated with development of B cell
autoimmunity leading to development of B cell autoimmunity leading to development of
T1DM postnatally.
❏ Toxoplasma
❏ Syphilis
❏ CMV
❏ Rubella
cjhernandez/djhorneja/ccorpuz
INTERNAL MEDICINE MOCK BOARD EXAM
UST MBR 2020
1. A 26 year-old sought consult since couple is desirous to have a pregnancy. She has regular
cycles, LMP 6 days ago. What phase of the endometrial cycle is she in today?
*Assuming the patient has a 28 Day cycle
❏ Menstrual → First 5 days
❏ Proliferative → starts at Day 6 and ends at ovulation; coincides with
follicular phase of ovarian cycle
❏ Secretory → after Day 14/ovulation
❏ Pre menstrual
2. Which product of conception is found at the ampullary portion of the fallopian tube?
❏ Zygote
❏ Blastocyst
❏ Embryo
❏ Fetus
cjhernandez/djhorneja/ccorpuz
PATHOLOGY MOCK BOARD EXAM
UST MBR 2020
1. The pleura that surrounds the lung tissue with long standing exposure to asbestos may
develop a malignancy composed of:
❏ Mesothelial cells
❏ Endothelial cells
❏ Mesenchymal cells
❏ Hematopoietic cells
2. Women exposed to secondary smoke commonly develop which type of malignancy?
❏ Squamous cells carcinoma
❏ Hamartoma
❏ Adenocarcinoma
❏ Sarcoma
3. Pleural fluid cytology resulting from a patient with severe ARDS will show:
❏ Polymorphonuclear inflammatory cells admixed with alveolar epithelial cells
in fibrin
❏ Mononuclear inflammatory cells admixed with some mesothelial cells in fibrin
❏ Lymphocytes and red blood cells in an exudates
cjhernandez/djhorneja/ccorpuz/cjgopez
❏ Acellular transudate
*Neutrophils adhere to the activated endothelium and migrate to interstitium and
alveoli and cause damage to type 2 pneumocytes
6. A 22 year old medical was rushed to the emergency room because of dyspnea. Pulmonary
PE revealed patient in respiratory distress with marked expiratory wheezes. Which is the
likely clinical impression?
❏ Bronchial asthma
❏ Bacterial pneumonia
❏ ARDS
❏ Pneumothorax
7. A 2 year old boy was admitted to the hospital because of dyspnea. The mother says that
the child had high grade fever and cough since 3 days ago, unable to eat and very irritable.
PPE revealed a child in respiratory distress, highly febrile with chills. Auscultation shows
crepitant rales on both lungs. Which is the likely diagnosis?
❏ Bronchial asthma
cjhernandez/djhorneja/ccorpuz/cjgopez
❏ Pneumonia
❏ Neoplasm
❏ Pneumothorax
8. A 22 year old sexually active male was admitted because of respiratory distress. He has
productive cough and is seropositive for HIV and culture + for MTb. Direct microscopy of the
submitted expectorates infection. Identify the pathogen.
❏ Pneumocystis carinii
❏ Entamoeba histolytica
❏ Escherichia coli
❏ Giardia lamblia
*Salient features: productive cough + HIV
*Opportunistic infections in HIV: Candidiasis, Toxoplasmosis, CMV, Cryptococcosis
9. A 30 year old young executive came down with high grade fever, productive cough and
severe dyspnea warranting admission to the hospital. Chest radiograph show confluent
consolidation in the upper lobe. Differential diagnosis would include which of the following?
❏ Lobar pneumonia → confluent consolidation
❏ Pulmonary tuberculosis
❏ Small cell malignancy consider oat cell carcinoma
*Salient features: high grade fever, prod cough, severe dyspnea, confluent
consolidation in upper lobe
12. Failure of which side of the heart is congested abdominal viscera (early phase) a
prominent feature of?
❏ Lateral
❏ Medial
❏ Left → pulmonary congestion (lung)
❏ Right → manifests as edema, GI tract and liver congestion
13. The central grumous core of atherosclerotic plaque is rich in which of these?
❏ Necrotic debris → necrotic center (foam cells, cholesterol crystals)
cjhernandez/djhorneja/ccorpuz/cjgopez
❏ Fibrous tissue → fibrous cap
❏ Smooth muscle cells → fibrous cap
❏ Lipids → Lipid core
16. Which is the lesion in the heart caused by vascular occlusion of up to 95% and shows
neutrophilic interstitial infiltrates of the ischemic area?
❏ Stable Angina
❏ Unstable Angina
❏ Prinzmetal Angina
❏ Acute Myocardial Infarction → approx 3 days already
17. A one and a half month old girl infant was brought to the ER due to tachypnea and
difficulty with feeding. There was no weight gain noted since birth. On auscultation she was
noted to have a continuous systolic murmur. The child is most likely suffering from which
pathology?
❏ Tetralogy of Fallot
cjhernandez/djhorneja/ccorpuz/cjgopez
❏ Ventricular Septal defect → causes left to right shunt causing loud, harsh,
holosystolic murmur at left lower sternal border
❏ Patent Ductus Arteriosus
❏ Coarctation of the Aorta
23. Anasarca among congestive heart failure patients is noted because of which mechanism?
❏ Decreased colloid oncotic pressure
❏ Increased vascular permeability
❏ Increased hydrostatic pressure
❏ Obstruction of the lymphatic channel
24. The common mechanism between extensive hemorrhage and myocardial infarction in
the evolution of shock is which of these?
❏ Decreased blood volume
❏ Decreased cardiac output → all of the types of shock will lead to this
❏ Internal fluid loss
❏ Vasoconstriction
cjhernandez/djhorneja/ccorpuz/cjgopez
25. Which of the following are characteristic of antemortem clots?
❏ Currant jelly clots → Post mortem clots
❏ Lines of Zahn → alternating light and dark areas
❏ Black tarry features
❏ Mostly red blood cells histologically
26. Virchow’s triad (factors favoring thrombosis) includes which of the following?
❏ Leukocytosis, thrombocytosis, erythrocytosis
❏ Platelets, red blood cells, leucocytes
❏ Stasis, vascular injury, hypercoagulability
❏ Hypertension, hyperlipidemia, hyperglycemia
27. An arterial thromboemboli is most likely to produce a hemorrhagic infarct in which
organ?
❏ Brain
❏ Lungs
❏ Heart
❏ Kidney
28. On a trip to the Caribbean, a 29 y/o man takes a scuba diving course. He goes on a wreck
dive to a depth of 50 meters (170 feet). His diving partner signals to him that it is time to
ascend but he continues to explore the wreck, noticing some time later than he is running
low on air. He panics and ascends to the surface quickly. Within 2 hours he then develops a
headache, tightness in his chest, and difficulty breathing. He experiences generalized pain in
all extremities. Which of the following injuries is the most likely cause for his findings?
❏ DIC
❏ Nitrogen bubble formation → Decompression sickness, form of gas embolism
cjhernandez/djhorneja/ccorpuz/cjgopez
❏ Hypothermia
❏ Free radical cellular injury
29. A 46 year old man has had increased sputum production and a chronic cough for the past
5 months. On physical examination there are expiratory wheezes heard over the right lung
anteriorly. A chest x-ray demonstrates a 6 cm. right perihilar mass. Laboratory studies
include sputum cytology, with the report that Atypical cells are present suggestive of
squamous cell carcinoma. Which of the following risk factors is most likely to be associated
with these findings?
❏ Exposure to silica
❏ Exposure to bird dust
❏ Cigarette smoking
❏ AIDS
*Squamous CC - smokers; AdenoCA - nonsmokers
30. A 31 year old man is working on the ungrounded electrical wiring in his house. He has
not flipped the circuit breaker switch to the “off” position. He accidentally touches the bare
black wire of a 110 volt 20 ampere circuit. Which of the following is most likely to be the
major life-threatening problem associated with this event?
❏ Thermal burns
❏ Cardiac arrhythmia
❏ Vascular thrombosis
❏ Hyperthermia
31. An elderly man is found dead in his bed at home after paramedics are called by his
neighbor, who noticed that the man was out working in his garden as he usually did, but
collapsed. When paramedics arrive, the man is noted to have marked salivation with
lacrimation. His pupils are pinpoint. He is incontinent of urine. On the way to the hospital he
vomits. Which of the following environmental
exposures best explains these findings?
❏ Insect sting
❏ Asbestos
❏ Ozone
❏ Organophosphate
*Miosis
32. Polymerase chain reaction is used to determine:
❏ Seropositivity
❏ Viral load
cjhernandez/djhorneja/ccorpuz/cjgopez
❏ Survival time
❏ Genetic sequence
37. After an ultrasound finding of a thickened endometrium and a large solitary mass, she
undergoes TAHBSO. On histopath examination, the endometrium diffusely reveals vari-sized
glands which are irregularly distributed in cellular stroma. No nuclear atypia i s seen. This
finding is consistent with:
❏ Simple hyperplasia of endometrium
❏ Secretory endometrium
❏ Adenocarcinoma
cjhernandez/djhorneja/ccorpuz/cjgopez
❏ Atrophic endometrium
38. If the large solitary mass is within the myometrium and has a white whorled cut surface,
with no necrosis, it is grossly which of these?
❏ Leiomyoma
❏ Endometrial polyp
❏ Leiomyosarcoma → grossly necrotic, atypia cells (polychromasia, increased
mitotic figures); malignant
❏ Fibroma
39. Which of the following ovarian tumors is most likely to be associated with endometrial
hyperplasia or carcinoma?
❏ Immature teratoma
❏ Dysgeminoma
❏ Granulosa cell tumor → stromal tumors of ovary, sometimes can
elaborate large amounts of estrogen causing hyperplasia leading to
carcinoma
❏ Mucinous cystadenoma
40. If the solitary mass were an ovarian mass that turned out to be a cyst with chocolate
brown g ranular inner surface, the most likely diagnosis is:
❏ Dermoid cyst
❏ Mucinous cystadenoma
❏ Endometrial cyst
❏ Serous cyst
41. The first marker to appear in the serum of patients with acute hepatitis B:
❏ HBsAg
❏ HBeAg
❏ Anti-HBc
❏ Anti-HBs
cjhernandez/djhorneja/ccorpuz/cjgopez
*Antigen: surface→ core (s,e) HbsAg, HbeAg
*Antibody: core→ surface (c,e,s) Anti-Hbc, Anti-Hbe, Anti-Hbs
cjhernandez/djhorneja/ccorpuz/cjgopez
45. Most common benign neoplasm of the
liver:
❏ Bile duct adenoma
❏ Hepatic adenoma
❏ Cavernous hemangioma
❏ Focal nodular neoplasia
46. Which of the following is a form of cellular adaptation?
❏ Hydropic change
❏ Dysplasia
❏ Metaplasia
❏ Cloudy swelling
*Other cellular adaptation: Hyperplasia, Hypertrophy, Atrophy
cjhernandez/djhorneja/ccorpuz/cjgopez
49. The cause of muscle atropy in poliomyelitis is:
❏ Decreased workload
❏ Loss of innervations
❏ Diminished blood supply
❏ Loss of endocrine stimulation
50. The point of no return of the cell leading to cell death is:
❏ When ribosomes detach from RER
❏ During high amplitude swelling of the mitochondria
❏ During pyknosis
❏ During intracellular release of lysosomal enzymes → 2 Phenomenon of
Point of no return: inability to reverse mitochondrial dysfunction, profound
damage to membranes
cjhernandez/djhorneja/ccorpuz/cjgopez
❏ Mallory body → cytoplasmic hyaline inclusion of hepatocytes seen in
alcoholic hepatitis and may be seen in other conditions of liver such as
cholestasis and NASH
❏ Councilman-like body → represent a dying apoptotic hepatocyte
54. Autopsy of a stillborn shows solid, liver-like lungs with pink fibrin-like materials along the
alveolar wall. The most probable diagnosis is:
❏ Viral pneumonia
❏ TORCH infection
❏ Bacterial pneumonia
❏ Hyaline membrane disease of the newborn → respiratory distress syndrome of
newborn; hyaline membranes are composed of sloughed off epithelium plus
necrotic debris of dying alveoli plus edema → consolidate as pink fibrin like
materials; also seen in Adult Respiratory Distress Syndrome
56. Which of the following is included in the Philippine newborn screening program?
❏ Cystic fibrosis → included in Expanded
❏ Congenital hypoparathyroidism → Congenital Hypothyroidism
❏ Sickle cell anemia
❏ Galactosemia
cjhernandez/djhorneja/ccorpuz/cjgopez
57. Which of the following is the EM finding in acute post streptococcal glomerulonephritis?
❏ Mesangial deposits
❏ Subepithelial humps
❏ Intramembranous deposits
❏ Subendothelial deposits
cjhernandez/djhorneja/ccorpuz/cjgopez
❏ Epithelial crescents → RPGN has expansion and proliferation of epithelial
cells lining Bowman’s Capsule, also called crescentic glomerulonephritis
❏ Wire loop lesions
61. Clinically mesangial lesions in the glomeruli are usually manifested as:
❏ Proteinuria
❏ Hematuria → 1st manifestation of any glomerulonephritis
❏ Nephritic syndrome
❏ Nephrotic syndrome
cjhernandez/djhorneja/ccorpuz/cjgopez
❏ Granular cast in urine
❏ Waxy cast in urine → broad waxy cast in CKD
❏ RBC cast in urine
64. Which of the following glomerulonephritis may manifest as both nephritic and nephritic
syndrome?
❏ Membranous nephropathy → manifest as nephrotic syndrome
❏ Membranoproliferative glomerulonephritis
❏ Focal proliferative glomerulonephritis → another term for Focal segmental
glomerulosclerosis (FSGS) causing nephrotic syndrome; not steroid responsive
❏ Minimal change disease → nephrotic syndrome; responsive to steroids
66. Neoplasm is a distinct dreaded disease the most important aspect of which compared to
pseudo-tumors is its characteristic?
❏ Spread pattern
❏ Association with necrosis → pseudotumors may have ischemic type of
necrosis
❏ Encapsulation
cjhernandez/djhorneja/ccorpuz/cjgopez
❏ Uncoordinated growth → mostly due to accumulation of mutations
67. Which of the following reactive changes is often confused histopathologically with
squamous cell carcinoma?
❏ Squamous metaplasia
❏ Hypertrophy
❏ Hyperplasia
❏ Atrophy
69. Which of the following is the most reliable local sign of cancer?
❏ Atypia of cells
❏ Presence of necrosis
❏ Presence of mitosis
❏ Destruction of basement membrane → differentiates invasive from in-situ
neoplasia
70. Which of the following growth pattern of cancer is most ominous biologically speaking?
❏ Polypoid CA
❏ Ulcerating CA
❏ Fungating CA
❏ Infiltrating CA → very difficult to excise; no gross delineation that’s why it
is more ominous
71. Which is the most common and spreads most commonly thru the lymphatics?
❏ Carcinoma
❏ Liposarcoma → hematogenous spread, rare lymphatic spread
❏ Teratoma → rare lymphatic spread
❏ Blastoma → rare lymphatic spread
72. Which tumor shows elements derived from the different germ cell layers?
❏ Carcinoma
❏ Liposarcoma
❏ Teratoma → 3 germ cell layers: endoderm, mesoderm, ectoderm
❏ Blastoma
cjhernandez/djhorneja/ccorpuz/cjgopez
73. Which tumor ominously spreads most frequently thru the hematogenous route?
❏ Carcinoma → lymphatics
❏ Liposarcoma → not always but most frequently
❏ Teratoma
❏ Blastoma
75. Among the prognostic parameters in cancer the most significant and most reliable is
which of these?
❏ Growth pattern
❏ Tumor grade
❏ Doubling time
❏ Tumor stage
76. The most important and most common infectious granulomatous inflammation of the
skin in the Philippines is:
❏ Hansen’s disease → Leprosy
❏ TB → not as commonly seen in skin
❏ Tophus
❏ Syphilis
77. Which of the following chronic dermatitides is hereditary, clinically exhibiting positive
Auspitz sign?
❏ Lichen simplex chronicus
❏ Psoriasis vulgaris
❏ Lichen planus
❏ Acne vulgaris
*Auspitz Sign → multiple minute bleeding points when scale is lifted from plaque.
cjhernandez/djhorneja/ccorpuz/cjgopez
78. This is the most common unwanted complication of chronic skin disorder:
❏ Fibrosis → disfiguring
❏ Abscess formation
❏ Erythema
❏ Edema
79. A 45 y/o female patient complaints of 3cm breast mass with history of previous trauma 2
months earlier. Which is the best diagnostic procedure?
❏ Excision biopsy → to r/o mimicker of breast CA (fat necrosis)
❏ Incisional biopsy
❏ Core needle biopsy
❏ FNAB
80. Most common tumor in this patient’s age bracket (30-40) is which of the following?
❏ Fibrocystic change
❏ Fibroadenoma → most common benign tumor
❏ Ductal papilloma
❏ Benign Phyllodes Tumor
81. Which of these spindle cell tumors of the breast is potentially malignant?
❏ Fibroma
❏ Lipoma
❏ Rhabdomyoma
❏ Phyllodes tumor
82. The hardness of malignant breast cancer is most commonly due to which of the
following?
❏ Calcification
❏ Osseous metaplasia
❏ Fibrosis → desmoplasia, stromal reaction to any invasive tumor causing tissue
to be fibrotic; reason why Breast Ca described as scirrhous or stony-hard
❏ Hyaline change
83. This benign breast lesion often associated with history of trauma is clinically hard and
irregular is often confused with breast cancer:
❏ Fibroadenoma
❏ Ductal papilloma
❏ Duct ectasia
❏ Fat necrosis
cjhernandez/djhorneja/ccorpuz/cjgopez
84. Which of the following breast malignancies is most frequently associated with central
necrosis?
❏ Ductal-in-situ → characteristic central necrosis or comedo necrosis
❏ Invasive ductal CA
❏ Lobular CA
❏ Mucinous CA
85. Mirror-image biopsy of the contralateral breast in a patient with breast cancer is often
undertaken in which of the above type of cancer?
❏ Ductal-in-situ → unilateral
❏ Invasive ductal CA
❏ Lobular CA → bilateral breast involvement
❏ Mucinous CA
86. In staging of cancer (the most important parameter) under the TNM system, the T
represents which of the following?
❏ Time
❏ Doubling time
❏ Translocation index
❏ Tumor size → may also refer to depth of invasion such as in colon
87. The determination of Estrogen Receptor Assay (ERA). Progesterone Receptor Assay
(PRA), and Her2neu is important for breast cancer patients for which of the following
objectives?
❏ Diagnosis of the breast cancer
❏ Prognosis of the tumor in terms of 5 years survival
❏ Prognosis of the tumor in terms of 10 years survival
❏ Assists in the decision to include chemotherapy and choice of drug
**ER and PR are hormone receptors and can be given hormonal therapy
*Her 2 neu positive → monoclonal ab trastuzumab or Herceptin
* Triple Negative → can’t use hormonal treatment nor monoclonal antibody
88. Most common complication of Gastric ulcer:
❏ Perforation
❏ Obstruction
❏ Bleeding → 15-20% of patients
❏ Intractable pain
cjhernandez/djhorneja/ccorpuz/cjgopez
89. Which polyp has the greatest malignant potential?
❏ Tubular adenoma → premalignant
❏ Peutz-jeghers polyp
❏ Hyperplastic polyp → not premalignant
❏ Villous adenoma → sessile, harder to remove
90. Most common site of squamous cell carcinoma of the oral cavity:
❏ Ventral surface of the tongue
❏ Hard palate
❏ Buccal cavity
❏ Pharynx
91. Soft tissue tumor with spindle cell in 45 degree angle insertion is likely which of these
tumors?
❏ Leiomyosarcoma → 90 degree angle insertion
❏ Malignant fibrous histiocytoma
❏ Fibrosarcoma → Herringbone formation
❏ Synovial sarcoma
92. Which part of the musculoskeletal system is the most common site of tuberculous
infection?
❏ Vertebral body
❏ Vertebral spine
❏ Joints
❏ Long bones
3. A patient is found unconscious and rushed to the ER. The duty physician detects severe
9
hypotension and notes extensive urticaria and multiple insect bites. Emergency measures
included administration of IV fluids and epinephrine. The c ondition is mainly due to systemic
release of:
❏ Serotonin
cjhernandez/djhorneja/ccorpuz/cjgopez
❏ Cytokine
❏ Leukotriene
❏ Histamine → released in anaphylactic shock
94. In the development of tolerance, the deletion (death or inactivation) of self-reactive B
cell clones occurs in the:
❏ Thymus → T cell clones
❏ Bone marrow
❏ Lymph nodes
❏ Spleen
95. A 15 y/0 boy who was a foot taller than his professional basketball center father may be
suffering from:
❏ Gigantism → epiphyseal plates are still open
❏ Acromegaly → increased growth hormone when epiphyseal plates are closed,
seen in hands and feet
❏ Klinefelter’s syndrome → XXY Male
❏ Marfan syndrome → connective tissue disorder, Autosomal dominant,
mutation in FBN1 gene
96. A single 1.0 cm thyroid nodule in a 35 y/o female was cold by radionucleotide scan. The
lobectomy specimen showed a well-encapsulated mass which microscopically showed
follicles confined to the interior of the fibrous capsule with the rest of the thyroid having a
normal histology. What would this growth most probably be?
❏ Follicular adenoma → confined to the interior of capsule
❏ Adenomatous nodule
❏ Follicular carcinoma → with lymphovascular or capsular invasion; no
morphological difference between an adenoma
❏ Medullary carcinoma
98. Which is the most common tumor that metastasizes to the brain?
❏ Lung
❏ Breast
❏ Kidney
❏ Gastrointestinal tract
*Lung→ Breast→ Kidney→ Thyroid→ Lymphoma (most common to
least common)
cjhernandez/djhorneja/ccorpuz/cjgopez
99. The process of coating a particle to target it for ingestion is called:
❏ Phagocytosis
❏ Opsonization
❏ Engulfment
❏ Adhesion
100. This is classified as a quiescent cell with a low level of replication but can undergo rapid
division in response to stimuli:
❏ Squamous epithelium, skin→ rapidly dividing cells, can replicate to high levels
in response to stimuli
❏ Liver
❏ Neurons → permanent cell, no regeneration
❏ Hematopoietic tissues → rapidly dividing cells
cjhernandez/djhorneja/ccorpuz/cjgopez
3. A 20 year-old is pregnant at 5-6 weeks AOG. Which of the following is the main source of
hormonal support to her pregnancy?
❏ Dominant follicle
❏ Graafian follicle
❏ Corpus luteum → up to 9 weeks AOG
❏ Placenta
4. Which of the following corresponds to the greatest transverse diameter of the fetal head?
❏ Occipitomental
❏ Occipitofrontal → greatest circumference
❏ Biparietal
❏ Subocipitobregmatic → smallest circumference
cjhernandez/djhorneja/ccorpuz
❏ Asthmatic, 12 weeks
❏ Chronic Hypertensive 37 weeks → due to stress brought about by
hypertension, pulmonary maturation stimulated and patients able to
survive
❏ Diabetic, 34 weeks
7. Which of the following position improve the venous return through the inferior vena cava
during pregnancy?
❏ Supine
❏ Right lateral decubitus
❏ Left lateral decubitus
❏ Prone
8. A 26 year-old sought consult due to missed period of 5-6 weeks. If she has a positive
pregnancy test, what explains her amenorrhea?
❏ Arrested folliculogenesis
❏ High estrogen and progesterone levels
❏ Presence of corpus luteum
❏ High HCG levels
9. A 30 year-old G1 is on her 7-8 weeks AOG. What positive evidence of pregnancy is present
at this time?
❏ FHT on hand held Doppler → appreciated at 10-12 weeks
❏ Embryo on TVS → seen at 6-7 weeks AOG; CRL can be measured
❏ Quickening → Primigravid:18-20 weeks; Multigravid: 16-18 weeks
❏ Fetal movement noted by OB
10. A 20 year-old primigravid complained of nausea, vomiting and easy fatigability. She has
irregular menstrual cycles. If pregnancy is entertained to explain her symptoms, how should
gestational age be determined?
❏ Base on LMP → not accurate in patients with irregular menstrual
cycle
❏ First trimester ultrasound → most accurate in determining AOG
❏ Uterine size by IE
❏ Fundic height measurement
11. Where is the uterus palpated in a 22 year-old G1 on her 8th week AOG?
❏ Xiphoid
❏ Umbilical → 20 weeks AOG
❏ Symphysis pubis → 12 weeks AOG
❏ Pelvic cavity
cjhernandez/djhorneja/ccorpuz
*16 weeks: midway between symphysis pubis and umbilicus
*20-34 weeks correlates FH w/ AOG
12. A 30 year-old, 20 weeks AOG consults for prenatal care. OB history shows 2016-
Spontaneous abortion at 8 weeks; 2017- CS for twins at 35-36 weeks, unremarkable; 2019-
Repeat CS 39 weeks, un remarkable. What is the complete gravidity, parity and OB score?
❏ G3P3 ( 1103 )
❏ G4P3 ( 1103 )
❏ G4P2 ( 1113 )
❏ G4P2 ( 1213 )
13. A 38 year-old G2P1(1001) on her 18-19 weeks AOG complained of vulvar pruritus with
fishy vaginal discharge. Speculum: cervix- violaceous, smooth with yellowish, frothy, fishy
discharge. IE: uterus compatible with 41/2 months. What is the etiology of the vaginal
discharge?
❏ Candidiasis → whitish curd-like discharge; pruritic but non-foul smelling
❏ Trichomonas vaginalis
❏ Bacterial vaginosis → grayish, foul-smelling discharge (fishy)
❏ Chlamydia trachomatis → hyperemic cervix
14. A 25 year-old 14-15 weeks AOG consults after her visit 4 weeks ago. She is
asymptomatic. Which of the following is done during this visit?
❏ Check FHT by doppler → detectable at 10-12 weeks AOG; Steth - 16 weeks
AOG
❏ Measure fundic height → start at 20 weeks AOG
❏ Get abdominal circumference → not done
❏ Perform speculum exam → done but priority is to check FHT first
*check viability of pregnancy at first visit
15. A 34 year-old G3P2 (0111) on her 20-21 weeks with Chronic Hypertension came for
prenatal check-up. OB History: G1- delivered by NSD, 32 weeks died due to cardiac anomaly
PPE: BP- 150/90; CR- 82/min, RR-20/min, FHT-140/min. Which of the following should be
requested to monitor fetal growth?
❏ Fetal biometry → every 3-4 weeks; at earliest can be done at 1-2 weeks
❏ Uterine artery Doppler → screening for preeclampsia or IUGR
❏ Umbilical artery Doppler → test the placental function
❏ Non stress test → assess reaction of fetal heart rate to fetal movement
16. At 34-35 weeks AOG, what test can assess the reaction of fetal heart rate to fetal
movement?
❏ Non stress test
cjhernandez/djhorneja/ccorpuz
❏ Contraction stress test → evaluate fetal heart rate in relation to uterine
contractions (stress)
❏ Biophysical profile → antenatal surveillance
❏ Uterine artery Doppler velocimetry → screening for preeclampasia
17. A 40 year-old G3P0 (1011) on her 16 weeks AOG, with Type 2 DM on Metformin sought
consult for prenatal diagnosis. What biometric parameter can best CONFIRM AOG in this
patient?
❏ Yolk sac → 5-6 weeks AOG
❏ Crown rump length → up to 12 weeks AOG
❏ Biparietal diameter
❏ Abdominal circumference → least biometric parameter used for aging
18. Which will confirm best that the membranes have ruptured?
❏ Pooling of fluid during speculum exam
❏ pH of vaginal secretion is 7.5
❏ (+) ferning test
❏ Bluish green color on Nitrazine test
19. A 26 year-old G1 admitted in labor with ruptured membranes and IE of 4 cms 80%
effaced, vertex station -3. What is position should the patient assume?
❏ Supine
❏ Sitting
❏ Walking
❏ Lateral recumbent
20. A 33 year-old G3P2 (2002) 40 weeks was admitted due to labor pains. Labor admission
test was non-reassuring. What is the management?
❏ Intermittent auscultation
❏ Continuous electronic FHR monitoring
❏ Tocolysis
❏ Amnioinfusion
*if delivery is given in the choices, then that would be the best answer
considering the AOG of the fetus
21. A 30 year G2P1(1001) in labor had IE findings of 7 cms, 80% effaced, LOT, station +1,
ruptured membranes. What phase of labor is she in?
❏ Latent → 0-4 cm
❏ Acceleration
❏ Maximum slope → 4-8 cm
cjhernandez/djhorneja/ccorpuz
❏ Deceleration → 8-9cm
22. A 32 year-old G1 in labor at 8 cms ruptured membranes ROA, station 0 had the following
EFM results Baseline FHR - 140-145/min, moderate variability with accelerations and
variable decelerations. What is the clinical significance?
❏ Cord compression
❏ Head compression → early deceleration
❏ Uteroplacental insufficiency → late deceleration
❏ Cord prolapse
23. What cardinal movement has occurred when the lowest portion of the presenting part is
at the level of the ischial spines?
❏ Engagement
❏ Descent
❏ Flexion
❏ Internal rotation
24. When is the best time to do intermittent auscultation of FHR in relation to a uterine
contraction?
❏ Before
❏ Onset
❏ Peak
❏ After
25. A 28 year-old G1 in labor had IE findings of fully dilated, OA station +3. When should she
be instructed to bear down?
❏ Before the contraction
❏ At the height of contraction
❏ Just before the end of the contraction
❏ Anytime she has the urge to push
cjhernandez/djhorneja/ccorpuz
26. Ritgen’s maneuver will facilitate which cardinal movement of labor?
❏ Internal rotation
❏ External rotation
❏ Extension
❏ Expulsion
28. A 23 year-old G1P1(1001) just delivered by NSD 30 minutes ago with no complications.
Where is the uppermost of the fundus be palpated?
❏ Midway between umbilicus and xiphoid
❏ Umbilicus
❏ Midway between symphysis pubis and umbilicus
❏ Pelvic cavity
*4-6 weeks - involution, postpartum size
31. What is the best management for a G3P0(0110) 20 weeks, no uterine contractions, and a
cervical length of 1.8 am on transvaginal ultrasound?
❏ 17-hydroxyprogesterone acetate
❏ Calcium channel blockers → if with uterine contractions
cjhernandez/djhorneja/ccorpuz
❏ Cervical cerclage → cervical length of <2.5 cm; other indication is no
uterine contractions
❏ Bed rest
32. A 35 year-old G2P1(1001) with LMP 8 weeks ago, came in due to vaginal spotting. FH: 20
cm, no fetal heart tones appreciated. IE: cervix soft, long, closed with minimal bleeding;
uterus at the level of the umbilicus, (+) left adnexal mass 8x8 cm. What is the most likely
diagnosis?
❏ Threatened abortion
❏ Ectopic pregnancy → missed menses, hypogastric pain and vaginal bleeding
❏ Molar pregnancy → uterus larger than AOG; incomplete mole: (+) fetal heart
tones, (+) placenta; complete mole: no fetal heart tones
❏ Normal intrauterine pregnancy
33. In abruptio placenta, what will dictate the manner of delivery favoring abdominal
delivery of a dead fetus?
❏ Maternal hemodynamic instability → avoid bearing down
❏ Breech presentation
❏ Unfavorable cervix
❏ Maternal Hypertension
35. What is the first line agent given to a multigravida patient with uterine atony?
❏ Oxytocin
❏ Methylergonovine
❏ Prostaglandin E2
❏ Prostaglandin F2
36. A multigravida underwent forceps delivery secondary to maternal cardiac disease. While
undergoing episiorrhaphy, there was sudden gush of blood per vagina. On physical exam the
BP was 80/50, PR 110/min, weak pulse, uterus contracted. What will be the next step in the
management?
❏ Do bimanual massage → uterus is contracted
❏ Do compression sutures
❏ Inspect for vaginal lacerations → lacerations due to forceps delivery
cjhernandez/djhorneja/ccorpuz
❏ Do immediate Hysterectomy
*stabilize patient as well with fluids
37. What should be entertained if a multigravid diabetic patient develops fever 6 days
postpartum?
❏ Endometritis → 2-3 days
❏ Atelectasis → within 24 hours
❏ Urinary tract infection → 1-2 days
❏ Septic thrombophlebitis
38. Which of the following is NOT used as a mechanical dilator for cervical ripening?
❏ Laminaria → Hygropic dilator
❏ Transcervical catheter → catheter inserted in the cervix
❏ Hegar’s dilators → metal dilator, used in D&C
❏ Membrane stripping
39. A patient who underwent induction of labor developed fetal bradycardia which
coincided with tachysystole on EFM tracing. What will be the immediate management?
❏ Emergency CS
❏ Sedate the Patient
❏ Discontinue oxytocin drip → BEST answer
❏ Give O2 per nasal cannula and observe
*Tachysystole > 5 contractions in a 10 min strip
cjhernandez/djhorneja/ccorpuz
40. Which of the following is a requisite for an external cephalic version?
❏ Anteriorly implanted placenta → Posterior placenta because may cause
abruptio placenta
❏ Epidural anesthesia
❏ Performed at 32 weeks → must be 36 weeks onwards
❏ Adequate amniotic fluid
*Convert breech or transverse lie to cephalic presentation → external cephalic
version
43. Which among the following patients has the highest risk for preterm delivery?
❏ G1P0 pregnancy 12 weeks with subchorionic hemorrhage
❏ G3P2(0110) Pregnancy 20 weeks gestation → 1 preterm, 1 abortion
❏ G2P1(1001) pregnancy 15 weeks whose last delivery was 12 months ago →
if less than 12 months, at risk for preterm delivery
❏ G1P0 pregnancy 28 weeks with bacterial vaginosis → infection is risk for
preterm delivery but can be treated
44. Which among the following BPS parameters is more commonly linked to IUGR?
❏ AFI < 2cm single deep pocket → Amniotic fluid is chronic indicator of hypoxia
cjhernandez/djhorneja/ccorpuz
❏ Absent breathing
❏ Absent fetal movement
❏ Non-reactive NST
*The rest of the choices are acute indicators of hypoxia
45. Once IUGR is established, which among the following is an indication to terminate the
pregnancy?
❏ Decrease resistance in the umbilical artery flow → favorable
❏ Increased resistance in the uterine artery
❏ Reversed end diastolic flow in the umbilical artery → 70% of placenta is
not functioning anymore
❏ Increased resistance in the middle cerebral artery
47. A 17 year-old G1P0 35 weeks gestation came in due to blurring of vision. BP 170/110. FH:
30 cm, FHT 145 bpm. IE was not done. Family history: (+) hypertension parents. What will be
the next management?
❏ Give corticosteroids for fetal lung maturity → the patient is already 35
weeks
❏ Give MgSo4 for neuroprotection → should be less than 32 weeks
❏ Perform a BPS for antenatal surveillance → not an option anymore
because we don’t plan to prolong pregnancy
❏ Do immediate CS → patient already at 35 weeks presenting with
preeclampsia with severe features; steroids not indicated
49. Which cardiac medication is contraindicated during pregnancy because it causes severe
growth restriction?
❏ Ace Inhibitors
❏ Amiodarone
❏ Warfarin
cjhernandez/djhorneja/ccorpuz
❏ Heparin
50. Which laboratory exam result is associated with an increased risk of fetal malformation
in a woman with pregestational diabetes mellitus?
❏ RBS > 200 mg/dl
❏ FBS> 126 mg/dl
❏ Hba1c > 10%
❏ 2nd hr 75gOGTT > 200
51. What will be the management of a G3P1(1011) 12 weeks gestation who came in with the
following lab results: low TSH, normal fT4?
❏ PTU → given for hyperthyroidism
❏ Thyroxine → for hypothyroid patients
❏ Radioactive iodine → contraindicated in pregnancy
❏ Repeat labs after 6 weeks
*patient has subclinical hyperthyroidism
52. The presence of which antibodies may result in fetal congenital heart block in a pregnant
patient with SLE?
❏ (+) ANA → screening but not specific
❏ anti-ds DNA → relatively specific for SLE
❏ anti Sm → specific for SLE
❏ anti-Ro (SS-A) → along with anti-La (SS-B)
cjhernandez/djhorneja/ccorpuz
❏ G3P2(2002) pregnancy 15 weeks, the current pregnancy with anencephaly
❏ G2P0 (0010) pregnancy 8 weeks gestation, Pregestational DM
54. A G1P0 24 weeks gestation came in due to fever. She also complained of chills and left
flank pain. What is the most likely diagnosis?
❏ Asymptomatic bacteriuria
❏ Acute pyelonephritis
❏ Cystitis → hypogastric pain, frequency
❏ Nephrolithiasis
55. What is the ideal anesthesia for a G1P0 39-40 weeks, in labor with bronchial asthma?
❏ General endotracheal anesthesia
❏ General IV anesthesia
❏ Epidural anesthesia
❏ Spinal anesthesia
56. Anemia in a patient with Beta thalassemia requires transfusion to maintain the
hemoglobin at what level?
❏ 9 g/L
❏ 10g/L
❏ 11g/l
❏ 12g/L
57. During pregnancy, exposure to cat feces may cause what type of infection?
❏ Toxoplasmosis
❏ Syphilis
❏ HPV
❏ HSV
58. What is the most common congenital defect which may occur if a pregnant patient
contracted rubella at 10 weeks gestation?
❏ Sensorineural deafness
cjhernandez/djhorneja/ccorpuz
❏ Ventricular septal defect
❏ Hydrocephalus
❏ Hydranencephaly
59. What medical intervention is given to a pregnant woman with active hepatitis B infection
to decrease vertical transmission to the neonate?
❏ Zidovudine → given to AIDS patients
❏ Lamivudine
❏ Ritonavir → given to AIDS patient
❏ Remdesivir →given to COVID patients
60. What type of culture is used for the screening of group B streptococcus?
❏ Cervical
❏ Anal
❏ Vaginal
❏ Anal and vaginal
61. A multiparous woman was rushed to the ER due to profuse vaginal bleeding after
delivering vaginally at home. The relatives claimed that the attendant at birth had a difficult
time delivering the placenta. What is the likely impression?
❏ Uterine rupture
❏ Uterine atony
❏ Uterine inversion →due to forceful delivery of placenta
❏ Birth canal lacerations
62. A G3P2(2002) pregnancy 11-12 weeks by LMP underwent exploratory laparotomy for a
ruptured ectopic pregnancy. What is the most likely site of rupture?
❏ Isthmus → 12%, narrowest portion
❏ Ampullary → early rupture at 5-6 weeks AOG; common area 70% of ectopic
pregnancies
❏ Interstitial → late rupture bc it is muscular so there is more room for expansion
❏ Fimbrial
cjhernandez/djhorneja/ccorpuz
63. Medical management of an ectopic gestation may be carried out if the following criteria
are met EXCEPT:
❏ Size of the ectopic is < 3.5 cm
❏ Asymptomatic
❏ Initial b-HCG level < 1,000mIU/ml
❏ (+) cardiac activity
65. In a fetus with face presentation, what position MAY undergo vaginal delivery?
❏ Mentum posterior → CS
❏ Mentum anterior
❏ Left mentum
❏ Right mentum
66. A 6-year old girl was brought for consultation because of vaginal discharge. What is the
most common etiology of her condition?
❏ Sexual abuse
cjhernandez/djhorneja/ccorpuz
❏ Malignant tumor
❏ Non-specific vulvovaginitis → more common than foreign body; mgt would
be perineal hygiene
❏ Foreign body
67. For a cooperative 15-year old girl, what is the proper position during gynecologic
examination?
❏ Knee-chest
68. In an asymptomatic 30-year old woman, what is the most important reason for
performing a Pap smear?
❏ Diagnosis of infections
❏ Screening for pre-cancer
❏ Hormonal evaluation
❏ Diagnosis of cancer
69. For a woman suspected to have reproductive tract cancer, which examination technique
will allow evaluation of the parametria?
❏ Abdominal
❏ Speculum
❏ Vaginal
❏ Rectal
70. A 68-year old G8 P8 (8008) was found to have prolapse of the uterus. Which pelvic
ligament is the most important in maintaining proper position of the uterus?
❏ Round
❏ Broad
❏ Cardinal→ strongest ligament; Mackenrodt’s Ligament; transverse cervical
ligament
❏ Infundibulopelvic
71. In a woman who comes for routine gynecologic evaluation, what is the first part of pelvic
examination?
❏ Bimanual internal exam
❏ Rectovaginal exam
❏ Inspection of external genitalia
cjhernandez/djhorneja/ccorpuz
❏ Vaginal speculum exam
72.In a 60-year old multigravid woman found to have a cauliflower-like friable bleeding
cervical tumor, what is the most important cause of her condition?
❏ Familial predisposition
❏ Human papillomavirus
❏ Oral contraceptive pills
❏ Early age at coitarche
73. A 70-year old woman presented with abdominal enlargement probably of ovarian origin.
Which component of the ovary accounts for the majority of benign and malignant tumors?
❏ Epithelium
❏ Germ cell
❏ Sex cord/stroma
❏ Embryological remnants
cjhernandez/djhorneja/ccorpuz
❏ 65 years old
❏ Nulliparity
❏ Estrogen therapy
77. A 32-year old G1P1(1001) consulted because of intermenstrual spotting. What diagnostic
procedure will best diagnose and treat the condition?
❏ Transvaginal ultrasound
❏ Saline infused sonography
❏ Endometrial biopsy
❏ Hysteroscopy → both diagnostic and therapeutic
78. What is the recommended management for complex atypical hyperplasia in a 30-year
old single nulligravid?
❏ Oral contraceptive pills
❏ Progestins
❏ Endometrial ablation
❏ Clomiphene citrate
79. What is the expected microscopic finding in vaginal discharge with bacterial vaginosis?
❏ Superficial cells heavily covered by coccobacilli with loss of distinct cell
margins → “clue cells”
❏ Gram positive & gram negative rods
❏ Many epithelial cells with few WBC & flagellated organisms → Trichomonas
vaginalis
❏ Pseudohyphae & mycelia → fungal infection
80. In which condition should the patient’s sexual partner be treated simultaneously?
❏ Bacterial vaginosis
❏ Trichomoniasis → sexually transmitted infection
❏ Candidiasis
cjhernandez/djhorneja/ccorpuz
❏ Atrophic vaginitis
86. A 28 year old G3P1 (1021) came for a Pap smear which showed atypical cells. What is the
next appropriate step in the management of this case?
❏ Cervical biopsy
cjhernandez/djhorneja/ccorpuz
❏ Colposcopy → to visualize transformation zone
❏ Administer HPV vaccine
❏ Conization of the cervix
87. What is the advantage of LEEP over cone biopsy of the cervix?
❏ Bigger specimen
❏ Shallower defect on the cervix
❏ Adequate specimen obtained
❏ Less blood loss
90. Which of the following factors increases the patient’s risk for endometrial
❏ Age at menopause
❏ Smoking
❏ History of endometriosis
❏ Use of tamoxifen→ in breast Ca patients, endometrial
thickness must be monitored
91. A 60 year old, single, nulligravid presented with pleural effusion, ascites, and a solid,
movable, 8 cm. diameter right ovarian mass. What benign ovarian condition presents as
such?
❏ Mature cystic teratoma
❏ Brenner tumor
❏ Fibroma → Meigs syndrome: fibroma, pleural effusion, ascites
cjhernandez/djhorneja/ccorpuz
❏ Mucinous cystadenoma
92. In cases of ovarian tumors, how can intraoperative frozen section help?
❏ Decide extent of surgery
❏ Correlate with tumor markers
❏ Know exact histopathology
❏ For prognostication
93. What type of tumor is most likely the cause of postmenopausal bleeding i n a woman
with a cystic adnexal mass?
❏ Granulosa cell tumor
❏ Gonadoblastoma
❏ Struma ovarii
❏ Thecoma
95.What is the pattern of the escutcheon of a BMI- 20, regularly menstruating 21-year old
with fully developed breasts?
❏ Diamond → male
❏ Inverted triangle
❏ Round
❏ Square
96. Specimens taken for Pap smear from the endocervix and ectocervix will be the
appropriate sites for which purpose?
❏ Cancer screening
❏ Diagnosis of cancer
❏ Hormonal evaluation
❏ Detection of infections
97. CytoHormonal Maturation Index (CHMI) of 0/90/10 is expected in which condition?
❏ Pre-adolescent → 80/20/0
❏ Pre-ovulatory → 0/40/60
❏ Pregnancy
❏ Post-menopause → 100/0/0
*Superficial cells: Estrogen; Intermediate: Progesterone; Parabasal: Androgen
cjhernandez/djhorneja/ccorpuz
98. Which of the following is MOST USEFUL in the management of a patient who presents
with primary amenorrhea with hypogonadotropic hypogonadism?
❏ Karyotype
❏ Barr bodies
❏ Abdominal ultrasound
❏ Magnetic Resonance Imaging
99. Which of the following examinations can best determine the presence of enterocoele?
❏ Inspection of the external genitalia
❏ Speculum examination
❏ Internal examination
❏ Rectovaginal examination
100. Heavy menstrual bleeding is defined as mean blood loss (in ml) of greater than or equal
to:
❏ 60
❏ 70
❏ 80
❏ 90
cjhernandez/djhorneja/ccorpuz
❏ Aortic stenosis
❏ Mitral stenosis
7. What is the ideal maintenance anti-HTN drug for diabetic patients with nephropathy?
❏ Metoprolol
❏ Amlodipine
❏ Enalapril → ACEIs and ARBs are renoprotective agents
❏ Spironolactone
8. In a patient with HFrEF who remains symptomatic despite taking ACE inhibitors, Beta
blockers and MR antagonists, what drug can replace ACE inhibitors?
❏ Ivabradine
❏ Angiotensin receptor neprilysin inhibitor → Valsartan/sacubitril
❏ Trimetazidine
❏ Amlodipine
9. What cardiac biomarker rises within 4-8 hours and remains elevated for 7-14 days?
❏ Troponin T
❏ Myoglobin → earliest but nonspecific; rises 2-4hrs, peaks at 6-12 hrs, return
to baseline after 24 hrs
❏ CK MB → rises 4-6 hrs, peak 12-24 hrs, return to baseline after 48 hrs
❏ LDH → rise 8-12 hrs, peak 24-48 hrs, return to baseline after 7 days
2 lead ECG shows > 5 mm ST segment elevation and significant q waves in leads
10. 1
V5-V6, I and AVL. What part of the myocardium is involved?
❏ Anterior → V3, V4
❏ Posterior
❏ Inferior → II, III, AVF
❏ Lateral
*Septal → V1, V2
11. A 45 year old male was brought to the ER because of sudden loss of consciousness. BP
0, HR 0, RR 0. Chest compressions were immediately started. ECG showed:
cjhernandez/djhorneja/ccorpuz
❏ Asystole
❏ Atrial fibrillation
❏ Ventricular tachycardia
❏ Ventricular fibrillation
12. A hypertensive patient, who does not take her medications regularly, was suddenly
awakened because of severe constricting chest pain as well as a “drowning sensation” and
diaphoresis. At the ER, BP was 200/110, CR 120 irregular, and there were soft heart sounds
and crackles over both lung fields. What does the patient have?
❏ Stage 1 HTN
❏ Stage 2 HTN
❏ Hypertensive urgency
❏ Hypertensive emergency → presence of end organ damage “Flash Pulmonary
edema”
cjhernandez/djhorneja/ccorpuz
13. What is the immediate management for this patient?
❏ Sublingual clonidine 150 mcg every 15-30 minutes until BP stabilizes, then
discharge the patient and have her follow up after 1 week
❏ Losartan 100 mg + Amlodipine 10 mg, discharge patient and follow up after 3
days
❏ Admit to regular room, start nicardipine drip
❏ Admit to monitored bed, start Isoket drip and Furosemide
15. A 60 year old hypertensive patient is brought to the ER with sudden slurring of speech
and left sided weakness. HR was 136, irregularly irregular, and an atrial fibrillation was
suspected. What PE finding is consistent with this arrhythmia?
❏ Varying intensity of S1
❏ Loud and widely split S2 →Atrial Septal Defect
❏ Left sided S3 → CHF
❏ Muffled S1 and S2 → pericardial effusion or anything that increases distance
from heart and chest wall
15. A 55-year-old hypertensive male, a 20-pack-year smoker, consulted due to dyspnea. He
says that he has been having progressive exertional dyspnea for the past 3 years. In the past
month, he has had to use 2-3 pillows at night to “breathe better”. He has verbalized that the
dyspnea and coughing would last a few weeks, would eventually resolve only to return again
after a few months. Initial PE revealed symmetrical but limited chest expansion, decreased
breath sounds, and occasional expiratory wheezes. What is the most likely condition?
❏ COPD
❏ Bronchial asthma → intermittent episodes of dyspnea and coughing with
symptom free interval; case is progressive
❏ Congestive heart failure → possible due to 2-3 pillow orthopnea but chest PE
points more to COPD
❏ Anginal equivalent
16. A 65-year-old male with a 20-pack-year smoking history presents with a history of
increasing dyspnea, weight loss, and intermittent coughing. Chest x-ray revealed a hilar
mass on the right hemithorax causing partial atelectasis of the right upper lobe. What is the
most likely PE finding?
❏ Bilateral crackles over lung bases occasional wheezing bilateral lung bases
❏ Unilateral wheeze on the right → hilar mass cause compression of right main
bronchus
❏ Dullness on right lung base, increased tactile and vocal fremiti
❏ Tracheal shifted to the left, decreased breath sounds over the right hemithorax
cjhernandez/djhorneja/ccorpuz
17. What spirometry parameter identifies the SEVERITY of airway obstruction in
Emphysema?
❏ FEV1 → Flow parameter
❏ FEV1/FVC → screening <0.70 (Obstructive Lung Defect); Flow Parameter
❏ FEF 25-75 → smaller airways
❏ FVC → volume parameter (surrogate for TLC); Restrictive Lung Defect
18. What parameter in the GOLD guidelines should make the physician prescribe an ICS to a
patient with Emphysema?
❏ Frequent exacerbations
❏ Severity of airway obstruction
❏ Symptoms score
❏ Oxygen saturation
*Usually, patients with COPD not prescribed with ICS, only LAMA and LABA
cjhernandez/djhorneja/ccorpuz
19. What PE finding should make the physician prescribe home O2 inhalation?
❏ Increased AP diameter of the chest
❏ Right parasternal heave
❏ Apex beat at 6th LICS AAL
❏ Bipedal edema → sign of right sided heart failure
20. A patient had an arterial blood gas drawn while on O2 inhalation at 2 LPM. pH 7.33
PaCO2 68 PaO2 105 HCO3 29 O2sat 98%. What is the acid base status?
❏ Partially compensated met alkalosis
❏ Uncompensated respi alkalosis
❏ Uncompensated metabolic acidosis
❏ Partially compensated respiratory acidosis → HCO3 increases and tries to
compensate but the pH is still not within the normal range (compensated →
pH within normal)
21. What IS NOT included as initial empiric therapy for a patient with Community Acquired
Pneumonia, mod risk?
❏ Doxycycline → can be empiric therapy for low risk CAP
❏ Sultamicillin
❏ Levofloxacin
❏ Azithromycin
*MOD RISK - B Lactam, Macrolide, Respiratory Fluoroquinolone
cjhernandez/djhorneja/ccorpuz
* Low-risk CAP:
● Without co-morbid illness:
○ Amoxicillin 1 gm TID OR Azithromycin 500 mg OD OR Clarithromycin
500 mg BID
● With stable co-morbid illness:
○ β-lactam/β-lactamase inhibitor combination (BLIC)b OR 2nd gen
oral cephalosporins +/- extended macrolides
■ Co-amoxiclav 1 gm BID OR Sultamicillin 750 mg BID OR
■ Cefuroxime axetil 500 mg BID +/- Azithromycin 500 mg OD
OR
■ Clarithromycin 500 mg BID
*Moderate risk CAP:
● IV non-antipseudomonal β-lactam (BLIC, cephalosporin) + extended
macrolides or respiratory fluoroquinolones (PO)
○ Ampicillin-Sulbactam 1.5 gm q6h IV OR Cefuroxime 1.5 g q8h IV OR
○ Ceftriaxone 2 g OD + Azithromycin 500 mg OD PO OR
○ Clarithromycin 500 mg BID PO OR
○ Levofloxacin 500mg OD PO OR Moxifloxacin 400 mg OD PO
● If aspiration pneumonia is suspected and, a regimen containing ampicillin
sulbactam and/or moxifloxacin is used, there is no need to add another
antibiotic for additional anaerobic coverage
● If another combination is used may add clindamycin to the regimen to cover
microaerophilic streptococci
○ Clindamycin 600mg q8h IV OR Ampicillin-Sulbactam 3 g q6h IVOR
Moxifloxacin 400mgODPO
*High-risk CAP
● No risk for P. aeruginosa IV non-antipseudomonal:
○ Β-lactam + IV extended macrolides or IV respiratory
fluoroquinolones
○ Ceftriaxone 2 gm OD OR
○ Ertapenem 1 gm OD + Azithromycin dihydrate 500 mg OD IV OR
○ Levofloxacin 500mg OD IV OR Moxifloxacin 400mg OD IV
● Risk for P. aeruginosa IV antipneumococcal antipseudomonal β-lactam
○ (BLIC, cephalosporin or carbapenem) + IV extended macrolides +
aminoglycoside
● If MRSA pneumonia is suspected, add Vancomycin 15 mg/kg q8-12 h OR
Linezolid 600mg q12h IV OR Clindamycin 600mg q8h IV
22. The patient was started on different antibiotics, but seemed to worsen after 2 days. A
repeat x-ray revealed Pleural effusion on the right. What is the importance of identifying
complicated parapneumonic effusion?
❏ We high doses of antibiotics
❏ We need to drain the fluid
❏ We need to cover for anaerobic organism
❏ The patient is high risk of having bacteremia
*Complicated Parapneumonic effusion → organisms are already in the pleural fluid
23. Which of the following will best identify a complicated parapneumonic effusion?
❏ pH 7.5 → 7.2
❏ Presence of Gram (-) bacilli → as long as there is presence of bacteria, it
becomes complicated; Management is to drain the fluid; antibiotics alone will
not resolve this
❏ Pleural fluid protein: serum protein ratio 0.9 → exudate but not necessarily
complicated parapneumonic effusion
❏ Turbid appearance → can also be found in exudates but not necessarily in
complicated parapneumonic effusion
cjhernandez/djhorneja/ccorpuz
24. Thoracentesis was done. Five minutes into the procedure, the patient had sudden
worsening dyspnea. What PE maneuver will best identify the most probable cause of the
sudden dyspnea, considering the clinical scenario?
❏ Inspection
❏ Palpation
❏ Percussion → expect hyperresonance
❏ Auscultation
*Sudden worsening dyspnea → may have punctured the lung
25. A 40-year-old teacher was diagnosed to have Asthma in December 2013, and was
maintained on ICS. She also underwent Spirometry, but the results were “normal”. How can
we best confirm a diagnosis of Bronchial Asthma?
❏ Wait for her to have symptoms and then do the spirometry
❏ Let her come to your clinic every week so you can measure her PEFR → better
if patient sent home and monitor peak flow at home day and night, then
compute for variability
❏ Request for an arterial blood gas → may not be helpful
❏ Do methacholine challenge test → expect drop in flow parameters
26. A 68 year old tricycle driver consulted and was told that he had TB. He says that he was
started on anti TB medications 2 years ago but he stopped these after 3 months because he
experienced joint pains. His current sputum smears are (+) for TB. What regimen should he
be started on?
❏ 2RHEZ/ 1RHEZS/ 5RHE
❏ 2RHEZ/ 4RHE → Categ 1
❏ 2RHEZS/ 1RHEZ/ 5RHE → Categ 2
❏ 9RHE
cjhernandez/djhorneja/ccorpuz
27. A patient with lung cancer was diagnosed to have Deep Venous Thrombosis. What PE
finding is most common for DVT?
❏ Pain in calf on dorsiflexion of foot → Homan’s sign; “most specific”
❏ Palpable cord
❏ Unilateral leg edema → “most common”
❏ Decreased dorsalis pedis and posterior tibialis
28. A 60 year old male consults for hemoptysis. On chest x-ray, a left hilar mass is seen.
What procedure is most appropriate for this patient?
❏ CT-guided aspiration biopsy → NOT recommended because mass is in the
middle and have to traverse the lung parenchyma
❏ Fiberoptic bronchoscopy
❏ Resection biopsy via thoracotomy → invasive
❏ VATS guided lung biopsy → invasive
29. A 47 -year-old chef consults because of excessive daytime sleepiness. A sleep disorder is
suspected. What is TRUE about Obstructive Sleep Apnea?
❏ His chest and abdomen are seen to move even if there is no airflow
❏ His snoring suddenly stops when he goes to REM
❏ There is absence of chest and abdomen movement synchronous with
hypoxemia
❏ Hypoxemia occurs even during the daytime
*OSA → only during the night and due to obstruction when patient is asleep; no
snoring → no airflow
30. Which drug has a narrow therapeutic index, and is both nephrotoxic and ototoxic?
❏ Vancomycin
❏ Imipinem cilastatin → nephrotoxic
❏ Furosemide → nephrotoxic and ototoxic; NOT narrow therapeutic index
❏ Cyclophosphamide → nephrotoxic: hemorrhagic cystitis
31. When there is reduced renal perfusion, which mechanism will aid in maintaining normal
glomerular capillary pressure?
❏ Vasoconstriction of the afferent arteriole
❏ Vasoconstriction of the efferent arteriole
❏ Vasodilatation of the efferent arteriole
❏ Vasodilatation of the systemic vessels
32. A 40-year-old male with chronic glomerulonephritis has altered sensorium and seizures.
PE: BP is 160/90 mmHg, he has crackles on the right lung base, and (+) asterixis. Labs:
creatinine 10 mg/dL (0.6-1.2), sodium 135 meq/L, potassium 5.3 meq/L (3.5-5). What is the
indication for Dialysis?
❏ Azotemia
❏ Hypertension
❏ Hyperkalemia
cjhernandez/djhorneja/ccorpuz
❏ Uremia
*Indications for Dialysis: A - Acidosis (Intractable) E - electrolyte imbalance
(intractable hypokalemia) I - intoxication O - volume Overload (intractable) U - uremia
33. Which of the following may be seen in the ECG tracing of a patient with hyperkalemia?
❏ Peaked T waves
❏ ST depression
❏ ST elevation
❏ U waves
34. A 20-year-old female with sore throat and cough consulted 2 weeks later due to
cola-colored urine. BP 150/90 mmHg. Urinalysis: pus cells 30-35/hpf, RBC 50/hpf, (+) RBC
casts, epithelial cells few, nitrite negative. Serum creatinine is high. What renal syndrome is
present?
❏ Nephritic → high blood pressure with hematuria
❏ Nephrotic → proteinuria, edema, hypoalbuminemia, hyperlipidemia
❏ UTI
❏ Urinary Tract Obstruction
35. A 44-year-old pedicab driver was admitted due to fever, calf pains and oliguria 1 week
after torrential rains. He was diagnosed to have acute kidney injury. Which laboratory result
will he most likely have?
❏ BUN/creatinine ratio 20:1
❏ FENa >1%
❏ Urine sodium <10 mmol/L
❏ Urine specific gravity >1.030
36. What microbial factor mediates binding to uroplakins on the luminal surface of bladder
uroepithelial cells?
❏ CXCR1
❏ Cytotoxin
❏ O fimbria
❏ Type 1 pili
37. Which of the following is an expected finding in a patient with CKD?
❏ Hypernatremia
❏ Hyperphosphatemia
❏ Hypochloremia
cjhernandez/djhorneja/ccorpuz
❏ Hypokalemia
38. A 31-year-old fireman suffered multiple physical injuries when he was trapped in a
collapsed building. PE: BP of 120/70 PR 100/min, RR 27/min, T 36¼C. (+) Multiple hematoma
on the trunk and extremities.
Labs Result Reference Values
BUN 100 mg/dl 7-20
Creatinine 10 mg/dl 0.6-1.2
FENa >1%
Based on the clinical presentation and lab work-up, what is the working diagnosis?
❏ Acute kidney injury, pre-renal type
❏ Acute kidney injury, post-renal type
❏ Acute kidney injury, intrinsic/renal type
❏ Chronic kidney disease
cjhernandez/djhorneja/ccorpuz
41. What medication should be stopped in a Diabetic and Hypertensive patient with
advanced chronic kidney disease and metabolic acidosis?
❏ Glipizide
❏ Metformin → may cause lactic acidosis
❏ Metoprolol
❏ Verapamil
42. Which medication can confer renoprotection in a patient with CKD and significant
proteinuria?
❏ Amlodipine
❏ Furosemide
❏ Nebivolol
❏ Ramipril
43. A 65 year old female presents with back pain. What additional information suggests a
potentially serious condition requiring further work-up?
❏ Spine radiograph showing osteophytes
❏ Back pain is worse on recumbency, specially at night
❏ She lost over 3 kg in the past 3 months because of anorexia following a family
dispute
❏ She has minimal relief with Paracetamol 500 mg taken TID for the past 5 days
44. A 36 year old female diagnosed with SLE 8 years ago is seen at the emergency room.
Which clinical data will most require aggressive immunosuppressive therapy?
❏ She is also on medications for a manic- depressive disorder
❏ Edema, proteinuria and cylindruria
❏ Leucopenia 3.2 x 109/L
❏ Slightly tender and swollen wrists and elbows
45. What is the most recognized complication of hyperuricemia?
❏ Nephrolithiasis
❏ Gouty arthritis
❏ Metabolic disease
❏ Obesity
*Hyperuricemia is defined as a serum urate concentration greater than 7 mg per dL
46. What is the most common extra-articular manifestation of ankylosing spondylitis?
❏ Sacroiliitis
❏ Vertebral compression fracture
❏ Uveitis
❏ Erythema nodosum
47. What is the initial drug of choice for patient with reactive arthritis?
❏ NSAIDs
cjhernandez/djhorneja/ccorpuz
❏ DMARDs
❏ Glucocorticoids
❏ Immunotherapy
48. A 28-year-old teacher consulted because of painful right wrist most especially on lifting
objects like her handbag. There are neither gross deformities, warmth, nor swelling of the
wrist joint, but there is a positive Finkelstein’s test. What condition does the patient have?
❏ De Quervain’s tenosynovitis
❏ Anserine bursitis
❏ Carpal tunnel syndrome → (+) Phalen’s Test, Tinel, Reverse Phalen
❏ Medial epicondylitis
49. A 20-year-old healthy soccer player was admitted due to acute left knee synovitis of 5
days duration. Synovial fluid aspirate revealed WBC of 50,000 with predominance of
segmenters. Gram’s stain showed no organism, cultures did not yield any growth. What will
be the choice of antibiotic in this case?
❏ Ceftazidime
❏ Ceftriaxone
❏ Cefazolin
❏ Cefuroxime
50. What is the minimum duration of treatment for septic arthritis?
❏ 1 week
❏ 2 weeks
❏ 3 weeks
❏ 4 weeks
51. A 32 year old Lupus patient has been on glucocorticoids for the past 7 years. She now
complains of progressive right hip pain especially on ambulation and when riding the
jeepney. She walks with a limp. There is painful limited abduction and internal rotation of
the right jip. What diagnostic procedure is most helpful?
❏ MRI
❏ CT scan
❏ Plain radiograph
❏ PET scan
cjhernandez/djhorneja/ccorpuz
52. What pathogen is an etiologic cause of reactive arthritis?
❏ Staph aureus → most common etiology of septic arthritis
❏ Yersinia enterocolitica → reactive arthritis usually occurs after GI/GU
infection
❏ Strep pyogenes
❏ Pseudomonas aeruginosa
53. Which is true about Addison’s disease?
❏ The most common cause is adrenal hemorrhage.
❏ Hypopigmentation is one of the features.
❏ An overnight dexamethasone suppression test is a good screening test.
❏ Mineralocorticoid deficiency is common → Addison’s is primary adrenal
insufficiency
54. Which is the best biochemical marker for recurrence of medullary cancer after a
complete thyroidectomy and radioactive iodine ablation?
❏ Thyroglobulin
❏ Thyroid binding albumin
❏ Anti-TPO antibody
❏ Calcitonin
55. What is the most common cause of hyperthyroidism?
❏ Toxic adenoma
❏ Thyrotoxicosis factitia → hyperthyroidism due to excessive exogenous thyroid
hormones
❏ Graves disease
❏ Jod Basedow phenomenon → iodine-induced hyperthyroidism (escape
phenomenon of Wolff-Chaikoff effect)
56. What is the most dreaded side effect of thionamides?
❏ Lactic acidosis
❏ Steven Johnson disease
❏ Agranulocytosis
❏ Arrhythmias
57. Impaired glucose tolerance (Pre-diabetes) is defined by a (75-gram) 2-hour glucose level
of:
❏ 100-125 mg/dL
→ Impaired fasting glucose
❏ 126-140 mg/dL
❏ 140-199 mg/dL
❏ 140-219 mg/dL
cjhernandez/djhorneja/ccorpuz
58. The peak of B-HCG at the end of the first trimester can cause mild hyperthyroidism.
B-HCG mimics the action of which of the following hormones?
❏ TRH (Thyroid Releasing Hormone)
❏ TSH (Thyroid Stimulating Hormone)
❏ T4 (Thyroxine)
❏ T3 (Triiodothyronine)
59. Which is the primary counterregulatory hormone that prevents hypoglycemia?
❏ Cortisol
❏ Epinephrine
❏ Glucagon
❏ Growth hormone
60. Which drug specifically acts to block the renal reabsorption of glucose?
❏ DPP-IV inhibitors
❏ Alpha-glucosidase inhibitors
❏ SGLT-2 inhibitors
❏ GLP-1 agonists
61. A 50-year-old female patient known diabetic and hypertensive for 10 years recently
complained of episodes of exertional dyspnea, bipedal edema, nocturnal cough and 2 pillow
orthopnea, which of the following oral hypoglycemic medications are contraindicated for
this patient?
❏ Metformin
❏ Glimepiride
❏ Sitagliptin
❏ Pioglitazone → contraindicated in CHF and liver cirrhosis
62. A 30-year-old female SLE patient went into remission in June, and discontinued all her
medications. Five days ago, she started having cough, body malaise, light headedness,
muscle weakness. BP was 120/80 at sitting but on standing dropped to 80/60. There was
hat condition is most likely?
hyperpigmentation of the skin on the dorsum of the arms. W
❏ SLE flare
❏ Dehydration
❏ Medication side effects
❏ Adrenal insufficiency → sudden discontinuation of steroids
63. Which statement is true of DKA?
❏ Bicarbonate is routinely given when serum bicarbonate is low → not routine
❏ 0.9% NaCl is preferred over 0.3% NaCl in dehydrated state
❏ Subcutaneous insulin is usually preferred over IV insulin → SC is not given due
to inconsistent absorption
❏ Administering insulin usually raises serum potassium level → decreases
potassium level
cjhernandez/djhorneja/ccorpuz
64. Impingement of the pituitary stalk by a large suprasellar tumor may cause a rise in which
anterior pituitary hormone?
❏ TSH
❏ Growth hormone
❏ ACTH
❏ Prolactin
65. A 40-year-old female patient came into the clinic complaining of epigastric pain 2-3 hours
after eating. This was accompanied by burning sensation going upward into the chest. Which
treatment is most suitable for this patient?
❏ AlMgOH take 1 tablet as needed → antacid not as good as PPI
❏ Omeprazole 20mg/tablet 1 tablet OD → most effective in acid reduction
because it blocks the final pathway for acid production
❏ Ranitidine 150 mg bid
❏ Prednisone 10mg/tab 1 tab OD
66. Which lesion is the most common risk factor for adenocarcinoma of the esophagus?
❏ Barrett’s
❏ Papilloma
❏ Stricture
❏ Web
67. What is the best test for Helicobacter pylori in a patient on proton pump inhibitor (PPI)?
❏ Histology
❏ Rapid urease test
❏ Stool antigen
❏ Urea breath test
68. M.M., a 39-year old, entrepreneur, smoker, has been having chronic burning epigastric
pain, triggered by hunger and relieved by food intake, which would occasionally awaken him
in the early morning. For the past two days, he has been having tarry stools. Where is the
most likely source of GI bleeding?
❏ Esophagus
❏ Stomach
❏ Duodenum → Duodenal ulcer presents with classic night awakening
❏ Terminal Ileum
cjhernandez/djhorneja/ccorpuz
69. Which of the following is the test of choice to document H. Pylori eradication following
treatment?
❏ Rapid urease test
❏ Urea breath test
❏ Gastric mucosal biopsy
❏ H.pylori serological test
70. A 70-year old male, hypertensive, presents with melena and later massive hematochezia.
Gastroscopy and colonoscopy showed negative results. What is the next most appropriate
step to do?
❏ Angiography → next best step if you cannot visualize
❏ CT scan
❏ Enteroscopy
❏ Tagged RBC scintigraphy
71. A. M. 41-year old, obese, female, previously well, complains of an acute right upper
quadrant pain. This was followed by jaundice and fever. There was no weight loss. What is
the best initial diagnostic test for this patient?
❏ Endoscopic retrograde cholangiopancreatography → although already
therapeutic, not an initial diagnostic test to be done
❏ Magnetic resonance cholangiopancreatography
❏ Percutaneous transhepatic cholangiography
❏ Ultrasound of the upper abdomen → INITIAL diagnostic test
72. A 60-year-old male has been having occasional postprandial epigastric pain radiating to
the right upper quadrant for six months. Two days before consultation, he complained of
severe, unrelenting epigastric pain radiating to the back. No vomiting. On P.E. in distress,
BP= 130/80, CR=110/min, RR= 26/min, afebrile, with pink palpebral conjunctivae, (+)
epigastric tenderness, no hepatomegaly. What is the most likely diagnosis?
❏ Acute pancreatitis
❏ Calculous cholecystitis
❏ Hepatitis A
❏ Liver Abscess
*BISAP scoring: *Ranson’s criteria
cjhernandez/djhorneja/ccorpuz
73. A 32-year-old female, asymptomatic, has elevated transaminases. She claims to have
complete immunization. She had a history of blood transfusion. She denies illicit I.V. drug
use. Which of the following tests should be done?
❏ IgM anti-HAV
❏ IgM anti-HBc
❏ Anti-HCV
❏ Anti-HDV
74. In a patient with jaundice, what is a P.E. finding that will support a diagnosis of liver
cirrhosis with portal hypertension?
❏ Caput medusa → blood vessels are already engorged
❏ Gynecomastia → hallmark of liver cirrhosis but not portal hypertension
❏ Icteric sclera → decompensated liver cirrhosis
❏ Spider angioma → hallmark of liver cirrhosis but not portal hypertension
(hyperestrogenism)
75. Y.M., a 68-year-old female, obese, has abdominal enlargement and pedal edema. She
denies alcohol intake. On P.E., she has icterisia, distended abdomen, (+) shifting dullness,
and palmar erythema. Laboratory tests show, ALT=3X ULN, reversed A/G ratio, prolonged
prothrombin time. What is the most likely cause of jaundice?
❏ Increased bilirubin production
❏ Decreased conjugation of bilirubin
❏ Hepatocellular dysfunction
❏ Common bile duct obstruction
*Decompensated liver cirrhosis → presence of icterisia and ascites (+) shifting
dullness
76. What is the mechanism of the development of portosystemic encephalopathy in liver
cirrhosis?
❏ Deposition of bile acids in the skin → cause intense pruritus
❏ Diminished catabolism of androstenedione
❏ Imbalance of branched chain and aromatic amino acids
❏ Portal hypertension
77. A 56-year-old female presents with ascites. What is next best step to do?
❏ CT Scan of the abdomen
❏ Paracentesis → to know etiology of ascites
❏ Start diuretics
❏ Ultrasound of the abdomen → may not give additional information since you
already know that patient has ascites
78. Which is a true statement regarding the diagnosis of tetanus?
❏ On microbiologic evaluation, C. tetani is gram-negative and anaerobic. → gram
positive
❏ Recovery of C. tetani from the wound pins down the diagnosis of the illness
cjhernandez/djhorneja/ccorpuz
❏ Risk factors and clinical manifestations are the basis for the diagnosis of
tetanus
❏ Increasing the duration of incubation increases the recovery of C. tetani
79. A 19-year-old student presented with 6 days fever, myalgia and backpains. On PE, she
was afebrile with petechiae on the soft and hard palate. To confirm your diagnosis, what
should you request for?
❏ ALT and AST
❏ Dengue IgM
❏ Dengue NS1 → detected within 1-2 days of symptom onset
❏ PT and aPTT
80. A 24-year-old military recruit residing with his platoon developed high fever, nuchal
rigidity, altered consciousness and rapid onset of purpuric rashes over the trunk and lower
extremities. What is the drug of choice for his condition?
❏ Penicillin G
❏ Ceftriaxone
❏ Ampicillin sulbactam
❏ Azithromycin
81. What organism usually causes headache, fever and vomiting i n a patient with HIV?
❏ Candida albicans
❏ Cryptococcus neoformans
❏ Neisseria meningitidis
❏ Streptococcus pneumonia
82. What historical data best favors the diagnosis of malaria?
❏ Family history of malaria
❏ Fever, chills and sweating
❏ History of travel to Palawan
❏ Residence in Manila
83. What does refractory shock mean?
❏ Blood pressure of < 80/50 mmHg which rises after infusion of saline solution
❏ Blood pressure of 80/50 mmHg > 1 hour after aggressive hydration and use of
dopamine and norepinephrine → hypotension after fluid resuscitation and
vasopressors
❏ Improvement of MAP to 70 mmHg after infusion of saline solution
❏ MAP of 80 mm Hg
84. In which of the following conditions is the treatment of the sexual partner of patients
suffering from vaginal infection very important?
❏ Bacterial vaginosis
❏ Mycoplasma vaginalis
❏ Trichomonas vaginalis → sexually transmitted
cjhernandez/djhorneja/ccorpuz
❏ Vulvovaginal candidiasis
85. Which of the following drugs should be given as chemoprophylaxis against Malaria in
pregnant women?
❏ Chloroquine
❏ Doxycycline
❏ Mefloquine
❏ Primaquine
86. When should you give rabies immune globulin as post exposure prophylaxis?
❏ After 1 week
❏ Any time
❏ Rabies immune globulin is not needed
❏ Within 7 days of exposure
*Passive immunization: RIG (20 IU/kg) should not be given later than 7 days
after the 1st vaccine dose
*Active immunization:
● Purified Vero cell Rabies Vaccine (PVRV): 0.5mL/dose at D0,3,7,14
● Purified Chick embryonal cell Rabies Vaccine (PCECV): 1mL/dose IM at
day 0,3,7,14
○ ID route: 0.1mL at 2 sides/dose at D0,3,7,14
*Pre-exposure prophylaxis: D0, 7, 21/28; if bitten by a dog with PEP, give
booster at D0 and D3
*Without pre-exposure prophylaxis:
● Post-exposure prophylaxis category II: active imm (4 doses) at D0, 3,
7, 14
● Post-exposure prophylaxis category III: active imm (4 doses) at D0, 3,
7, 14 + Immunoglobulin
87. SR, a 35-year-old male, presented with painless papule on the tip of his penis which he
noted 2 weeks after casual sexual contact. Which drug should he be given?
❏ Acyclovir
❏ Azithromycin
❏ Aqueous penicillin
❏ Benzathine penicillin G → DOC for syphilis
88. JB, a 46-year-old male, presented with fever, crampy abdominal pain, with tenesmus and
bloody mucopurulent stool. He subsequently developed renal failure, pallor and
thrombocytopenia? Which is the likely etiologic agent?
❏ EHEC → Hemolytic uremic syndrome, caused by E0157:H7
❏ Salmonella
❏ Shigella
❏ Vibrio cholera
89. A 30 year old medical technologist has fever of 10 days accompanied by abdominal pain
and constipation. BP 110/80, CR 80, T 20, T 39.8C. She also had icterisia and
hepatosplenomegaly. What is the most likely diagnosis?
❏ Chikungunya
cjhernandez/djhorneja/ccorpuz
❏ Dengue
❏ Enteric fever → prolonged fever
❏ Viral hepatitis
90. A 23 year old inquired about HIV testing given a history of sexual contact with a
commercial sex worker 4 days ago. What should you recommend?
❏ Antibody testing after a week
❏ Antibody testing after 2 weeks
❏ P24 antigen testing during consultation
❏ P24 testing after 2 weeks
91. A 40 year old patient complains of fever, myalgia and RUQ pain. BP 120/70, HR 92, RR 19,
T 39.8C. He also has faint maculopapular rash on the dorsum of the left foot which he
noticed 2 weeks ago after swimming in a freshwater lake in Agusan. Liver span = 15 cm along
RMCL. Hgb 110, WBC 15,000 (seg 60, lym 20, eos 20), plt 160,000. UTZ showed hyperechoic
fibrotic bands along the portal vessels. Stool exam did not show parasite ova, and he refused
proctoscopy. What is our empiric treatment?
❏ Albendazole
❏ Mebendazole
❏ Praziquantel → DOC for Schistosoma and all other flukes
❏ Pyrantel pamoate
*Symmer’s fibrosis → hyperechoic fibrotic bands along the portal vessels caused by
Schistosoma mansoni
92. Mrs. A.B. is a 60-year old female who underwent modified radical mastectomy, right,
with histopathology findings of Invasive Ductal Carcinoma, T1N0M0. Immunohistochemical
assay showed: ER strongly positive, PR strongly positive, HER2 negative. What is the
adjuvant treatment of choice?
❏ Anthracycline-based chemotherapy
❏ Anti-HER2 targeted therapy
❏ Aromatase inhibitor
❏ Selective estrogen receptor modulator
93. A 19 year old male noticed axillary lymphadenopathies in the past month. A biopsy of
the LN reveals Hodgkin’s lymphoma, nodular sclerosing variety. A bone marrow biopsy is
scheduled. What other procedure is also part of the routine staging evaluation for patients
with Hodgkin’s lymphoma?
❏ PET scan
❏ Gallium scan
❏ Staging laparotomy
❏ CT of chest, abdomen and pelvis
94. Which of the following cells is diagnostic for Hodgkin's Lymphoma?
❏ Faggot cells → Seen in Acute Promyelocytic Leukemia
❏ Nuclear inclusions
❏ Pincenez cells
cjhernandez/djhorneja/ccorpuz
❏ Reed Sternberg cells
95. What is the diagnosis of a patient presenting with the triad of portal vein thrombosis,
hemolysis and pancytopenia?
❏ Acute promyelocytic leukemia
❏ Paroxysmal nocturnal hemoglobinuria
❏ Hemolytic uremic syndrome → Triad: Thrombocytopenia, Microangiopathic
hemolytic anemia, Renal Failure
❏ Thrombotic thrombocytopenic purpura → Pentad: Thrombocytopenia,
Microangiopathic hemolytic anemia, Fever, Renal Failure, Neurologic symptoms
96. A 24-year-old male with a history of poorly treated chronic ulcerative colitis is found to
have anemia with a hemoglobin of 9 g/dL and a reduced mean corpuscular volume. His
ferritin is 250 g/L. Which of the following is the most likely cause of his anemia?
❏ Folate deficiency
❏ Hemoglobinopathy
❏ Inflammation
❏ Iron deficiency
97. During a routine visit, a 68-year-old female complains of 3 months of fatigue, abdominal
fullness, and bilateral axillary adenopathy. On PE, vital signs are normal, and she has
bilateral palpable axillary and cervical adenopathy and splenomegaly. A complete blood
count is notable for a WBC count of 88,000 with 99% lymphocytes. Which of the following is
the most likely diagnosis?
❏ Acute lymphoblastic leukemia
❏ Acute myelogenous leukemia
❏ Chronic lymphocytic leukemia
❏ Hairy cell leukemia
98. Which of the following tumor characteristics confers a poor prognosis in patients with
breast cancer?
❏ Estrogen receptor positive
❏ Good nuclear grade
❏ Low proportion of cells in S-phase
❏ Overexpression of HER-2/Neu
99. A 42-year-old male received radiation exposure at a nuclear power plant. He is now
concerned about his future cancer risk. Which of the following statements is CORRECT?
❏ Malignancies occur within 10 years of exposure
❏ Leukemia has the shortest latency period of all malignancies
❏ Large exposure is required to develop the most serious malignancies
❏ Therapeutic radiation therapy given without chemotherapy does not increase
the risk of a second malignancy
cjhernandez/djhorneja/ccorpuz
100. What is the ECOG status of a patient who is capable of only limited self-care, confined
to bed or chair >50% of waking hours?
❏ ECOG 1
❏ ECOG 2
❏ ECOG 3
❏ ECOG 4
cjhernandez/djhorneja/ccorpuz
PHYSIOLOGY MOCK BOARD EXAM
UST MBR 2020
2. One of the following steps in the excitation contraction coupling is similar to both skeletal
and smooth muscles:
❏ activation of myosin
❏ exposure of active sites in actin
❏ release of calcium from sarcoplasmic reticulum
❏ binding of calcium to troponin C
*Rest of the choices are involved in skeletal muscle excitation contraction coupling
3. Force of contraction of the heart may be increased by:
❏ successive application of supramaximal stimuli
❏ stimulation of the Na-Ca pump
❏ inhibition of beta-1 receptor
❏ inhibition of the Na-K pump → Ca will remain inside the cytosol leading to
greater force of contraction; MOA of Digoxin given in Heart Failure
4. This is observed in the length-tension relationship of both cardiac and skeletal muscles:
❏ bell-shaped active tension curve
❏ length and tension are inversely related → directly related
❏ active and passive tension are inversely related at all lengths → bell shaped in
active, direct relationship in passive tension
❏ constant passive tension curve
cjhernandez/djhorneja/ccorpuz
5. This step in excitation contraction coupling is found both in skeletal and cardiac muscles:
❏ binding of calcium to troponin I → skeletal muscle
❏ release of calcium from sarcoplasmic reticulum is IP3-mediated → cardiac
muscle
❏ opening of dihydropyridine receptor → for cardiac muscles only; for skeletal
muscles → DHP receptor undergoes conformational change
❏ sequestration of calcium
*Cardiac muscles will require extracellular calcium unlike skeletal muscle
6. This is a voltage-gated ion channel:
❏ presynaptic calcium channel
❏ postsynaptic nicotinic sodium channel → ligand gated
❏ chloride channel coupled to GABA → ligand gated
❏ IP3 receptor
7. Potassium conductance is greatest during this phase of the action potential of a neuron:
❏ Depolarization → Na+ activation gates open; sodium influx
❏ Overshoot
❏ Repolarization → potassium efflux
❏ Afterpotential
cjhernandez/djhorneja/ccorpuz
❏ Subthreshold stimulus will result to a submaximal response → no such thing as
submaximal response
❏ Submaximal stimulus has no response → once threshold potential reached,
there is firing of AP
❏ Supramaximal stimulus gives rise to supramaximal response → no
supramaximal response, once threshold is reached action potential fires and
remains the same
9. In contrast to a chemical synapse, an electrical synapse:
❏ speed of communication is slower → chemical synapse
❏ pre and postsynaptic cells are separated by a synaptic cleft → chemical synapse
❏ neurotransmitter involved is aspartate → chemical synapse
❏ impulses are conveyed via gap junctions
10. At rest, this response is UNEXPECTED:
❏ gastric enzyme secretion
❏ increased salivary secretion
❏ relaxation of gastrointestinal sphincter
❏ Mydriasis → sympathetic response
11. This is transported in intestinal epithelial cells by a Na+-dependent cotransport process:
❏ fatty acids → move via facilitated diffusion
❏ Fructose → move via facilitated diffusion
❏ Alanine → neutral; passive diffusion
❏ Oligopeptides → Na-Oligopeptide cotransport
12. Cholecystokinin inhibits:
❏ pancreatic HCO3- secretion
❏ gastric emptying → proximal stomach is relaxed and pyloric sphincter is
contracted, gastric contents cannot empty
❏ contraction of gallbladder
❏ relaxation of sphincter of Oddi → bile moves to duodenum
*all other choices are stimulated by CCK
13. This is the site of Na+-bile acid cotransport:
❏ stomach
❏ duodenum
❏ Ileum → active reabsorption
❏ Colon → passive reabsorption of bile acids after deconjugation by colonic
bacteria
14. When parietal cells are stimulated, they secrete:
❏ HCl and intrinsic factor
❏ HCl and pepsinogen → pepsinogen is secreted by chief cells
cjhernandez/djhorneja/ccorpuz
❏ HCO3- and intrinsic factor
❏ mucus and pepsinogen
15. This is a lineage specific growth factor for megakaryocytes:
❏ G-CSF → granulocytes (basophils, eosinophils, neutrophils)
❏ GM-CSF → granulocytes and monocytes
❏ EPO → erythrocytes
❏ TPO
16. This is a fetal form of hemoglobin:
❏ alpha 2, gamma 2
❏ alpha 2, delta 2 → HbA2, variant of adult hemoglobin
❏ alpha 2, beta 2 → normal adult hgb
❏ alpha 2, epsilon 2 → embryonic; Gower 2 Hemoglobin
17. The following statements are correct, EXCEPT:
❏ The intrinsic pathway is usually triggered by thrombin formed by the extrinsic
pathway.
❏ Intrinsic pathway begins with exposure of blood to collagen from traumatized
tissue.
❏ The prothrombin time measures the intrinsic to common pathway → PT
extrinsic, aPTT - intrinsic
❏ Hemophiliacs have a defect in the intrinsic pathway → FVIII deficiency
18. Natural barriers to thrombin formation, EXCEPT:
❏ smooth endothelial surfaces
❏ protein C
❏ prostacyclin
❏ thromboxane A2 → produced by activated platelets are prothrombotic
*rest of choices are naturally occuring anticoagulant
19. Heparin is a potent anti-coagulant and its action is manifested by:
❏ prolonged bleeding time → platelet disorders
❏ prolonged PT - Warfarin
❏ prolonged PTT
❏ all of the above
20. TRUE of the Rh blood type:
❏ plasma with D antigen is Rh (+)
❏ incidence of Rh (-) among asians is high
❏ anti-Rh antibodies are formed at birth
❏ Rh (-) individuals can be repeatedly transfused with Rh (+) blood
cjhernandez/djhorneja/ccorpuz
21. The tendency for blood flow to be turbulent is increased by:
❏ increased hematocrit
❏ partial occlusion of blood vessel
❏ increased viscosity
❏ decreased velocity of blood flow → more laminar than turbulent
22. If a patient has a BP of 130/80 mmHg, the pulse pressure is:
❏ 113.3
❏ 96.6
❏ 80
❏ 50 → Pulse Pressure = SBP - DBP (130-80)
23. An increase in this parameter will decrease blood pressure:
❏ total peripheral resistance → inc BP
❏ afterload
❏ venous return → increase preload → inc stroke volume → Cardiac output
❏ Aldosterone → Inc water and Na+ retention which will inc BP
*BP= Cardiac Output x Total Peripheral Resistance
24. Decreased conduction in the AV node will manifest as prolongation of this parameter in
ECG:
❏ P wave
❏ PR interval
❏ QRS complex
❏ QT interval
25. Increase in heart rate can be due to:
❏ hypoxemia
❏ parasympathetic stimulation
❏ hyperkalemia
❏ hypercalcemia
*CD CASET - Carbon dioxide, Calcium, Sympathetic, Epinephrine and High Temperature
26. An increase in contractility is seen on a Frank-Starling diagram as:
❏ increased cardiac output for a given end diastolic volume
❏ increased cardiac output for a given end systolic volume
❏ decreased cardiac output for a given end diastolic volume → should be in CO
❏ decreased cardiac output for a given end systolic volume
cjhernandez/djhorneja/ccorpuz
If contractility held constant → inc in LVEDV → CO also increases
27. Characteristic of a cephalic phase of an integrated response to a meal:
❏ increase in sympathetic outflow → parasympathetic
❏ chewing occurs → oral phase
❏ stimulation of salivary gland secretion
❏ bolus of food stimulating mechanical receptors → esophageal or gastric phase
28. Responsible for the production of enzyme-rich pancreatic fluid:
❏ secretin
❏ Cholecystokinin → stimulates acinar cells to release secretion of pancreatic
juices and enzymes
❏ acetylcholine
❏ epinephrine
29. This transport mechanism allow the absorption of nutrients across the small intestinal
epithelial brush borders by facilitated diffusion:
❏ fatty acids → facilitated diffusion
❏ Galactose → secondary active Na transport SGLT1
❏ Glucose → secondary active Na transport SGLT1
❏ amino acids → Na dependent amino acid cotransporter
30. The transporter involved in the absorption of fluid in small intestines during the
postprandial period:
❏ Cl-HCO3 antiporter → chloride shift for
maintenance of acid base balance
❏ Glucose transporter 5 → fructose
cjhernandez/djhorneja/ccorpuz
❏ NaH antiporter → facilitation of H+ ions
❏ Na-glucose transporter 1 →
movement of
glucose and Sodium to
interstitium
drives water into the lumen due
to inc
Hydrostatic pressure
31. An ingested drug undergoes hepatic oxidation catalyzed by cytochrome P450. This is an
example of hepatic:
❏ Excretion
❏ Detoxification → 2 phases: phases 1 → CYP450 (HORD: Hydroxylation,
oxidation, reduction, deamination); phase 2 → glucuronidation, sulfation,
acetylation
❏ Regulation
❏ Synthesis
32. The highest pattern of bile acid synthesis and secretion rate is observed during:
❏ Surgical resection of ileum → bile acid turnover is increased
❏ Increased bile acid ingestion
❏ Colonic uptake of bile acids
❏ Intestinal hypomotility
33. During a fatty meal, the main source of bile acid pool in the enterohepatic circulation
comes from:
❏ Spill-over from liver into systemic circulation
❏ Absorption of deconjugated bile acids from colon
❏ Uptake of conjugated bile acids from ileum → Active reabsorption of bile via
Na-bile acid cotransport
❏ Hepatic synthesis of bile acids
cjhernandez/djhorneja/ccorpuz
34. This hormone enhances insulin secretion:
❏ IGF-1
❏ GLP-1 → aka Incretin, stimulated when orally ingest sugar/carbs; produced
by intestinal cells
❏ Somatostatin
❏ IGF-II
35. This is TRUE of luteinizing hormone:
❏ hypothalamic hormonal controller of LH is mainly inhibitory
❏ LH is not controlled by negative feedback
❏ LH is primarily controlled by inhibin → FSH is controlled by inhibin
❏ LH can be stimulated by a steroid hormone
*Hypothalamic controller of LH is GNRH
36. This hormone is synthesized from tyrosine:
❏ TSH → thyroid hormones and not TSH
❏ Epinephrine → along with Dopamine, norepinephrine and thyroid hormones
❏ Vasopressin → protein hormone
❏ Aldosterone
37. Class switching is facilitated in:
❏ B cell dependent T cell activation
❏ B cell independent T cell activation
❏ T cell dependent B cell activation → class switching is facilitated when T
helper cell activates B lymphocytes
❏ T cell independent B cell activation
38. Pathogen presented by MHC class I molecules:
❏ Ascaris lumbricoides
❏ Influenza virus - intracellular infection/pathogen will alter MHC Class I
❏ Mycobacterium tuberculosis
❏ Streptococcus pneumonia
*MHC class I found on all nucleated cells of the body
39. Advantage of meningococcal conjugated vaccine over the meningococcal polysaccharide
vaccine:
❏ Less costly and more stable
❏ Less pro-inflammatory cytokines elicited with less hypersensitivity → more
inflammatory reactions, better activation of B lymphocytes
❏ No difference between the two preparations
❏ Production of more memory cells
*conjugated vaccines → polysaccharides coupled to protein carrier, response to
vaccine is increased because T cells are also activated → more rapid and heightened
immune response
cjhernandez/djhorneja/ccorpuz
40. Condition related to the rapid production of IGE and release of histamine:
❏ Bronchial asthma → Type I Hypersensitivity involves IgE mediated response
❏ Glomerulonephritis → Type 3 Immune Complex Mediated Hypersensitivity
Reaction
❏ Goodpasture’s syndrome → Type 2 Complement Mediated Hypersensitivity
reaction
❏ Tuberculous Infection → Type IV - Delayed/Cell mediated
41. Characterized by complement mediated cell lysis:
❏ Bronchial Asthma → Type I Hypersensitivity involves IgE mediated response
❏ Endocarditis → Type 3 ADCC Hypersensitivity reaction
❏ Glomerulonephritis → Type 3 ADCC Hypersensitivity reaction
❏ Goodpasture’s syndrome → Type 2 Complement Mediated Hypersensitivity
(Other ex: Transfusion reactions, Autoimmune hemolytic anemia,
Erythroblastosis Fetalis)
42. Paul Walker suffered from a vehicular accident along EDSA. He had several head
contusions and abdominal trauma. His BP is 70/50, his pulses are weak and thread. The
following becomes operational:
❏ GFR will be decreased, with increased proximal tubule Na+ reabsorption.
❏ Increased filtered load of Na+ to help maintain H2O balance → decreased
filtered load of Na+ in hypotensive and hypovolemic patients
❏ Increased Na+ delivery to the distal tubule for more Na+ reabsorption →
decreased Na+ delivery
❏ Glomerulotubular balance will be maintained
43. The main tubular segment where Na+ reabsorption is adjusted:
❏ Proximal tubule
❏ TALH
❏ Distal tubule
❏ Collecting duct → will determine how much sodium goes to final urine
44. A patient has a pH of 7.39 and a PaCO2 of 35mmHg. The following conclusions can be
drawn regarding the acid base status of the patient.
❏ Normal acid-base status
❏ Compensated metabolic acidosis
cjhernandez/djhorneja/ccorpuz
❏ Partly compensated respiratory acidosis
❏ Normal anion gap metabolic acidosis
45. One of the following ABG results is compatible with that of a medicine board taker who
is hyperventilating secondary to panic attack after an examination:
pH pCO2
A 7.3 57
B 7.33 31
C 7.49 30
D 7.51 55
*hyperventilating → low CO2 & high pH → alkalotic
46. Anion gap may be determined as:
❏ Na+ + (HCO3- + Cl-) → should be Na - (HCO3 + Cl)
❏ unmeasured anion - unmeasured cation
❏ 2Na+ - (Cl- - HCO3-)
❏ 2Na+ + K+ + glucose + BUN
47. Vascular bed wherein hypoxia causes vasoconstriction:
❏ Coronary
❏ Pulmonary → hypoxic pulmonary vasoconstriction permits redistribution of
blood flow to better ventilated portion of lungs
❏ Cerebral
❏ Muscle
48. An 18 year old male has severe asthma and wheezing, to treat this patient, the physician
should administer:
❏ a1- adrenergic antagonist → found in CNS and PNS
❏ B1- adrenergic agonist → heart
❏ B2- adrenergic agonist → relax bronchial smooth muscle
❏ muscarinic agonist → for asthmatic patients give muscarinic antagonist
49. TRUE during inspiration:
❏ Intrapleural pressure is positive → negative
❏ The volume in the lungs is less than the functional residual capacity (TRC) →
beginning inspiration, lung volume at FRC
❏ Alveolar pressure is higher than atmospheric pressure → more negative so that
air will move from higher to lower concentration (mouth to lungs)
❏ Intrapleural pressure is more negative than it is during expiration
cjhernandez/djhorneja/ccorpuz
50. This volume remains in the lungs after a tidal volume (TV) is expired:
❏ Vital Capacity (VC) → IRV + TV + ERV
❏ Expiratory reserve volume (ERV)
❏ Residual volume (RV)
❏ Functional residual capacity (FRC) → ERV + RV
cjhernandez/djhorneja/ccorpuz
● Lung zone 1 - Pulmonary artery collapsed due to high pressure of alveoli (PA);
clinically not present; greatest ventilation because highest alveolar pressure
● Lung zone 2 - Pulmonary artery pressure is greater than pulmonary vein pressure
which collapses pulmonary vein
● Lung zone 3 - Pulmonary artery and pulmonary vein pressure greater than
alveolar pressure hence continuous blood flow; greatest blood flow
● Apex has greatest ventilation while bast has greatest blood flow
52. The site with the highest airway resistance:
❏ Trachea
❏ Largest bronchi
❏ Medium-sized bronchi → airway generation 5
❏ Alveoli
53. A 49 year old man has a pulmonary embolism that completely blocks blood flow to his
left lung. As a result, this will occur:
❏ Ventilation/perfusion (V/Q) ratio in the left lung will be zero
❏ Systematic arterial PO2 will be elevated → decreased due to low oxygen
❏ V/Q ratio in the left lung will be lower than in the right lung
❏ Alveolar PO2 in the left lung will be approximately equal to the PO2 in inspired
air
*in patients with pulmonary embolism or blockage of perfusion, pressure of CO2 and
O2 the same as inspired air because no gas exchange happen hence, VQ is infinite
*VQ of normal lung = 1
*VQ in obstruction of airway is less than 1
54. This condition could potentially lead to a coagulation defect in both extrinsic and
intrinsic pathways:
❏ Heritable defect involving von willebrand's factor → intrinsic defect
cjhernandez/djhorneja/ccorpuz
❏ Mutation in the genes coding for platelet Gp1B receptor → for binding of F8,
F10, F11 and thrombin so more on intrinsic defect
❏ Mutation in the genes coding for platelet Gp2B3A receptor → affects
fibrinogen; intrinsic and common pathway defect
❏ Blockage in bile flow → bile is important for fat digestion as well as fat
soluble vitamins (ADEK; Vit K is needed for activation of clotting factors 2,
7, 9, 10 involved in both intrinsic and extrinsic pathway)
55. A 15 year old male is diagnosed with Klinefelter's syndrome. This physical findings is
expected to be found on examination:
❏ Breast development, sparse body and facial hair and grossly male external
genitalia - 47XXY
❏ Short stature, swelling, broad chest, low hairline, low-set ears, and webbed
necks → Turner Syndrome 45XO
❏ Male phenotype with ambiguous genitalia → Pseudohermaphrodism
❏ Female phenotype with ambiguous genitalia → Pseudohermaphrodism
56. This is TRUE when comparing the cardiac output of the right and left heart:
❏ cardiac output is higher in the left side due to increased systemic vascular
resistance
❏ cardiac output is lower in the right side due to decreased pulmonary vascular
resistance
❏ Cardiac output in the right side of the heart is the same as the left side
❏ Cardiac output in the left side of the heart is doubled due to higher pressure in
the aorta
*right ventricles have smaller mass but due to increase pressure in pulmonary
arteries needed for left ventricle to overcome, it has greater mass; However, they
exert equal cardiac output
57. This condition suppresses the secretion of growth hormone:
❏ Decreased serum cortisol levels
❏ Increased dopamine secretion
❏ Decrease thyroid hormone levels
cjhernandez/djhorneja/ccorpuz
❏ Increased serum glucose → inhibits secretion of growth hormone; basis of
oral glucose challenge test
58. A 1 year old male was rushed to the hospital due persistent watery diarrhea. He looks
dry and was lethargic. This type of fluid when infused will not increase much the
extracellular fluid volume:
❏ Plain normal saline solution → remains in ECF; isotonic solution
❏ Plain lactated ringers solution → isotonic solution, remain in ECF
❏ D5W → hypotonic solution, tendency to go INSIDE of cell; will not help much
in increasing ECF among the other choices
❏ D5NSS → hypertonic in vitro; but in vivo/infused will be isotonic
59. A 50 year old male patient was rushed to the ER due to dull crampy generalized vague
abdominal pain. He had absence of bowel movement for 3 days already with minimal flatus
accompanied by vomiting and anorexia. He had a previous appendectomy. The abdominal
pain felt by the patient is most likely conducted by these nerve fibers:
❏ A alpha nerve fibers
❏ A delta nerve fibers → fast and sharp localized somatic pain
❏ B fibers
❏ C fibers → unmyelinated
60. 17 year old female patient with heavy menses, was noted to be pale and with easy
fatigability. This may bring about a decreased oxygen carrying capacity of the blood. One
knows that every 1g of Hgb can carry this much oxygen:
❏ 2.56mL
❏ 1.34 mL
❏ 100 mL
❏ 50 mL
61. In a nude person sitting inside a room at room temperature, majority of the heat loss is
through what mechanism?
❏ Evaporation
❏ Radiation
❏ Conduction to air
❏ Convection
62. All of the following are true of Type 1 Muscle fibers except?
❏ Also called slow fibers
❏ Have a more extensive blood vessel system and more capillaries to supply extra
amounts of oxygen
❏ Have a greatly increased number of mitochondria
❏ Have an extensive SR for rapid release of calcium ions to initiate contraction →
relies on phosphagen system
cjhernandez/djhorneja/ccorpuz
63. Which of the following drugs blocks transmission of action potential at the
neuromuscular junction?
❏ D-tubocurarine → blocks acetylcholine receptors on muscle fibers
❏ Physostigmine → acetylcholinesterase inhibitors
❏ Diisopropyl fluorophosphate
❏ Carbachol → cholinergic agonist, activate cholinergic receptors
64. During repolarization of the nerve action potential, opening of the potassium gate
causes:
❏ Sodium influx
❏ Sodium efflux
❏ Potassium efflux
❏ Potassium influx
65. The incisura of the aortic pressure is seen during which phase of the cardiac cycle?
❏ Rapid ventricular ejection
❏ Slow ventricular filling
❏ Isovolumetric relaxation
❏ Isovolumetric contraction
66. Phase 3 of the cardiac action potential is due to which of the following mechanisms?
❏ Opening of the fast sodium channels → phase 0
❏ Closure of the fast sodium channels
cjhernandez/djhorneja/ccorpuz
❏ Opening of the calcium channels → phase 2; plateau
❏ Opening of slow potassium channels
67. Which of the following factors does not increase lymph flow?
❏ Elevated capillary hydrostatic pressure
❏ Increased plasma colloid osmotic pressure → must be decreased
❏ Increased permeability of the capillaries
❏ Contraction of the surrounding skeletal muscles → movement of the body,
pulsation of surrounding vessels
68. What would be the effect of adding isotonic saline in the extracellular fluid
compartment?
❏ Increased extracellular fluid osmolarity
❏ Decreased intracellular fluid osmolarity
❏ Increased extracellular fluid volume → no movement of water; no change in
tonicity of solution
❏ Decreased intracellular fluid volume
69. Magnesium is primary reabsorbed in which part of the nephron?
❏ Proximal convoluted tubule → majority of ions absorbed here
❏ Loop of Henle
❏ Distal convoluted tubule
❏ Collecting tubules
70. Which of the following factors decreases renal calcium excretion?
❏ Hypervolemia
❏ Metabolic acidosis
❏ Hypophosphatemia
❏ Hyperparathyroidism → Inc PTH will inc serum Calcium via inc bone
resorption, inc Calcium reabsorption and decrease Calcium excretion
*Rest of the choices will promote calcium excretion
71. The basic rhythm of respiration is generated mainly by which collection of neurons in the
respiratory center?
❏ Ventral respiratory group → medulla, forceful breathing during exercise or
stress
cjhernandez/djhorneja/ccorpuz
❏ Dorsal respiratory group → basic rhythm of respiration
❏ Pneumotaxic center → Pons; switch off for inspiration
❏ Apneustic center → Pons, switch on for inspiration
72. Which of the following tactile receptors is particularly sensitive to movements of objects
over the surface of skin and to low-frequency vibration and is found in non-hairy part of the
skin particularly the fingertips and lips?
❏ Free nerve endings → pain, hot and cold temperature
❏ Meissner's corpuscles
❏ Merkel's disc → slow adapting, light/discriminative touch
❏ Pacinian corpuscles → opposite of Meissner’s; high frequency
vibration and deep pressure
73. Majority of the carbon dioxide in the blood is transported as?
❏ Dissolved carbon dioxide → 11% in plasma, 4% in blood
❏ Carbaminohemoglobin → 21%
❏ Carboxyhemoglobin
❏ Bicarbonate → 64%
cjhernandez/djhorneja/ccorpuz
74. If both the afferent and efferent arteriole dilates, what will happen to the GFR and RBF?
❏ GFR will increase, No change in RBF
❏ GFR will increase, RBF will increase → more blood enters glomerulus →
inc hydrostatic pressure → inc GFR
❏ GFR will increase, RBF will decrease
❏ GFR will decrease, RBF will increase
75. What zone of pulmonary perfusion does the lung bases belong if the patient is in an
upright position?
❏ Zone 1 → theoretically does not exist, present only if ventilation is altered
in case of intubation and hooked to mechanical ventilator
❏ Zone 2 → apex and mid-lung
❏ Zone 3
❏ Mixed Zone 2 and 3
76. Glucose is absorbed in the small intestine through what mechanism?
❏ Secondary active transport → SGLT1
❏ Primary active transport
❏ Simple diffusion
❏ Facilitated diffusion
77. The phosphagen system used almost entirely in which of the following sports?
❏ Weight lifting → Type 2 muscle fibers are used in sudden burst of energy
❏ 200 meter dash → Type 1 and 2 muscle fiber combination; 100m dash → Type 2
❏ 100 meter swim → type 1 muscle
❏ Cross country skiing → type 1 muscle
78. Destruction of the parietal cells seen in chronic gastritis is accompanied by a decreased
production of hydrochloric acid and which of the following?
❏ Gastrin
❏ Pepsin
❏ Bicarbonate
❏ Intrinsic factor
79. Which of the following is NOT true regarding the action of cortisol?
❏ Stabilizes lysosomal membranes
❏ Increased permeability of capillaries → decreases permeability
❏ Decreased migration of wbcs in the inflamed area
❏ Decreases lymphocyte reproduction
80. Which of the following factors increases gastric emptying?
❏ Distention of the duodenum
❏ Presence of irritation in the duodenal mucosa
cjhernandez/djhorneja/ccorpuz
❏ Osmolality of the chyme → hyperosmolality means no digestion yet hence no
gastric emptying yet
❏ None of the above
81. This is the first step in the process of urine formation:
❏ Tubular filtration
❏ Tubular secretion → 3rd
❏ Glomerular filtration
❏ Tubular reabsorption → 2nd
*Amount excreted in urine = amount filtered + amount reabsorbed
82. A patient who took an overdose of aspirin started having tinnitus, headache, dizziness or
vertigo, confusion, tachypnea, metabolic acidosis, nausea, vomiting, dehydration/thirst and
hypoglycemia. This physiologic processes is expected to occur:
❏ Shift in hemoglobin dissociation curve to the right → due to acidosis or low pH
and tachypnea low CO2
❏ Decreased baroreceptor stimulation
❏ Increased intracellular influx of potassium → in metabolic acidosis there must be
EFFLUX of potassium
❏ Decreased renal reabsorption of bicarbonate → increased reabsorption since
patient is acidotic
83. This is TRUE regarding the sympathetic nervous system:
❏ It is responsible for accomodation of vision to near objects →
Parasympathetic
❏ It is responsible for relaxation of urinary and gastrointestinal sphincters →
Parasympathetic
❏ It has short preganglionic fibers and long post ganglionic fibers
❏ It only utilizes norepinephrine as neurotransmitter → also utilizes Ach
84. This is NOT TRUE of skeletal muscle contraction:
❏ Action potential travels down depolarizing the terminal axonal promoting
release of acetylcholine
cjhernandez/djhorneja/ccorpuz
❏ During membrane depolarization, the inside of the cell becomes positively
charged
❏ Acetylcholine diffuses across the synaptic cleft stimulating muscarinic
acetylcholine receptors → ACh stimulates nicotinic acetylcholine receptors
❏ Acetylcholinesterase leads to break down of acetylcholine terminating the
synaptic transmission
*Fates of Ach: Broken down by acetylcholinesterase, reuptaken or diffuses out of the
post-synaptic terminal
85. A 24 year old female patient is pregnant at 8weeks AOG presents with intractable
vomiting. This potential acid base disturbance can be seen in this patient:
❏ Respiratory acidosis
❏ Compensated metabolic acidosis
❏ Respiratory alkalosis
❏ Metabolic alkalosis → vomiting loses HCL
86. A neonate was jaundiced at the 8th hour of life. His mother is Rh (-), baby is Rh(+). Total
bilirubin levels were found to be 25mg/dL at the 2nd day of life. Patient was deemed at risk
for a CNS complication. The most likely to affected part of the brain is:
❏ Basal ganglia → predilection in case of kernicterus
❏ Cerebellum
❏ Medulla
❏ Cerebral cortex
87. A 30 year old patient was involved in a motor vehicular accident. He has sustained
multiple fractures over the lower extremities and multiple rib fractures as well. His initial BP
was 80/50 HR 138, he was assessed to have hypovolemic shock. This parameter is known to
increase cardiac output:
❏ Increased venous capacitance → there will be pooling of blood →
decrease CO
❏ Decreased heart rate → decreased CO (CO = SV x HR)
❏ Decreased afterload
❏ Decreased inotropy → decreased contraction → decrease CO
88. This conditions increases gastric emptying time:
❏ Decreased Cholecystokinin Secretion → must be increased
❏ Decreased Intake of Fatty Foods → will decrease gastric emptying time
❏ Increased Motilin Secretion
❏ Presence of Fat and Peptides in the duodenum
89. Assuming equal amounts of glucose were provided, this route produces a greater
increase in pancreatic insulin secretion:
❏ Oral glucose intake → effect of incretin (GLP1) by enteroendocrine cells,
higher secretion of insulin
❏ Rapid IV glucose infusion → bypasses gut, no stimulation of incretin secretion
❏ Sugar tablets underneath the tongue → bypasses gut as well
cjhernandez/djhorneja/ccorpuz
❏ A and B produces equal rise in insulin
90. A 30 year old female patient presents with hypertension, supraclavicular fat pad,
multiple abdominal striae, obesity, red flushed skin, history of visual disturbances
specifically involving temporal visual fields of both eyes. This laboratory finding is expected:
❏ Decreased serum free cortisol → increased
❏ Suppression of cortisol secretion following low dose dexamethasone
administration → not suppressed after low dose dexamethasone
administration
❏ Decreased urine free cortisol → increased
❏ Increased ACTH
*This is a case of Cushing’s disease (not syndrome) due to visual disturbance; there
is presence of tumor producing high ACTH
91. A pharmaceutical group developed a drug that can inhibit phospholamban. The effect on
cardiac myocytes is:
❏ There would be cardiac muscle relaxation → Calcium gets sequestered
decreasing cytosolic Calcium leading to cardiac muscle relaxation
❏ SERCA would be uninhibited and there would be more uptake of calcium ions
into the sarcoplasmic reticulum
❏ It would cause a decrease in activation of the calcium dependent
phosphatases implicated in cardiac hypertrophy
❏ All of the above
*Phospholamban regulates the action of SERCA, once it binds to SERCA it inhibits it
hence more Calcium left in cytosol → i ncreasing cardiac contractility
92. Patient suffered a stroke and developed apraxia. Which area/s of the brain when
damaged could potentially cause apraxia?
❏ Posterior parietal lobe
❏ Premotor cortex → somatic motor association area
❏ Primary motor cortex
❏A&B
*Apraxia is a motor defect particularly in motor planning
cjhernandez/djhorneja/ccorpuz
93. The representational hemisphere:
❏ Is the right cerebral hemisphere in most right-handed individuals
❏ Is the left cerebral hemisphere in most left-handed individuals → categorical
hemisphere
❏ Includes the part of the brain concerned with language functions
❏ Is the site of lesions in most patients with aphasia
*Representational hemisphere → involved in creativity and arts (Right side)
*Categorical hemisphere → involved in motor and speech (Left side)
94. Lymphocytes:
❏ All originate from the bone marrow after birth → from lymphoid organs after
birth
❏ Are unaffected by hormones → affected
❏ Are part of the innate immune response → major role in adaptive immune
response
❏ Are part of the body's defense against cancer → clonal deletion
95. Red blood cells are considered powerful acid-base buffers. When carbonic acid
dissociates into hydrogen and bicarbonate ions, most of the hydrogen ions bind to
hemoglobin and the bicarbonate ions diffuse out of the red blood cells into the plasma in
exchange for what ion?
❏ Sodium
❏ Phosphate
❏ Potassium
❏ Chloride → CO2 converted to bicarbonate and moves out of cell in exchange
for chloride ions
96. Gastric pressures seldom rise above the levels that breach the lower esophageal
sphincter, even when the stomach is filled with a meal, this is due to
❏ Peristalsis
❏ Gastroileal reflex
❏ Segmentation
❏ Receptive relaxation → after a meal, no reflux or regurgitation even if
stomach filled with food due to this mechanism
97. The pressure in a capillary in skeletal muscle is 35 mmHg at the arteriolar end and 14
mmHg at the venular end. The interstitial pressure is 0 mmHg. The colloid osmotic pressure
is 25 mmHg in the capillary and 1 mmHg in the interstitium. The net force producing fluid
movement across the capillary wall at its arteriolar end is:
❏ 3 mmHg out of the capillary
❏ 3 mmHg into the capillary
❏ 10 mmHg out of the capillary
❏ 11 mmHg out of the capillary → J = (35+1) - (25 + 0)
cjhernandez/djhorneja/ccorpuz
98. This hormone acts on its target tissue/s using inositol 1,4,5-triphosphate (IP3) as a
second messenger.
❏ Insulin
❏ Atrial natriuretic peptide
❏ Parathyroid hormone
❏ Thyroid releasing hormone
99. This would cause a shift to the right in the oxygen-hemoglobin dissociation curve?
❏ Increase in pH
❏ Decrease in CO2
❏ Decrease in temperature
❏ Exercise
cjhernandez/djhorneja/ccorpuz
cjhernandez/djhorneja/ccorpuz
SURGERY MOCK BOARD EXAM
UST MBR 2020
cjhernandez/djhorneja/ccorpuz
4. Most common osteoporotic fracture occurs in the
❏ Radius
❏ Hip
❏ Shoulder
❏ Spine
6. A 1-month old baby presents with persistent bilious vomiting for the past 2 days with
minimal abdominal distention. There was a passage of meconium after birth. A possible
surgical condition is:
❏ Hirschsprung's disease → no passage of meconium; difficulty/delayed passage
of meconium
❏ Hypertrophic pyloric stenosis → may be excluded because this obstruction is
higher up hence no bile yet in this case
❏ Malrotation → Tx: Ladd’s procedure - counterclockwise detorsion of the
bowel, surgical division of Ladd's bands, widening of the small intestine's
mesentery, performing an appendectomy, and reorientation of the small
bowel on the right and the cecum and colon on the left (the appendectomy is
performed so as not to be confused by atypical presentation of appendicitis at
a later date).
❏ Incarcerated inguinal hernia → prominent bulge in PE with a little distention of
abdomen since involved segments are distal
*Minimal abdominal distention - obstruction is high
cjhernandez/djhorneja/ccorpuz
7. A 2-month old baby presents with an enlarged right scrotum without inguinal bulge. The
condition is most compatible with:
❏ Inguinal hernia → indirect in children; presence of inguinal bulge-
❏ Hydrocoele → 2 types: Noncommunicating, no connection with abdomen;
Communicating, with a communication to the abdominal wall hence, pooling
of fluid to most dependent portion
❏ Cryptorchidism → undescended testes, no inguinal bulge; PE: no testicle
palpated in scrotum
❏ Testicular torsion → swelling with pain, erythema and color changes
8. A 1-day old baby has no anal opening. Meconium was noted coming out from the urethral
meatus. Which of the statements is true?
❏ He has high imperforate anus → NOT, more common is rectourethral fistula
❏ Prognosis is excellent with regard to continence → not so good prognosis
❏ Cross-table lateral x-ray would probably reveal the rectal gas beyond the
pubo-coccygeal line
❏ Anoplasty without diverting colostomy is the procedure of choice → complex
reconstruction we do diversion to allow healing and impair infection
*Meconium exiting urethra implies a fistula
*Most common in males: Low malformation - Rectourethral bulbar fistula; seen on
X-ray with colonic gas below pubococcygeal line
cjhernandez/djhorneja/ccorpuz
9. A 1-day old baby was born with a 4-cm. abdominal wall defect to the right of the umbilical
cord. Small intestines eviscerate through the defect. Which is true among the statements?
❏ Prognosis is poor because of multiple chromosomal and cardiovascular
abnormalities
❏ The baby can be fed early after closure of the defect → cannot be done
because intestinal atresia is seen with gastroschisis
❏ Intestinal atresia is rarely seen
❏ Fluid resuscitation maybe 2-3 times the normal → higher fluid requirement to
prevent fluid from abdomen from evaporating
cjhernandez/djhorneja/ccorpuz
12. Myasthenia gravis patients without concomitant thymoma will experience improvement
or resolution of muscle weakness after thymectomy in:
❏ 95% → good response on patients with Myasthenia gravis to thymectomy
whether or not there are symptoms; If surgery cannot be done, Radiotherapy
and chemotherapy must be given to prevent local invasion of thymoma
❏ 75%
❏ 50%
❏ 25%
13. A 50 year-old male with a significant smoking history had hemoptysis. Chest xray showed
a 2cm hilar mass at his lung. Possible consideration:
❏ Adenocarcinoma
❏ Bronchoalveolar Ca
❏ Squamous Cell Ca → most common lung CA; centrally located
❏ Large Cell Ca
14. A three year-old female was brought to the ER due to easy fatigability and cyanosis on
physical exertion. On history, patient was noted to be assuming the squatting position
whenever she gets tired from playing. On PE, there is clubbing of the fingers and toes. A
harsh systolic ejection murmur was heard over the pulmonic area and left sternal border.
What is the most likely diagnosis?
❏ Patent Ductus Arteriosus → acyanotic; connection between pulmonary artery
and aorta, classic machinery like murmur
❏ Tetralogy of Fallot → 4 components: Pulmonic stenosis, Ventricular Septal
Defect, Overriding Aorta, Right Ventricular Hypertrophy; systolic murmur at
pulmonic area due to pulmonic stenosis
❏ Aortic Stenosis → systolic ejection murmur
❏ Transposition of Great Arteries → LVOT obstruction; mimics aortic stenosis in
the murmur
cjhernandez/djhorneja/ccorpuz
*Squatting position → compensatory mechanism wherein there is inc peripheral
resistance→ dec magnitude of left to right shunt across VSD
*”Tet spells” → cyanosis when crying or feeding
15. What characteristic finding will you see in chest x-ray of the patient above?
❏ Coeur en sabot → boot-shaped heart because of RVH
❏ Egg-shaped heart → seen in TGA; “Egg on a string”- due to enlarged heart,
string - stress induced thymic atrophy
❏ Water bottle sign → seen in pericardial effusion
❏ Meniscus sign → sign of pleural effusion
cjhernandez/djhorneja/ccorpuz
*Split thickness graft → acquire small portion of dermis only, no hair growth
and no sweat
17. Z plasty is one of the most commonly used flap has the following characteristic/s:
❏ Change the direction and appearance of the resulting scar
❏ An example of an axial flap → Z plasty is a transposition flap that rearranges
direction of wound to make it visually pleasing and to avoid contractures
❏ A form of an advancement flap
❏ A vascularized flap →z plasty is not constructed with vascular pedicle
21. A 24 yo construction worker fell from the 3rd floor and sustained multiple injuries. He
was rushed to the emergency room where initial neurologic evaluation was done. The
patient has abnormal flexion and moans to painful stimuli. There is also no eye opening
despite doing sternal rub. What is the patient’s Glasgow Coma Score?
❏5
❏6
❏7
❏8
cjhernandez/djhorneja/ccorpuz
23. This type of CNS tumor originates from the roof of the 4th ventricle and is considered the
most common malignant pediatric brain tumor.
❏ Glioblastoma
❏ Ependymoma
❏ Medulloblastoma
❏ Pilocytic Astrocytoma
25. The sources of most cerebral metastases are (in decreasing frequency):
❏ breast, lung, GI tract, kidney, melanoma
❏ lung, breast, kidney, GI tract, melanoma
❏ melanoma, lung, breast, kidney, GI tract
❏ melanoma, lung, breast, GI tract, kidney
26. An 18 year old male is brought to the ER because of a gunshot wound to the neck. He is
tachypneic, stuporous and with faint radial pulses. There is profuse bleeding from the bullet
entry site and bubbling of air when he breathes. There is decreased breath sounds on the
left hemithorax. What will be your priority in management?
❏ Apply pressure on the wound with a gauze
❏ Insert a chest tube on the left
❏ Perform endotracheal intubation → ABCDE always in trauma
❏ Request for a STAT chest x-ray
27. A 25 year old female was a victim of hit-and-run by a speeding van. At the ER she was
awake but tachypneic. BP 110/70, PR 100. PE: bruises on the right chest area, equal breath
sounds; chest movements appear to be paradoxical. What is your most likely diagnosis?
❏ Flail chest → 3 or more consecutive fractures in 2 places, there will be
separate segment from chest wall which will alter breathing mechanics→
paradoxical
❏ Massive air leak → air leaking out of respiratory system
❏ Open pneumothorax → with a connection to the environment; sealed with a
vaselinized gauze but not closed fully since it would just cause pneumothorax;
closed but still allowing some air to exit
❏ Tension pneumothorax → shifting of mediastinal contests causing circulatory
compromise
cjhernandez/djhorneja/ccorpuz
28. A 15 year old male was brought to the ER because of a lacerated wound he sustained on
his right inner thigh after falling in an open manhole. BP 90/60, PR 110, RR 20. PE: 5 cm
gaping laceration on the medial aspect of the right thigh with gushing active bleeding. What
is the best management option?
❏ Apply firm pressure on the wound using a sterile gauze → best initial
management for bleeding
❏ Call interventional radiology for possible embolization of the vessel
❏ Explore the wound and try to isolate and clamp the injured blood vessel
❏ Place a tourniquet on the thigh proximal to the lacerated wound
29. Which among these is part of the secondary survey?
❏ Determination of Glasgow Coma Scale (GCS) → primary survey
❏ Establish a secure peripheral IV access → Circulatory part in primary survey
❏ Head to toe physical examination
❏ Insertion of a chest tube for pneumothorax → breathing management; primary
survey
30. What is the initial management of a 50 y/o Nun presenting with a 2x2 cm, firm but
movable mass over her left breast. She has no family history of any cancer?
❏ Breast MRI → can be requested if mammogram is equivocal
❏ Mammography → initial imaging of choice over 40yrs; old women has higher
estrogen, higher risk for developing cancer
❏ CNB
❏ Excision biopsy → confirmatory
31. A 32 y/o pregnant patient at 7th week AOG, was diagnosed with IDCA stage 1, what is
the most appropriate form of treatment?
❏ Neoadjuvant chemotherapy followed by mastectomy → not recommended
during 1st trimester
❏ Mastectomy with Axillary LN staging and or treatment → dissect and/or
treatment; Surgery is most appropriate at this stage of pregnancy (1st tri)
❏ Offer breast conservation treatment since patient is young → NOT
recommended due to possible progression of cancer during pregnancy resulting
to poorer outcome
❏ Just observe and start treatment after delivery → chance of progression of
cancer
*Radiation can only be given after delivery
cjhernandez/djhorneja/ccorpuz
32. What is the most abundant component of skin?
❏ Collagen
❏ Elastin
❏ Keratin
❏ Sebum
33. Which of the following is the correct sequential order of the phases of wound healing?
❏ Hemostasis, Inflammation, Repair, Remodeling
❏ Inflammation, Hemostasis, Proliferation, Remodeling
❏ Inflammation, Proliferation, Remodeling, Maturation
❏ Remodeling, Inflammation, Hemostasis, Repair
34. How would you manage an open wound seen 12 hours after injury?
❏ Debridement followed by wound suturing → ideally done when there is a
time interval, clean area first before closing wound
❏ Healing by granulation
❏ Healing by secondary intention
❏ Wound suturing alone
35. Fibroblasts in a healing wound are derived from:
❏ Endothelium
❏ Epithelium
❏ Local mesenchyme → theory: comes from the blood from the wound; but
later found to be derived from local mesenchyme
❏ Vascular fibrosis
36. A 50/M sought consult because of blood-tinged stools and weight loss. A colonoscopy
was requested and revealed a fungating mass at 40cm from anal verge. Based on the
measurement by colonoscopy, the lesion is located in:
❏ descending colon
❏ rectum
❏ distal transverse colon
❏ sigmoid colon → 17-57 cm from anal verge
41. The following are risk factors for Esophageal carcinoma, EXCEPT:
❏ Nitroso compounds
❏ Zinc & molybdenum deficiency
❏ Smoking
❏ Mucosal erosions caused by Hiatal Hernia
42. The junction of cystic duct and common bile duct, confluence of D2 & D3 and junction of
body and neck of pancreas are the borders of:
❏ Triangle of Calot→ “Hepatocystic triangle”; Superior: inferior edge of liver;
Medial: common hepatic duct, Lateral: Cystic duct; Content: cystic artery
❏ Pasaro’s Triangle → Gastrinoma triangle; most common area where
gastrinomas are seen
❏ Hesselbach's Triangle → Inguinal Triangle; Medial: Lateral border of Rectus
abdominis; I: Inguinal Ligament; L: Inferior Epigastric Vessels; indicate
presence of direct inguinal hernia
cjhernandez/djhorneja/ccorpuz
❏ Triangle of Doom → L: Gonadal vessels; M: Vas deferens; I: Peritoneal edge
cjhernandez/djhorneja/ccorpuz
❏ CEA → can also be used in some thyroid cancers
47. A patient presenting with recurrent fractures and kidney stones due to primary
hyperparathyropidism most likely has:
❏ Parathyroid cancer
❏ Adenoma → still the most common cause of primary hyperparathyroidism
❏ Parathyroid hyperplasia
❏ MEN syndrome → Multiple Endocrine Neoplasia
48. A patient after thyroid surgery was noted to have hoarse voice. Which nerve structure
could have been affected?
❏ Hypoglossal
❏ Recurrent laryngeal → close proximity to inferior thyroid artery
❏ Facial
❏ Glossopharyngeal
*External branch of superior laryngeal nerve → another nerve can be injured during
thyroidectomy, supplies the cricothyroid muscle and injury causes alteration to the
pitch of voice, unable to reach high notes; close proximity to superior thyroid artery
49. After thyroidectomy for 5 cm papillary cancer, which adjuvant treatment should be
given?
❏ Radioactive iodine → for well diff Ca, indicated for: 1) large tumors >4cm 2) LN
involvement; but LND must be done if with mets 3) aggressive histologic type
4) invasion to other structures 5) gross tumor left after surgery
❏ Targeted therapy
❏ Radiation
❏ Chemotherapy
cjhernandez/djhorneja/ccorpuz
50. Which of the following features most likely correlates with survival and prognosis of
patients with head and neck cancer?
❏ Site of primary cancer
❏ Size of tumor
❏ Lymph node metastasis → regional metastasis will correlate with prognosis
❏ Family history
51. A 60 year old male complains of a 2 cm well defined moveable mass of three months
duration in the right submandibular area. There were no other accompanying symptoms.
Which of the following is the most useful and accurate diagnostic tool/s to help in the
management?
❏ FNAC → can determine if malignant or not; most accurate
❏ CT scan
❏ Ultrasound
❏ Physical examination → physical appearance only
52. A 55 year-old male with a left submandibular mass, was determined to be metastatic
cancer. Which of the following is the LEAST LIKELY source?
❏ Floor of the mouth → Level 2
❏ Tongue → Level 2
❏ Thyroid → Level 3,4,5,6
❏ Submandibular gland → at Level 1
53. A 45 year-old female underwent right thyroid lobectomy for what seems to be colloid
goiter. However, final biopsy showed papillary cancer. Which findings would make you
advise her that a completion thyroidectomy is needed?
❏ 2 cm papillary cancer → small size
❏ Tall cell variant
❏ Papillary cancer does not go beyond the capsule → better since confined to
gland
❏ 0.3 cm lymph node with fatty hilum → benign especially presence of fatty
hilum; sign of malignancy if there is absence of fatty hilum
cjhernandez/djhorneja/ccorpuz
54. A 25 year-old male presents with a 5 year history of a slow-growing right preauricular
mass. This is most probably a:
❏ Lymph node
❏ Parotid tumor → usually benign
❏ Abscess → inflammation, erythema, pain, fever
❏ Soft tissue cancer → aggressive and shorter duration of history
55. A 45 year-old female, underwent throid surgery. A few minutes at the recovery room,
the dressing was soaked with blood. She complained of difficulty of breathing. What should
be done?
❏ Check the airway
❏ Remove the dressing and open the wound → decompress if there is
obstruction; control bleeders and do repair, notify OR
❏ Intubate
❏ Refer to the resident
56. Injury to this nerve would manifest altered sensation to the upper anterior thigh
❏ Ilioinguinal nerve
❏ Ilio-hypogastric nerve
❏ Genital branch of the Genitofemoral nerve
❏ Femoral branch of the Genitofemoral nerve → sensory innervation to upper
anterior thigh
57. 44 y/o male post Open Inguinal Hernia repair with Mesh placement 6months PTC.
Consults you for persistent pain over the post op site. PE: Well healed wound with no signs
of infection. No abdominal or inguino-scrotal masses noted. Initial plan of management:
❏ Reassure patient that it is normal post-op pain and will resolve spontaneously
❏ Conservative management with bed rest and a course of NSAIDs
❏ Consider inguinal hernia recurrence and plan for laparoscopic repair
❏ Surgical intervention to remove mesh or offending tacks
58. A Hernia located in the superior lumbar triangle is known as:
❏ Grynfeltt-Lesshaft hernia
cjhernandez/djhorneja/ccorpuz
❏ Littre’s hernia → Intestinal Meckel’s diverticulum within the hernia
❏ Amyand hernia → rare form of an inguinal hernia in which the vermiform
appendix is located within the hernial sac
❏ Petit hernia → inferior lumbar triangle
59. Risk Factors of Inguinal Hernia:
❏ Connective tissue disorders
❏ Cigarette Smoking
❏ Prematurity
❏ All of the Above
60. The following structures are contained in the Triangle of pain, EXCEPT:
❏ Ilioinguinal nerve → present in inguinal canal not in triangle of pain
❏ Lateral femoral cutaneous nerve
❏ Femoral branch of the genitofemoral nerve
❏ Femoral Nerves
61. Which tissue-to-tissue repair addresses both inguinal and femoral ring defects?
❏ Bassini Repair → Tissue repair; suturing the transversalis fascia and the
conjoined tendon to the inguinal ligament behind the spermatic cord with
monofilament nonabsorbable suture
❏ McVay Repair → similar with Bassini but Cooper’s ligament is used instead of
inguinal ligament
❏ Shouldice Repair → four-layer inguinal hernia repair; the transversalis fascia is
incised from the internal ring laterally to the pubic tubercle medially, and upper
and lower flaps are created
❏ Lichtenstein Repair → Mesh repair
62. A 49 y/o female, hypertensive sought consult for a 4cm bulging umbilical mass. She is
otherwise asymptomatic. PE: 4cm soft umbilical mass, nontender, no skin changes. Plan of
Management:
❏ Watchful Waiting
❏ Emergency Surgery for repair of Umbilical Hernia
❏ Elective Surgery using Tissue Repair
❏ Elective Surgery w Repair of umbilical Hernia with Mesh Application → mesh
needed esp for more than 2cm mass
cjhernandez/djhorneja/ccorpuz
63. A 98 y/o male, sought consult for a 3x4cm left inguinal mass noticed 2 weeks PTC. No
change in urinary or bowel habits. PMHx: HPN, DM, COPD, Previous Stroke (3years PTC) On
PE: BP 139/80 PR:80 RR:21 Temp 36.7. Physical Exam: (+) 3x4 nontender, nonerythematous
inguino-scrotal mass on the left. Abdominal exam was unremarkable. Plan of management:
❏ Prepare for elective Inguinal Hernia Repair with mesh application
❏ Watchful Waiting → surgery not advised due to old age and comorbidities
❏ Immediate/Emergency surgery for Inguinal Hernia due to comorbidities
❏ Prepare for elective Inguinal Hernia (Tissue-to-tissue) Repair
64. A 3 y/o female presents with a 3cm bulging umbilical mass. Patient is active with
occasional abdominal pain. Mother denies vomiting or change in bowel habits. Ultrasound
confirms the presence of an umbilical hernia. What is the appropriate management?
❏ Prepare patient for emergency surgery
❏ Elective repair of umbilical hernia within 1 year of diagnosis
❏ Elective repair of umbilical hernia in 2 years
❏ No Surgical Management
*since guidelines say surgery is indicated until 3 years old
65. Which is NOT part of the boundaries of the inguinal canal?
❏ Anterior - External Oblique Aponeurosis
❏ Inferior - Inguinal Ligament
❏ Superior - Internal Oblique Muscle
❏ Posterior - Rectus sheath → must be Transversus
66. A 50 year old man presents with microscopic hematuria. Which will dictate that he needs
aggressive diagnostic procedures?
❏ Microscopic hematuria with exercise
❏ Microscopic hematuria after a bout of viral upper respiratory tract
❏ Microscopic hematuria with dysuria of one day duration
cjhernandez/djhorneja/ccorpuz
❏ Microscopic hematuria with proteinuria → may point to a glomerular disease
67. Which lower urinary tract symptoms tell you of bladder outlet obstruction?
❏ Urgency and frequency → Urgency is immediate unstoppable urge of
urinating; Frequency is increase in in number of times of urinating
❏ Hesitancy and feeling of incomplete voiding
❏ Urgency incontinence → no control of urinary functions
❏ Nocturia → waking up more than once to go to the bathroom to void
68. A 21 year old promiscuous male executive comes with a slight dysuria and yellowish
discharge. He was given cefixime 200 mg capsule 2 capsules one take. What will you do to
make a diagnosis?
❏ Urethral swab and gram staining → invasive; urine tests still the best diagnostic
test
❏ Urinalysis of first voided urine
❏ Urine nucleic acid amplification test of first voided urine for GC and CT →
Culture is gold standard
❏ Give him ceftriaxone 250 mg IM After negative skin test
69. A 40 year old asymptomatic woman is on her 7 week of gestation. She presents on
routine urinalysis showing 15-20 wbc , 2 bacteria/hpf. You should:
❏ Give beta lactamase inhibitors → safe for pregnancy
❏ Give her nitrofurantoin → not good in first trimester; increase malformation
❏ Assure her she is alright
❏ Give her sulfonamides → Pregnancy Category D, associated with cleft palate
and bone abnormalities
70. A 60 year old man presents with LUTS- hesitancy, straining and urgency. DRE- showed a
grade 1 prostate. Ultrasound KUB is normal. You will give:
❏ Alpha adrenergic blocker →Prazosin, Tamsulosin → relaxes muscle in prostate
and bladder
❏ 5-alpha reductase inhibitor → Finasteride, not required for these patients
❏ Recommended TURP → not recommended since too small yet
❏ All of the above
71. Inhibition of complexation and aggression of crystals above the supersaturation level to
form urolithiasis is best exemplified by this stone inhibitor:
❏ Sodium
❏ Calcium
❏ Potassium citrate
❏ Sodium citrate
72. A 20 year old woman was in a vehicular accident. She is stable. Ct urogram showed a 1
cm laceration of the left renal parenchyma. With a 30 cc perirenal hematoma. You should:
❏ Go for renal exploration
❏ Go for selective renal artery embolization
cjhernandez/djhorneja/ccorpuz
❏ Just monitor her → lesion is small; blood still contained within renal capsule
❏ Transfuse immediately and prepare for surgery electively
73. BPH stands for:
❏ Benign prostatic hypertrophy
❏ Benign prostatic hyperplasia
❏ Beginning prostatic hyperplasia
❏ Beginning prostatic hypertrophy
74. In which scenario will a possible skull metastasis from a breast cancer is most likely?
❏ spread of cancer to the supraclavicular nodes → local disease and does not
equate to distant mets
❏ grossly ulcerating tumor involving most of the overlying skin → local
disease and does not equate to distant mets
❏ involvement of the Batson’s plexus → aka vertebral venous plexus, most
common route of metastasis from lungs, breast, etc.
❏ presence of lung metastasis involving the apex of the lungs →
hematogenous route, no Batson’s plexus involvement
75. True statement with regards to breast cancer in males;
❏ Most breast cancer in males are DCIS → Invasive ductal carcinoma
❏ A similarly stage cancer has a worse prognosis in males compared to females →
same prognosis; difference is that in males, there is possibility of delayed
diagnosis
❏ Excision with negative margin will suffice, since males do not have much breast
tissue → same principles with male and female; MRM and LND must be done as
well
❏ Tamoxifen maybe given as adjuvant treatment even if they not have ovaries →
80% are hormone receptor sensitive hence Tamoxifen can be given
76. A 17 y/o college student came in with a chief complaint of premenstrual breast pain. On
PE you noted the breast to be nodular with a dominant mass over the UOQ of the right
breast. The mass is soft and well circumscribed measuring 5 cm. What is the most
appropriate course of action?
❏ Request for mammography → not indicated
❏ Observe for it will most likely regress → >2cm mass will not likely regress
❏ Do an initial FNAB, aspirate the fluid if cystic →ideally, request imaging first
(if included in choices) then if cystic then aspirate
❏ Excision biopsy of the dominant mass
*given the PE, this is most likely a breast cyst and aspirating will definitely remove
mass with low chance of recurrence
77. A 25-year-old woman arrives in the ER following an automobile accident. She is acutely
dyspneic with a respiratory rate of 60 breaths per minute. Breath sounds are markedly
diminished on the right side. Which of the following is the best NEXT step in the
management of this patient?
❏ Take a Chest X-ray
❏ Intubate the patient
cjhernandez/djhorneja/ccorpuz
❏ Administer intravenous fluids
❏ Decompress the right pleural space → most likely patient has pneumothorax
so insert chest tube
78. A 36-year-old man who was hit by a car presents to the ER with hypotension. On
examination, he has tenderness and bruising over his right lateral chest below the nipple. An
ultrasound examination is performed and reveals free fluid in the abdomen. What is the
most likely organ to have been injured in this patient?
❏ Liver→ largest solid abdominal organ
❏ Kidney
❏ Intestine
❏ Pancreas
*Solid organs are most commonly affected in blunt trauma
79. A 48-year-old man is in a high-speed motorcycle collision and presents with an obvious
pelvic fracture. On examination, he has a scrotal hematoma and blood at his urethral
meatus. Which of the following is the most appropriate next step in his management?
❏ Placement of a Foley catheter → not done especially if there is urethral injury;
evaluate first via retrograde urethrogram
❏ Cystoscopy
❏ CT of the pelvis → visualizes fracture or contents of pelvis but urethra cannot
be assessed properly
❏ Retrograde urethrogram → for evaluation of urethral injury
*probably there is an injury in the lower urinary tract/urethra
80. A 21-year-old female sustains a stab wound to the middle of the chest. Upon arrival to
the ER she is anxious, with blood pressure of 85/56 mm Hg, respiratory rate of 32, and pulse
rate of 125. What is her estimated blood loss in milliliter?
❏ <750 → Class I
❏ 750-1500 → Class II, only orthostatic hypotension
❏ 1500-2000 → Hypotension, tachycardic, anxious (class III)
❏ >2000
81. A 60-year-old diabetic man undergoes incision and drainage of an infected boil on his
back. The wound is left open and packed daily. Week by week, the wound grows smaller and
eventually heals. Which of the following terms describes the method of wound closure by
the patient?
❏ Primary intention → close defect immediately
cjhernandez/djhorneja/ccorpuz
❏ Secondary intention → usually for sacral ulcers, infected wounds; allow
granulation and epithelialization
❏ Tertiary intention → same with delayed primary closure, allow granulation and
healing but eventually sutured
❏ Delayed primary closure → tertiary intention
82. A 43-year-old man presents with a painless ulceration over the left medial malleolus
with surrounding brawny induration. He has no other co-morbidities. What is the likely
diagnosis?
❏ Curling ulcer → gastric ulcers related to burns
❏ Venous stasis ulcer
❏ Cushing ulcer → gastric ulcers related to stroke or intracranial pathology
❏ Marjolins ulcer → ulcer from chronic non healing wounds from Squamous Cell
Carcinoma
83. This cell enters the wound site at about 1 week post injury and bridges the transition
from inflammatory to proliferative phase:
❏ Lymphocytes
❏ Macrophages
❏ Neutrophils
❏ Fibroblasts
84. 10 year old child consulted due to a 2 day history of otalgia in the right ear. He claims
that the pain started after aggressively cleaning his ear with cotton buds. 1 day prior to
consult patient noted discharge and tragal tenderness. Otoscopy revealed a hyperemic,
swollen external auditory canal with minimal discharge and an intact tympanic membrane
on the right. Left ear was unremarkable. What management would be appropriate for the
patient?
❏ Oral Antibiotics
❏ Topical Otic Drops → first line treatment of the case which is Otitis Externa
❏ Observation
❏ CT scan
85. Vertigo associated with a unilateral hearing loss should raise suspicion for:
cjhernandez/djhorneja/ccorpuz
❏ Vestibular Neuritis → Labyrinthitis; vertigo and nystagmus with no assoc
hearing loss
❏ Meniere’s disease → fluid in labyrinth; will also present with fullness of ear
❏ Cerebrovascular disease
❏ BPPV → caused by otoliths that are displaced to semicircular canal
86. 35 year old male came in due to rotatory on and off vertigo lasting seconds. Patient
claims the episodes started a few days ago when he hit his head and attacks are often felt
when he looks to one side quickly. (+) dix-hall pike. What structure is most likely affected in
this case?
❏ Lateral semicircular canal
❏ Endolymphatic sac
❏ Posterior Semicircular canal → involved in BPPV: (+) Dix-Hall pike
❏ Utricle
87. What is/are the causative agent/s for head and neck squamous cell cancer?
❏ Tobacco
❏ Alcohol
❏ Human Papillomavirus
❏ all options are correct
88. A 65 years old male, smoker with 2 cm ulcerating tongue cancer. There are no palpable
nodes. Which lymph node dissection is recommended?
❏ Radical neck dissection
❏ Modified radical neck dissection → LN 1-5 dissected but preserve the non
lymphoid structures; for clinically positive LN
❏ Supraomohyoid neck dissection → for clinically negative LN; usually for oral
cancers
❏ Central node dissection → removes Level 6 or central compartment, indicated
for Medullary Thyroiod cancer
cjhernandez/djhorneja/ccorpuz
*Lateral → laryngeal and pharyngeal cancers; Supraomohyoid → Oral
*Anterolateral → thyroid Ca
cjhernandez/djhorneja/ccorpuz
❏ Propranolol → decrease peripheral conversion of T4 to T3 but will not render
patient euthyroid; also helps with decreasing sympathetic activity
❏ Thyroxine → not useful
90. After a thyroidectomy, the patient complains of hoarseness of voice. Which is affeted?
❏ Superior laryngeal nerve → affects pitch (low-pitched voice)
❏ Hypoglossal nerve
❏ Recurrent laryngeal nerve
❏ Spinal accessory nerve
91. A 35 years old male with 2 cm thyroid nodule underwent biopsy revealing papillary
cancer. There are no palpable nodes, no metastasis. What is the stage?
❏ Stage 1 → for <55y/o, only 2 stages, Stage 1 - (-) metastasis; Stage 2 - (+)
metastasis
❏ Stage 2
❏ Stage 3
❏ Stage 4
92. A 49 year old male was seen at the ER because of difficulty of breathing. He was recently
diagnosed to have glaucoma and was given an eyedrop which he uses a few minutes ago.
What could this eyedrop be?
❏ Latanoprost → PGF antagonist, increase uveoscleral outflow → decrease IOP
❏ Pilocarpine → muscarinic agonist causing pupillary constriction and ciliary
constriction → decreases IOP; causes bronchoconstriction and aggravates
asthma
❏ Timolol → non selective beta blocker which decreases production of aqueous
humor, blockade of B2 leads to bronchoconstriction; Contraindicated in
Asthma
❏ Brimomidine → alpha 2 agonist, decreases production of aqueous humor,
causes vasoconstriction which leads to decrease flow of aqueous humor
93. 65 year old female, known hypertensive woke up with non-painful red eye on the left
and 20/20 vision in both eyes on examination. No discharge was present. What is your
diagnosis?
❏ Acute angle closure glaucoma → painful red eye
cjhernandez/djhorneja/ccorpuz
❏ Subconjunctival hemorrhage → bleeding from scleral blood vessels;
sometimes spontaneously; self-limiting, no pain, no visual impairment
❏ Allergic conjunctivitis → red eye with tearing and itchiness
❏ Acute uveitis → inflammation of uvea, uvea is composed of iris, ciliary body
and choroid; (+) pain
94. Mrs. Bautista noted that her 2 year old boy developed exotropia of the right eye 6
months prior to consult, he was observed to cry loudly when his left eye was covered. A
white spot is seen at the center of his right pupil. Which is a life-endangering condition that
you would need to consider in this case?
❏ Retrolental fibroplasia → “Retinopathy of Prematurity”, disorganized growth of
retinal vessels leading to scarring and retinal detachment, usually seen in
premature babies due to hypoxia
❏ Persistent Hyperplastic primary vitreous → PHPV; rare congenital anomaly
following failure of regression of the primary vitreous which is cloudy which
causes blindness
❏ Retinoblastoma → primary malignant intraocular cancer common in children
❏ Coat’s disease → congenital retinal vasculopathy characterized by retinal
telangiectasia and subsequent exudative retinopathy; non-hereditary, unilateral,
common in males; leaking of blood products in retina causing leukocoria and
blindness
*Exotropia - outward deviation of eye
95. A 50 yr old female who underwent elective cholecystectomy is diagnosed to have
surgical site infection on day 4 post-op. The likely source of the infection:
❏ Airborne bacteria in the operating room
❏ Perforated gloves of the surgeon/ assistants
❏ The patient’s endogenous bacteria
❏ A breach in the sterility of the drapes and gowns
96. A 58 year old man with abdominal pain was diagnosed with a perforated diverticulitis of
the sigmoid colon. He underwent emergency sigmoid resection and drainage of abscess.
Which class of surgical wound does the patient have?
❏I
❏ II
❏ III
❏ IV
cjhernandez/djhorneja/ccorpuz
97. Disadvantage of excision biopsy on a presumed malignant solid tumor:
❏ Tumor dissemination
❏ Bleeding
❏ Inadequate tissue sample
❏ Low sensitivity
98. Which of the following patients is in SIRS (Systemic Inflammatory Response Syndrome)?
❏ Patient A was hit by a bicycle while crossing the street. BP 100/80, HR 85, RR 16,
Temp 38.5C, WBC 10,000/uL
❏ Patient B, person under investigation for SARS COV 2. With a history of recent
travel to Wuhan, China. With cough and dyspnea. BP 120/70, HR 90, RR 16,
Temp 37.5C, WBC 6,000/uL, intubated and hooked to mechanical ventilator.
❏ Patient C, had a mañanita last night consuming 1 bottle of whiskey by himself.
He was brought to the ER due to severe epigastric pain. BP 110/80, HR 100, RR
18, Temp 37C, WBC 11,000/uL no band forms
❏ Patient D, known case of leukemia. He was brought to the hospital for his
scheduled chemotherapy. BP 90/70, HR 80, RR 16, Temp 36.5C, WBC 5,000/uL
with 10% band forms
cjhernandez/djhorneja/ccorpuz
99. The brain will use which of the following as a source of energy during prolonged
starvation?
❏ Ketones → from fatty acid oxidation
❏ Muscle glycogen
❏ Liver glycogen
❏ Renal glucose
100. Healing of this layer in the gastrointestinal wall is essential for quickly achieving a
watertight seal?
❏ Serosa
❏ Mucosa
❏ Submucosa
❏ Muscularis propria
cjhernandez/djhorneja/ccorpuz
PHARMACOLOGY MOCK BOARD EXAM
UST MBR 2020
1. Which of the following drugs would be BEST absorbed in the stomach following the
Henderson-Hasselbach equation?
❏ Drug E : pKa = 2.9
❏ Drug I : pKa = 6.9
❏ Drug A : pKa = 8.2
❏ Drug O : pKa = 10.2
*Most drugs are weak acids or weak bases
*Acid is a proton donor, and base is a proton acceptor
*Acidic substances in acidic pH will be unionized and more absorbable
(Acid-Acid/Base-Base: Unionized (absorbed); Acid-Base/Base-Acid: Ionized (excreted)
2. Which of the drugs, taken in very high dose, would have enhanced excretion if you give
large amount of ascorbic acid?
cjhernandez/djhorneja/ccorpuz
❏ Dry mouth → Atropine effects: dry as a bone (dry mouth), red as a
beet (red skin), mad as a hatter (restlessness), hot as a hare (hot skin),
blind as a bat (mydriasis)
❏ Pupillary constrictions
6. A female patient was given an a1-agonist. You would expect her to develop the following:
❏ Aspirin
❏ Celecoxib → dose related risk including death, MI, stroke heart
failure
❏ Ibuprofen
❏ Naproxen
9. The key factor in the mechanism of action of atypical antipsychotics (e.g. clozapine) is
blockade of these receptors, acting as inverse agonists:
❏ D1
❏ a1
❏ 5-HT2A
❏ D2 and H1
*Typical antipsychotics - more of Dopamine compared to 5-HT2A
*Atypical antipsychotics - 5-HT2A > Dopamine
10. The condition when a maximum response is elicited prior to saturation of all receptors in
the tissue:
cjhernandez/djhorneja/ccorpuz
❏ Partial agonism → less than maximal response even with full receptor
occupancy
❏ Upregulation
❏ Spare receptors
11. Which of the following H1 receptor antagonists would you prescribe because of its
sedative effect thereby reducing awareness of itching?
❏ Euphoria
❏ Vasodilation
❏ Decreased sensory stimulation
❏ Decreased preload and afterload → net effect decreased in myocardial
O2 demand
*Patient has pulmonary edema and most common cause is CHF
13. A 20 year old medical student was diagnosed with major depressive disorder. Which of
the following would you prescribe because of its efficacy and better tolerated adverse
effects?
14. Which drug used in management of seizure disorders is most likely to elevate the plasma
concentration of other drugs administered concomitantly?
❏ Carbamazepine
❏ Clonazepam
❏ Phenobarbital
❏ Valproic acid → CYP inhibitor prevents metabolism of other drugs
leading to increased plasma concentration; acts as a protein binding
displacer
*The rest are inducers
cjhernandez/djhorneja/ccorpuz
15. Which of the following drugs, if given at low dosage, is associated with lower incidence
of colon cancer?
❏ Celecoxib
❏ Ibuprofen
❏ Aspirin → inhibits COX2 enzyme which is upregulated in colon cancer;
normalizes expression of Epidermal growth factor;
❏ Paracetamol
16. A 38 year old teacher with chronic kidney disease has to be given an NSAID. The most
appropriate NSAID to prescribe is a:
❏ COX-2 inhibitor
❏ Non-selective COX inhibitor
❏ Non-acetylated salicylates → weak inhibitors of prostaglandin synthesis
❏ Indomethacin
*NSAIDS → nephrotoxic; higher chance of AKI and higher doses may lead to
irreversible toxic effects; inhibits prostaglandin
7. A 55 year old teacher was diagnosed with breast cancer with bone metastasis. She is
1
suffering from severe persistent pain. Which of the following analgesics given continuously
is the best for her?
❏ Celecoxib → for mild to moderate pain
❏ Ibuprofen → for mild to moderate pain
❏ Morphine → moderate to severe
persistent pain continuously in malignancy
❏ Methotrexate → for moderate to severe
pain
cjhernandez/djhorneja/ccorpuz
18. A 15 year old teen had anaphylactic (severe allergic) reaction to a histamine releasing
substance. The initial (first line) drug that you would give is:
❏ Atropine
❏ Epinephrine → physiologic antagonist of Histamine; reverse effects
immediately
❏ Prednisone
❏ Diphenhydramine
19. A 65 year old with Parkinson’s disease on maintenance Levodopa should be informed
that the drug:
❏ Causes fewer CNS side effects if given together with a drug that inhibits hepatic
dopa decarboxylase → Levodopa has less peripheral toxicity but more CNS
effects such as behavioral effects
❏ Fluctuates in its effectiveness with increasing frequency as treatment
continues
❏ Prevents extrapyramidal adverse effects of antipsychotic drugs → NOT
effective to antagonize EPS
❏ Protects against cancer in patients with melanoma → may activate malignant
melanoma because Levodopa is a precursor of melanin
20. A 20 year old student has migraine headache. The most appropriate drug to give is:
❏ Amitriptyline
❏ Pergolide
❏ Sumatriptan → 5HT1B/1D partial
agonist
❏ Ketanserin
21. Which statement about phenytoin is CORRECT?
22. A 25 year old female was diagnosed with myoclonic seizures and migraine headache. The
most appropriate drug to give is:
❏ Phenytoin
❏ Lamotrigine → alternative
❏ Felbamate
cjhernandez/djhorneja/ccorpuz
❏ Valproate → Drug of choice for myoclonic seizures, broad spectrum
anti-seizure medication, can be used as mood stabilizer for Bipolar disorder
*Alternative: Levetiracetam, Zonisamide, Lamotrigine
23. A 5 year-old child presents with absence seizures. Which drug would be effective in this
child without causing excessive sedation or tolerance?
❏ Phenobarbital
❏ Diazepam
❏ Gabapentin
❏ Ethosuximide → DOC for absence seizures along with Valproic acid
(EVA: Ethosuximide, Valproic Acid for Absence)
24. A 28 year old brilliant female artist presents with signs and symptoms of bipolar mood
disorder with hallucinations and other psychotic features.The most appropriate drug for his
bipolar mood disorder is:
❏ Lorazepam
❏ Fluoxetine
❏ Carbamazepine
❏ Lithium carbonate
25. Blockade of _____ receptor is responsible for inhibition of mast cell release:
❏ H1
❏ H2 → inhibits gastric acid secretion
❏ H4
❏ 5-HT → serotonin
26. A group of teachers presents with signs and symptoms of scombroid poisoning. The most
appropriate treatment is maximal doses of:
❏ 5-HT blocker
❏ H1 blocker → scombroid is a common type of fish poisoning high in
histamine levels from improper storage or cooking
❏ H2 blocker
❏ B2 agonist
❏ Nigrostriatal → EPS
❏ Tuberoinfundibular → hyperprolactinemia
❏ Mesolimbic-mesocortical → Mesolimbic: positive symptoms; Mesocortical:
negative symptoms
❏ Medullary-periventricular
cjhernandez/djhorneja/ccorpuz
28. This antipsychotic has very high D2 receptor blockade but it does not cause
extrapyramidal symptoms (EPS):
29. A 25 year old pregnant lupus patient is receiving furosemide for a long time due to
persistent edema. Which of the following may be an important effect of chronic usage of
this drug?:
30. An 18 year old being treated for acute severe asthma may experience paradoxical
hypoxemia from ventilation perfusion mismatch induced by which drug?
❏ Antimuscarinic
❏ Corticosteroid
❏ B2 agonist
❏ Methylxanthine
31. A 25 year old is taking theophylline (sustained release). A need for higher doses of
theophylline is needed if the patient is also taking this drug:
32. A 65 year old male who has been taking a diuretic for a heart condition is admitted to
the emergency room because of a “fainting spell” at home with no history of trauma. BP is
120/60 when lying down and 60/20 when he sits up. Neurologic examination and ECG are
within normal limits. Which of the following drugs is the most likely cause of his fainting
spell?
cjhernandez/djhorneja/ccorpuz
❏ Furosemide → loop diuretic, “High ceiling diuretic” which increases urinary
excretion vigorously
❏ Hydrochlorothiazide → thiazide diuretic
❏ Spironolactone → Potassium sparing diuretic (Aldosterone antagonist)
33. A 55 year old patient with hypertension and mild heart failure was started on Thiazide
diuretics. Two days after, he developed a viral infection accompanied by anorexia followed a
week later by seizures and unconsciousness. ECG showed sinus tachycardia. Blood is sent for
electrolyte analysis which will probably show:
34. A 56 year old smoker on maintenance medication for chronic obstructive pulmonary
disease has been complaining of tachycardia. Which of the following drugs could have
caused it?
35. A 60 year old dyspneic male is admitted for peripheral edema and heart failure. The
most appropriate diuretic to give is:
❏ Hydrochlorothiazide
❏ Furosemide
❏ Mannitol
❏ Acetazolamide
36. RRD had surgery for aortic dissection and was eventually discharged after 2 weeks. His
blood sugar was controlled but urinalysis revealed +3 proteinuria. Give the antihypertensive
agent of choice:
cjhernandez/djhorneja/ccorpuz
*Proteinuria is a marker for kidney damage
37. Carla, 3 year old girl was diagnosed with lupus nephritis a couple of weeks ago which
necessitated treatment with corticosteroid. The most important adverse effect of steroid
therapy that you would closely monitor for Carla is:
38. A 70 year old patient complains of paresthesias and occasional nausea with one of her
prescribed drugs. She is found to also have hyperchloremic metabolic acidosis. She is
probably taking:
39. RA, a 75 year old hypertensive male was seen for hesitancy and dribbling of urine. He is
currently on amlodipine. PE revealed a BP of 140/90. What additional antihypertensive can
be prescribed to RA?
❏ Sexual dysfunction
❏ Glucosuria
❏ Postural hypotension
❏ Insomnia
41. A 50 year old male was recently added with an antihypertensive drug due to
uncontrolled BP. 2 weeks later, he developed malar rash with joint pains. What is the most
likely agent given to him?
❏ Clonidine
❏ Reserpine
❏ Hydralazine → Hydralazine induced lupus, occurs in 5-10% of
patients
cjhernandez/djhorneja/ccorpuz
❏ Doxazosin
43. AT was referred because of signs of depression while on a fairly high dose of imidazoline
receptor agonist. The internist decided to taper the dose of the antihypertensive while
overlapping it with amlodipine to prevent rebound hypertension. Name the first drug:
❏ Propranolol
❏ Reserpine
❏ Methyldopa
❏ Clonidine → alpha 2 agonist, causes rebound hypertension taper it slowly to
avoid this SE
44. A patient with hypertension and diabetes on multiple drugs developed Stevens Johnson
syndrome. The most likely culprit is:
45. A 45 year old obese male was found to be hypertensive with a BP of 160/100. His blood
exams showed elevated blood sugar and cholesterol levels. Which of the following
antihypertensive drugs that has beneficial metabolic effects is most appropriate for this
patient?
❏ Prazosin → improve Lipoprotein lipase activity and has favorable effects with
plasma cholesterol levels
❏ Hydrochlorothiazide → SE: dose dependent hyperlipidemia and hyperglycemia
may worsen the condition
❏ Metoprolol
❏ Carvedilol
46. A 40 year old female has a history of frequent episodes of renal colic with
calcium-containing renal stones. The most useful diuretic agent for him is:
cjhernandez/djhorneja/ccorpuz
❏ Spironolactone → K+ sparing diuretic; no effect on calcium excretion
47. A patient with long-standing diabetic nephropathy and hyperkalemia and recent-onset
mild heart failure requires a diuretic. Which of the following agents should be avoided?
48. A 50 year old asthmatic, despite high doses of inhaled steroid and long acting
bronchodilator still develops frequent exacerbations. Your next step is to add:
❏ Indacaterol → LABA
❏ Tiotropium → LAMA
❏ Montelukast → LT Antagonist; only an adjunct and may not have an effect good
enough to relieve an exacerbation
❏ Doxofylline → methylxanthine derivative, narrow Therapeutic Index (dose for
efficacy is close to toxicity)
49. The major beneficial effect of combining thiazides and calcium channel blocker is:
50. If a patient's blood pressure is 150/100 and his baseline heart rate is 98/min, and he also
complains of palpitation, which of the following will you add to his treatment regimen?
❏ Ivabradine
❏ Metoprolol → Beta blocker slows heart rate
❏ Amlodipine
❏ Trimetazidine
51. A 65 year old female was brought to the emergency room because of severe constricting
substernal chest pain which started 4 hours earlier. Her electrocardiogram showed 3 mm ST
segment elevation in V1-5. You offered her PCI (percutaneous coronary intervention) but she
declined for financial reasons. The best thing to give to achieve reperfusion and relieve the
chest pain:
❏ Abciximab
❏ Subcutaneous enoxaparin
❏ Alteplase → fibrinolytic, recombinant plasminogen activator; given in
STEMI if PCI cannot be done
❏ ASA and clopidogrel
cjhernandez/djhorneja/ccorpuz
52. A patient with chronic coronary artery disease came in with a 2D Echocardiogram
showing an ejection fraction of 30%. Which of the following must be avoided?
53. Which of the following drugs, when taken with warfarin can result in a pharmacokinetic
interaction that results in decreased prothrombin time?
55. A 40 year old male, post MI patient, consults because of palpitation every time he uses
his salbutamol inhaler. His 2D Echo showed an ejection fraction of 30%. Which of the
following is the most appropriate drug to give?
❏ Metoprolol
❏ Verapamil
❏ Ivabradine → bradycardic drug, Lf inhibitor, decreases hyperpolarization
❏ Nifedipine
56. Which of the following is an advantage of giving an ACEI compared to an ARB when given
to a patient with heart failure?
58. Which of the beta blockers have been shown to improve survival in patients with heart
failure?
❏ Propranolol
❏ Atenolol
❏ Carvedilol → reduce mortality in CHF patients
❏ Sotalol
*Reduces mortality in patients with heart failure: Metoprolol, Bisoprolol, Carvedilol
59. Which of the following anti-lipid drug is the most effective in lowering LDL and found to
stabilize the plaque?
60. A chronic heart failure patient on several drugs consults because of anorexia, nausea and
vomiting. Which of the following medications that he is taking is the most likely cause of
these symptoms?
❏ Captopril
❏ Furosemide
❏ Spironolactone
❏ Digoxin → Gastrointestinal side effects; Toxicity: 1) GI 2) Neurologic
61. Which of the following lipid lowering drugs acts by inhibiting intestinal absorption of
cholesterol?
❏ Prazosin
❏ Hydrochlorothiazide → increases serum cholesterol and triglyceride levels
❏ Losartan
❏ Clonidine
64. A 19 year old female with chronic kidney disease presents with signs and symptoms of
bacterial meningitis. Considering the antibiotics’ CNS penetration, the most ideal medication
to give is:
❏ Amikacin → aminoglycoside, will not cross BBB
❏ Azithromycin → macrolide, will not cross BBB
❏ Chloramphenicol - hepatic metabolism, will cross BBB with or w/o
inflammation; rate of resistant is high
❏ Cefoxitin → 2nd gen cephalosporin; Will not cross BBB except Cefuroxime
however will not treat meningitis
65. Which of the following antibiotics should be given with extreme caution if her serum
creatinine is 4.8 mg/dl?
❏ Moxifloxacin → most of FQs are eliminated via renal and hepatic route
❏ Chloramphenicol → <30% renally excreted
❏ Ceftriaxone → along with Cefoperazone (Cephalosporin with Hepatobiliary
excretion)
❏ Meropenem → more than 70% is renally excreted; has highest urinary
concentration of antibiotics after administration
66. A patient has a history of severe allergic reaction to ampicillin. Which of the following
should be avoided?
67. A patient has meningococcal meningitis. The most effective antibiotic to give that will
also eradicate the N. meningitides in the nasopharynx is:
❏ Clindamycin
❏ Ceftriaxone
cjhernandez/djhorneja/ccorpuz
❏ Chloramphenicol
❏ Penicillin G
❏ Gastrointestinal symptoms
❏ Hypoglycemia
❏ Lactic acidosis → also side effect, but less common
❏ Anemia
69. If patient is pregnant, on her 10th week age of gestation, which of these antibiotics
should NOT be given because of potential adverse effect on the fetus?
71. This antibiotic has poor penetration of the blood brain barrier, hence must not be given
to patients with meningitis:
❏ Chloramphenicol
❏ Meropenem
❏ Vancomycin
❏ Cefaclor → will not cross BBB
72. Antimicrobial combination therapy is warranted in the following:
❏ Prophylaxis against influenza
❏ Treatment of polymicrobial infection → difficult to treat infection so give 2 or
more drugs to achieve more broad spectrum coverage
❏ Elderly patients suffering from community acquired pneumonia, low-risk
❏ TB prophylaxis in children below 5 years
*Other choices need monotherapy only.
73. What is the drug of choice for Type 2 diabetic patients with renal failure?
❏ Sulfonylurea
❏ ∝-glucosidase inhibitor
cjhernandez/djhorneja/ccorpuz
❏ Insulin
❏ Metformin
❏ Cefoxitin crosses the blood-brain barrier → 2nd gen, does not cross BBB
❏ Ceftriaxone and Ceftazidime are both eliminated mainly via biliary secretion →
only Ceftriaxone (along with Cefoperazone), Ceftazidime → renal
❏ Instability of penicillins in gastric acid can limit their oral absorption
❏ Probenecid inhibits proximal tubular reabsorption → given to decrease renal
clearance
*Ceftriaxone and Cefoperazone → Biliary excretion
75. A 34 year old woman is admitted for bacteremia. Gram stain revealed presence of Gram
(-) bacilli. Her records showed she had urticaria, hypotension and respiratory difficulty after
she took oral penicillin V about 6 months ago. The most appropriate drug regimen for
empiric treatment is:
❏ Ceftriaxone
❏ Meropenem
❏ Aztreonam - no cross allergenicity, targets only gram negative
❏ Piperacillin-tazobactam
76. A patient with native valve culture-positive infective enterococcal endocarditis warrants
treatment with what antimicrobial?
❏ Ceftriaxone
❏ Vancomycin
❏ Aztreonam
❏ Ampicillin-sulbactam
77. Which statement about vancomycin is accurate?
❏ It is bacteriostatic
❏ It does not bind to penicillin binding proteins → glycopeptide
❏ It undergoes hepatic metabolism
❏ It has good oral bioavailability → oral Vancomycin for Colitis
78. A 3 year old male child, being treated with an unrecalled antibiotic, is brought to the ER
because of vomiting, diarrhea, and abdominal distention. On PE he was found to be
ashen-colored, hypothermic and hypotensive. and hypothermia. The antibiotic that is likely
to be the cause of this problem is:
❏ Doxycycline
❏ Erythromycin
❏ Chloramphenicol → causes gray baby syndrome
cjhernandez/djhorneja/ccorpuz
❏ Azithromycin
❏ Gentamicin and tobramycin are the most likely to cause renal damage
❏ Ototoxicity due to amikacin includes vestibular dysfunction, which is often
reversible → irreversible once you have hearing loss already
❏ Ototoxicity is reduced if diuretics are used to increase the renal excretion of
the drug
❏ Oliguria is an early sign of aminoglycoside toxicity
82. Which of the following statements is correct regarding the use of fluoroquinolones?
83. Which adverse effect is most common with the use of sulfonamides?
87. A 35 year old businessman presents with neurologic signs and symptoms. Stool exam
showed Taenia saginata eggs. MRI showed calcified cyst. The most appropriate medication
to give is:
88. A 30 year old farmer was diagnosed with schistosomiasis and T. solium infection. The
most effective medication for both infestations is:
❏ Albendazole
❏ Praziquantel → all flukes and T. solium
❏ Ivermectin
❏ Pyrantel palmoate
89. Patients receiving glucocorticoids may develop the following undesirable effects:
❏ Hypertension, osteoporosis, hyperkalemia, and edema → hyperkalemia is
wrong
❏ Peptic ulcer, moon facies, natriuresis, and aseptic necrosis of the hip →
natriuresis is wrong because Na retention is present in those taking steroids
❏ Acute pancreatitis, acne, hidden infections, and glycosuria
❏ Cataract, thinning of the skin, muscle wasting and loss of appetite → gain in
appetite
cjhernandez/djhorneja/ccorpuz
90. A 40 year old nurse on maintenance medication for her chronic Hepatitis B infection
developed signs and symptoms of renal toxicity, as adverse effect of which medication?
cjhernandez/djhorneja/ccorpuz
❏ Drug U = 2L
❏ Drug V = 5L
❏ Drug W = 8L
❏ Drug X = 15L → high Vd wherein most of the drug are distributed in the
tissues of the body, low plasma concentration needing a loading dose
96. Drug A has a half-life of 20 hours. It is expected that its steady state concentration will be
achieved on the ___ hour:
❏ 24th
❏ 48th
❏ 100th → takes about 4-5 half-lives to reach steady state concentration
❏ 150th
97. The following drug is a known inducer of cytochrome P450 enzyme activity:
❏ Rifampicin
❏ Cimetidine
❏ Grapefruit
❏ Ketoconazole
*Rest of the choices are CYP inhibitors
98. A patient on anti-lipidemic agent is complaining of muscle aches and has elevated liver
enzymes. He must be taking:
❏ Simvastatin
❏ Ezetimibe
❏ Cholestyramine → constipation
❏ Niacin → flushing
99. Accumulation of acrolein in the urinary bladder is observed with the use of this
antineoplastic agent:
❏ Methotrexate → Liver toxicity
❏ Cisplatin Nephrotoxic
❏ Chlorambucil → Myelosuppression
❏ Cyclophosphamide → Acrolein causes Hemorrhagic Cystitis; Antidote:
MESNA
100. The use of this class of antineoplastics is associated with development of secondary
leukemias several years from the time of treatment:
❏ Antimetabolites
❏ Alkylating agents → Leukemogenic potential years after
❏ Antitumor antibiotics
❏ Topoisomerase inhibitors
cjhernandez/djhorneja/ccorpuz
PREVENTIVE MEDICINE MOCK BOARD EXAM
UST MBR 2020
1. A 60 year old female presents with chronic dyspnea and a long history of smoking. She
was diagnosed to have COPD and treatment was initiated. In addition to pharmacologic
therapy, what additional recommendations should be included?
❏ Exercise avoidance
❏ Smoking cessation
❏ Hemophilus Influenza B vaccination
❏ All of the above
2. The parents of a 4 year old and newborn child presents for wellness care. Which of the
following is a true statement that may be part of your counseling for the parents to
encourage vaccination?
❏ Hepatitis A is recommended for all children
❏ Inactivated poliovirus vaccine has been associated with vaccine related
polio infection
❏ Pneumococcal vaccine is recommended only for high risk adults
❏ Children receiving aspirin therapy should receive influenza or varicella
vaccines
3. A mother brings her 12 year old son for a routine health care visit. She expresses
concerns about their family’s history of elevated cholesterol and heart disease. Which of
the following will you advise regarding cholesterol?
cjhernandez/djhorneja/ccorpuz
5. A 45 year old previously healthy male presents for annual physical examination. His last
physical examination was performed 10 years ago and was normal. He has no significant
past medical history nor medications. He does not smoke but consumes 2-3 alcoholic
drinks weekly. His family history is significant for hypertension in the mother. Only
pertinent finding in the PE is BP of 148/98 mmhg. Repeat measurement after 1 month is
150/100. Which of the following should be requested?
❏ Cardiac stress testing
❏ Electrocardiogram
❏ No testing
❏ Renal scan
6. Compared with the general population, all of the following patients are at risk for folate
deficiency EXCEPT:
❏ 20 year old man with sickle cell anemia
❏ 20 year old woman who is a strict vegetarian
❏ 60 year old man on hemodialysis
❏ 70 year old man who drinks a quart of gin a day
7. The use of estrogen-containing oral contraceptives increases the risk of
❏ Breast cancer
❏ Ovarian cancer
❏ Thromboembolism
❏ Osteoporosis
8. A 25 year old woman wants to lose weight that is why she joined a health club. She uses
the stationary bike in which each 40 minute session burns up 500 calories. She is taking 2
sessions a week. Assuming her caloric intake is the same, how many weeks will it take to
lose 6 lbs?
❏9
❏ 15
❏ 21
❏ 27
9. Prevention of human brucellosis depends on
❏ Pasteurization of dairy products derived from goats and sheep
❏ Treatment of human cases
❏ Control of insect vector
❏ Immunization of farmers and slaughterhouse workers
10. Preventing the risk of overmedicalization and iatrogenic harm is what level of
prevention?
❏ Primordial
❏ Secondary
cjhernandez/djhorneja/ccorpuz
❏ Tertiary
❏ Quarternary
11. A researcher would like to study the causes of the rarer forms of sarcoma. He discovered
a registry of this form of cancer with access to a large database of patients which
unfortunately is only a few years old. They were given funding only for 1 year. What type
of study design would you recommend?
❏ Historical cohort
❏ Cross sectional
❏ Experimental
❏ Case control (retrospective) → rare forms of disease
cjhernandez/djhorneja/ccorpuz
12. All of the following statements regarding a normal (Gaussian) distribution are true except
❏ Mean = median =mode
❏ Approximately 68% of observation fall within 1 standard deviation of the mean
❏ The number of observations between 0 and 1 standard deviation from the
mean is the same as the number between 1 and 3 standard deviations from
the mean
❏ The shape of the curve does not depend on the value of the mean
13. An investigator is designing a randomized clinical trial to see whether Vitamin E will
prevent cancer in smokers. Which of the following is least important in planning the
sample size of the study?
❏ Prevalence of smoking in the population
❏ Expected incidence of cancer in the placebo group
❏ Magnitude of the preventive effect that the investigator wished to be able to
detect
❏ Frequency with subjects will be lost to follow up or die from non cancer
causes
14. In a case control study to determine the association between blood type and invasive
cervical cancer, the researchers examine the distribution of ABO blood type among those
with cervical cancer and matched controls without cervical cancer. They chose male
subjects for controls and females as cases. Which of the following is a correct statement?
❏ Because males cannot get cervical cancer, recall bias would be eliminated in the
study
❏ Inclusion of males would increase the difficulty of finding matched controls
❏ Inclusion of males would tend to overestimate the risk of cervical cancer
❏ The distribution of ABO blood type is not sex-dependent and thus would be
the same for matched controls of either sex
15. If the household are selected at random and every person in each household is included
in the sample, what is the sampling design?
❏ Paired sampling
❏ Systematic sampling → this method involves initial determination of the
sampling interval (k) by dividing the total population number (N) by the
calculated sample size (n). Simple random sampling is then done.
Afterwards, every nth random number will be included as the subject of the
study.
❏ Stratified sampling → target population are grouped into several levels,
(educational level, economic status, regions etc) then simple random
sampling is done per level.
❏ Cluster sampling → no specific criterion that identifies a group to be better
or poorer than the other, but the population can be groups of
heterogeneous elements, this method may be applied
cjhernandez/djhorneja/ccorpuz
16. Survival of a particular patient for at least 5 years is what type of scale?
❏ Dichotomous
❏ Ordinal → Arranged in rank ordered categories; (E.g., Social class, Likert
scale, Satisfactory scale, agree to disagree, murmur range, level of
edema)
❏ Interval → Value of zero is arbitrary (E.g., Fahrenheit & Celsius)
❏ Ratio → (+) properties of all variables; zero is fixed/absolute; Highest
form; (E.g., Age, metric system)
17. You have just finished a case control study to measure association of alcohol use and
lower respiratory tract infection. What method will you use to control smoking as a
confounding variable?
❏ Matching
❏ Stratification
❏ Randomization
❏ Multivariate analysis
18. In a study about headache, doctors gave patients Tylenol, Advil and Bioflu to see which
medication would relieve the headache pain the fastest. The doctors timed how long it
took the medication to work. What is the dependent variable?
❏ Amount of medication
❏ Type of pain reliever
❏ Headache
❏ Amount of time that pain is relieved
19. The measures that are used to determine the degree or extent of variation in a data set
are called:
❏ Mean → Average
❏ Median → Middle
❏ measures of dispersion
❏ Measures of central tendency
20. The distance between the high and low scores in statistics is:
❏ Range
❏ Mode → highest in frequency
❏ Median
❏ Standard deviation → most common measure of spread; Used with the mean
to describe distribution of observations
21. Injuries or illnesses such as cancer cases requiring chemotherapy or radiotherapy are
Philhealth classified under
❏ Intensive cases
cjhernandez/djhorneja/ccorpuz
❏ Catastrophic cases
❏ Ordinary cases
❏ Remedial cases
22. A man who has undergone vasectomy at a local Family Planning Clinic is considered
sterile:
❏ After 25 ejaculations
❏ One week after the procedure
❏ Immediately after the procedure
❏ After a zero sperm count
23. Violence and injury prevention programs of the DOH includes the following except:
❏ Zero fireworks related injury
❏ Stop bullying in schools promotion
❏ End mental health stigma and discrimination
❏ Suicide prevention hotline
24. A couple has 6 children, 4 boys and 2 girls. The most appropriate family planning for them
is
❏ Permanent method
❏ Barrier method
❏ Method recommended by the doctor
❏ Method they decide and practice
25. Which among the following has no field association with Under 6 Program of the
Department of Health?
❏ Integrated Management of Childhood Illnesses (IMCI)
❏ Knock Out Tigdas (KOT)
❏ Universal Health Care High Impact 5 (UHC-Hi 5)
❏ Rehabilitation and Prevention of Impairment and Disability (RPIOD) →
crossed the lifespan (disability may present in both the young and adults)
26. Which of the following is not included in the Philhealth benefit packages?
❏ TB treatment through DOTS
❏ Rabies treatment
❏ COVID 19
❏ AH1N1
cjhernandez/djhorneja/ccorpuz
27. Which of the following is not a provision of the Philippine Medicine Policy 2017-2022?
❏ Parallel importation of patented cheaper medicines → Cheaper Medicine
Law 2008 (RA 9502)
❏ Strengthen capability of the Bureau of Food and Drugs →FDA Act of 2009
❏ Achieve self-sufficiency in basic drug manufacturing → 1998
❏ International monitoring of drug price → Part of Cheaper Medicine Bill,
Maximum Drug Retail Price
29. All of the following can be eligible for accreditation as TB DOTS facilities except
❏ LGU health clinic
❏ Factory clinic
❏ HMO
❏ None of the above
30. The DOH recommends the following allocation scheme for the TB DOTS benefit package
❏ 25% for consultation services of the referring physician during the treatment
course
❏ 35% for the health facility staff including the treatment partner who had a role
in the delivery of services to the patient
❏ 40% for operational costs involved in providing quality care for TB patients. _
❏ All of the above
31. Universal health care can be provided in which of the following ways?
❏ Handing out vouchers or giving people refundable tax credits so that they are
able to purchase health care in the private sector.
❏ The government can issue a mandate requiring all people to have some form of
health coverage.
❏ Using the private sector in order to subsidize health care coverage.
❏ All of the above
cjhernandez/djhorneja/ccorpuz
32. Under the Republic Act 9994 or the Expanded Senior Citizens Act of 2010, the Mandatory
Philhealth Coverage states that all indigent Senior Citizens shall be covered by the
National Insurance Program of PhilHealth. Where should the enrollment fund come
from?
❏ Department of Social Welfare & Development
❏ Local Government Unit → where the seniors are residing, handled by the
Seniors Citizen Office
❏ Department of Health
❏ Philhealth
33. Which of the following is a correct provision of the senior citizens act?
❏ 15% discount on dental services excluding VAT
❏ Death benefit of P2000 to nearest surviving relative
❏ 5% on water and electricity covering 40 cumm and 200 kilowatt hour
respectively
❏ Mandatory Philhealth coverage
35. Which Republic Act amended RA 7277, the Magna carta for Persons with disability
❏ RA 7305 → Magna Carta of Public Health Workers
❏ RA 9244
❏ RA 7883 → Barangay Health Workers Benefits and Incentives Act of 1995
❏ RA 9710 → Magna Carta of Women
36. All of first year medical students were given Hepatitis vaccination prior to starting school.
This exhibits what type of immunity?
❏ Active immunity
❏ Passive immunity → immunoglobulins
❏ Non-specific immunity
❏ Herd immunity
cjhernandez/djhorneja/ccorpuz
37. A baby was born premature with severe infection. The baby was given IV antibiotics and
also IV immunoglobulin. What type of immunity does immunoglobulin provide?
❏ Active immunity
❏ Passive immunity
❏ Non-specific immunity
❏ Herd immunity
38. In the natural history of disease, an infected person may present with no symptoms? In
what stage of the natural history of disease?
❏ Susceptibility
❏ Presymptomatic Disease → preclinical
❏ Clinical Disease
❏ Disease Outcome → recovery, complications, death
39. A student suddenly developed cough and dyspnea. What stage in the natural history of
disease is this?
❏ Susceptibility
❏ Presymptomatic Disease
❏ Clinical Disease → full clinical horizon of disease
❏ Disease Outcome
40. A measles outbreak occurred in a barangay in Sampaloc. This is transmitted through direct
contact. What is the BEST description of this disease?
❏ Communicable
❏ Contagious
❏ Both A and B
❏ Neither
41. Which social determinant of health does language and literacy rate belong to?
❏ Economic stability
❏ Education
❏ Health and Health care
❏ Neighborhood and built environment
42. Digital rectal exam was performed to all male patients seen at a medical mission as part of
prostate cancer awareness. What type of level of prevention?
❏ Primary
❏ Secondary → getting disease at the earliest; screening
cjhernandez/djhorneja/ccorpuz
❏ Tertiary
❏ Quarternary
43. Several Dengue cases were noted in a barangay. What type of epidemic curve will this
outbreak usually present with?
❏ Classical → Short ascending limb and climb; Rapid transmission due to big dose
of identified organism; Ex. Food poisoning
❏ Inverted → Person to person spread; Long ascending and short descending
limb; Ex. Insect borne diseases (Malaria, dengue)
❏ Bell-shaped → Spread is rapid; transmission is simple and rapid elimination and
reduction of susceptible cases; Rapid ascending and rapid descending limb; Ex.
Measles, poliomyelitis
❏ Circular
44. A 65 -year old male had previous stroke. He now undergoes regular physical therapy for
ambulation training, What stage of the natural history of disease does he belong to?
❏ Susceptibility
❏ Presymptomatic Disease
❏ Clinical Disease
❏ Disease Outcome → rehabilitation
45. What is the ability of the agent to invade and adapt itself to the host?
❏ Antigenic properties → ability to induce production of antibodies; (ex.
Dengvaxia itself is not harmful, if given to patients with prev dengue may cause
dengue hemorrhagic fever due to antigenic properties (4 strains), condition of
person)
❏ Infectivity → ability of an agent to invade and adapt itself to the human host
❏ Virulence → Measure of the severity of the reaction produced; (ex. SARS-COV
in 2003, 2 deaths over 12 confirmed via PCR)
❏ Invasiveness → ability to penetrate into or grow within the host away from the
original site; (ex. Viral infection from upper respiratory tract and later involving
lower respiratory tract)
46. In the Philippines, Dengue cases has been found to be present throughout the entire year.
What type of disease frequency does Dengue belong to?
❏ Epidemic → cases occur beyond expected numbers
❏ Endemic → a disease constantly occurring in a geographical area
❏ Pandemic → epidemic occurring within more than one country or territory;
involvement of regions under WHO (Philippines belong to Western Pacific
region)
❏ Sporadic → occasional or infrequent occurrence of a disease
cjhernandez/djhorneja/ccorpuz
47. In 2003, Severe acute respiratory syndrome (SARS) took the lives of nearly 800 people
worldwide. What is the mode of transmission of this disease?
❏ Common vehicle → etiologic agent is transmitted by water, food, air, or
inoculation. Ex. Vehicle - water, food, soil ; Vector - snails, mosquitoes
❏ Enzootic → diseases occurring in animals
❏ Propagated → from host to host; direct (person to person); Indirect (from
reservoir, i.e. airborne, infected blood)
❏ Zoonotic → disease of animals transmissible to man (brucellosis, anthrax,
plague, rabies)
48. Food poisoning due to a contaminated water source will usually create what type of
epidemic curve?
❏ Bell-shaped → Spread is rapid; transmission is simple and rapid elimination
and reduction of susceptible cases; Rapid ascending and rapid descending limb;
Ex. Measles, poliomyelitis
❏ Classic → Short ascending limb and climb, long descending; Rapid
transmission due to big dose of identified organism
❏ Inverted → Person to person spread; Long ascending and short descending
limb; Ex. Insect borne diseases (Malaria, dengue)
❏ U-shaped
49. Classify under which social determinant of health does crime and violence belong to:
❏ Economic stability → GNP, GDP , Consumer Price Index, Peso Dollar Exchange
❏ Education
❏ Health and Health care → availability, accessibility and human resource for
health
❏ Neighborhood and built environment
50. Malaria is endemic to the province of Palawan and is transmitted through the Anopheles
mosquito. What is the mode of transmission of this disease?
❏ Airborne
❏ Common vehicle →water borne diseases, food poisoning
❏ Direct → person to person; ex. skin lesions such as impetigo
❏ Propagated/Serial → transmitted through vector from a reservoir (infected
person) to a susceptible host
51. Jonel is a 42 year company driver. He recently had a Stroke secondary to Hypertension due
to long hours of work. This is compensable under the ECC on the following conditions:
❏ He had pre-existing hypertension before being employed but was compliant
with medications
cjhernandez/djhorneja/ccorpuz
❏ The acute onset of signs and symptoms occurred when he was on vacation leave
from work
❏ His hypertension is uncontrolled since he stopped taking his anti-hypertensive
medications
❏ No diagnostic tests were done ex: CT scan, to support his compensation claims
52. In the epidemiologic criteria for causality, reversibility can be established by asking the
following question:
❏ When in relation to the exposure did your symptoms start?
❏ Do your symptoms appear when you are in areas of high exposure?
❏ Are your co-workers having the symptoms?
❏ Do your symptoms disappear when you are away from work or during
vacations?
*Temporality - When in relation to the exposure do/did the symptoms start?
*Exposure-response - Are the symptoms especially worse when undertaking task or in
areas with high exposures?
*Strength of association - Do any other persons where you work have similar
symptoms?
*Consistency - Reports of the same symptoms/same exposure
*Specificity - Other exposures or causal factors
*Analogy - Same symptoms/similar agents
*Biological plausibility - Symptoms compatible with mechanism of disease
53. Cindy is a recently hired househelp. She developed contact dermatitis on both her hands
due to exposure sodium hydroxide. Which describes more an Irritant rather than allergic
type of dermatitis?
❏ Pruritus is the main symptom
❏ Lesions appear rapidly within minutes to a few hours after exposure
❏ Patient usually has genetic predisposition
❏ First exposure sensitizes the person and the later exposure elicits the reaction
cjhernandez/djhorneja/ccorpuz
54. The following is an allowed exposure time for noise:
❏ Live concert at 105 dB - 2 hours
❏ Lawnmower at 90 dB - 8 hours
❏ Sandblasting at 115 dB - 1 hour
❏ Movie house at 95 dB - 8hours
*NIHL → Most frequently, the hearing loss is bilateral, although asymmetry can
exist, particularly when the source of the noise is lateralized; OSHA level 85 dbA for
an 8 hour workday Permissible Exposure level 90 dBA for 8 hours (OHS, Philippines)
55. Liza has asthma like symptoms of shortness of breath and wheezing. What would make
you definitely suspect that this occupational asthma rather than work-aggravated
asthma?
❏ She developed the symptoms after a single very high exposure to an irritant
chemical
❏ She has no history of pre-existing asthma or childhood asthma
❏ She has had similar episodes in the triggered by cold temperatures, excessive
exertion or exposure to aerosols
❏ She is on a reliever inhaler but has to use it more frequently when at work
56. Johnny, a 40 year old construction worker, presented at the company clinic with 2 week
history of cough but no fever. He is a non-smoker. You requested for Sputum AFB. Results
came back negative on the three smears. Your next plan of action is:
❏ Treat the patient symptomatically and reassess after
❏ Request for a repeat AFB smear
cjhernandez/djhorneja/ccorpuz
❏ Request for a chest radiograph (PA and lateral view) → TB in adult:
predilection is upper portion; pneumonia - look for obliteration of cardiac
borders, middle lobe involvement, lateral view is needed
❏ Request for a culture
*Current guidelines only require 2 smears
57. Francis is applying for a position as a nurse in a tertiary hospital. Which of the following
Hep B policies in the workplace does not apply to him?
❏ Hep B screening should not be mandatory except if applying for occupations
known to be high risk for transmission
❏ HBsAg positive applicants are disqualified from employment
❏ Hep B status of the job applicant should be confidential
❏ Hep B Vaccination is required for all health care workers
58. A 40 year old male patient who works as a pest control worker consulted because of
headache, nausea and vomiting. You elicited that he just finished a major project and was
exposed to large amounts of organophosphates as part of his job. There was 30%
depression on the cholinesterase enzyme activity. He is suffering from:
❏ Mild poisoning
❏ Moderate poisoning
❏ Severe poisoning
❏ None of the above
59. A 28 year patient was exposed to hydrogen sulfide which rendered the patient
immediately unconscious and death occurred in a few minutes. The physiologic response
of the patient suggests exposure to what air concentration of H2S gas?
❏ 10 ppm
❏ 150 ppm
❏ 200 ppm
❏ 500 ppm
60. Tony works in a battery manufacturing company. Because of the risk of lead toxicity, he
undergoes periodic blood Lead determination. Lately, he has been feeling dizzy and weak
and manifested with wrist drop. His Blood lead level is 60mg/dl. Your recommendation
is:
❏ Remove from the site of exposure immediately and manage accordingly
❏ Terminate him from work because he is not fit for the job
❏ Allow him to continue working because findings are insignificant
❏ Allow him to continue working until his blood Lead level is at an alarming
level
cjhernandez/djhorneja/ccorpuz
61. The standardized mortality ratio indicates fewer deaths than expected when it is:
❏ Between 0 and 1
❏ Less than crude death rate
❏ <100
❏ Negative
*We standardized when we are asked to compare 2 settings
63. Eighty percent of susceptible household contacts of a child with chickenpox develop this
disease:
❏ Secondary attack rate → A person who acquires the disease from
exposure to a primary case is called SECONDARY CASE.
❏ Morbidity rate
❏ Prevalence rate
❏ Incidence rate
64. Suppose that 3 patients were followed-up after tonsillectomy for recurrent ear infections.
One patient was followed up for 10 months, one for 20 months, and a 3rd for 25 months.
10 ear infection were documented in these 3 patients during the specified period. Which
of the following measures would allow investigators to make full use of their collected
data?
❏ Incidence density
❏ Attributable risk
❏ Period prevalence → easures the frequency of all current cases of
disease (old and new cases) for a prescribed period of time (e.g.,
one year)
❏ Point prevalence → measures the frequency of all current cases of
a disease (old and new cases) at a given instant or specified
moment of time (e.g., one month or one quarter)
*Incidence rate
- Measures the rapidity with which a disease occurs or the frequency
of addition of new cases of a disease
cjhernandez/djhorneja/ccorpuz
- (No. people who get a disease In a specified period/Total no.
population at risk of Experiencing the event during This period) x
1000
65. A most gracious governor invites a dozen of his closest friends to a formal luncheon at his
residence. His kitchen is well known for a salmon appetizer. Within 24 hours, 11 of 17
guests experienced abdominal pain, vomiting and diarrhea. Though the governor who is
not particularly keen on his house specialty, was in fact fine. Of the 11 symptomatic
diners, 4 had fever while 7 do not; 5 presented with elevated WBCs while 6 did not; 6 ate
shrimp salad while 5 did not; 9 ate the salmon appetizer and 2 did not. The 11
symptomatic guests recovered within 3 days. The guests present had shared no other
meals at any time in recent days. The attack rate for this case is:
❏ 4/11/20
❏ 11/13/20
❏ 5/11/20
❏ 11/17/20 → guests who are symptomatic/total no of guests
67. Reporting incidence rate of disease x, it would be necessary to know which of the
following relevant information:
❏ Age distribution of the population
❏ The midyear population at risk ---> midyear is July 1
❏ Case fatality rate
❏ Duration of clinical illness
cjhernandez/djhorneja/ccorpuz
68. Reporting prevalence of a disease X means getting hold of which of the following
information?
❏ Cure rate
❏ Number of cases at a given time → both old and new cases
❏ Duration of the illness
❏ The rate at which new cases are discovered
69. Data are presented based on a cohort study of coronary heart disease. Which parameter
best describes the statement. During the first 8 years of the study, 45 persons developed
coronary heart disease per 1000 persons who entered the study free of disease:
❏ Incidence rate → rapidity with which a disease occurs or the
frequency of addition of new cases of a disease
❏ Period prevalence → cases over a defined period, ex. Years
❏ Point prevalence → shorter time reference, ex. Months, quarter
❏ Relative risk → ratio of the probability of an event occurring in the
exposed group versus the probability of the event occurring in the
non-exposed group.
70. Rates are synonymous with risks. Compared with risks, rates:
❏ Are based on a more accurate denominator
❏ Are generally lower
❏ Are significantly higher
❏ Show the force of mortality better
71. The most frequent site of serious contact with an environmental hazard is the:
❏ Eye
❏ Skin
❏ Lung
❏ Gastrointestinal tract
72. The indoor air pollutant of greatest public health importance is:
❏ Carbon monoxide
❏ Tobacco smoke → Smoking: number one risk factor for cardiovascular
disease
❏ Dioxin
❏ Dust
cjhernandez/djhorneja/ccorpuz
73. Organic compounds that linger in the environment, bio-accumulate through the food web,
and pose a risk of causing adverse effects to human health and the environment:
❏ Persistent Organic Pollutants (POPs)
❏ Poisonous and toxic fumes
❏ Ozone Depleting Substances (ODS)
❏ Greenhouse gases
74. Any solid, semi-solid or liquid waste or residue generated from a wastewater treatment
plant, water supply treatment plant, or water control pollution facility, or any other such
waste having similar characteristics and effects:
❏ Sewage → water-borne human or animal wastes from industries and
household; MWSS (Metropolitan Waterworks and Sewerage System)
❏ Septage → septic tank and cesspools
❏ Effluent → coming out from the industries to body of water
❏ Sludge
75. Function of management which could guarantee wise use of available resources is:
❏ Planning → come up with predetermined course of action
❏ Evaluation → to determine if objectives are attained
❏ Staffing → Manning and keeping positions provided for by the
organizational structure; defines manpower requirements for jobs
❏ Situational analysis
*NOH 2017-2022 Endpoint: Healthy Philippines
77. An organized system of health care delivery, offering a comprehensive set of benefits, in
which members are voluntarily enrolled, and paying for a fixed, prepaid period:
❏ Life insurance → packages are not usually health related
❏ Capitation → seen in a socialized country like UK, they dont shell out money and
pay; they just seek appointment with health care providers and the government
subsidizes everything
❏ Managed care → payment of the premium defined by HMO, upon payment the
member has contract with HMO for one year, he receives out-patient, in-patient
benefit entitlement; the one who manages the funds is the HMO; credible in
accredited institutions as determined by HMO. If you are not able to use
benefits for that year, it cannot be carried over to the next year.
cjhernandez/djhorneja/ccorpuz
❏ Out-of-pocket care → 54% from household
78. In computing for the cost of illness, which of the following data are required?
❏ Average days of non-morbid condition
❏ Number of death
❏ Average cost of savings
❏ Average daily income or wage
79. The accepted terminology for the total market value of final goods and services produced
by the national economy which is utilized as a tool for allocation of national budget is:
❏ Gross Domestic Product → t otal monetary or market value of all the
finished goods and services produced within a country's borders in a
specific time period.
❏ Consumer Price Index → measure that examines the weighted average of
prices of a basket of consumer goods and services, such as transportation,
food, and medical care
❏ Gross National Product → total produce of country including both goods
and services in a setting of 1 year; WHO recommends that country spend
5% of GNP for health
❏ Health expenditure
80. Permissible exposure limits (PELs) are used by industries and government agencies to
describe the allowable amount of a toxic chemical to which persons may be
occupationally exposed. PELs are correctly described by which of the following
statements?
❏ They are based on consideration of the acute and long-term health
effects of hazardous substances
❏ They have been established for most chemicals in use by industry
❏ Employers are not responsible for surveying and sampling for chemical
exposure
❏ They do not require a monitoring system to evaluate the levels of
chemicals in the worker’s environment
81. Which of the following diseases is found almost exclusively among persons who have
worked with or have been exposed to asbestos?
❏ Bronchogenic carcinoma
❏ Byssinosis
❏ Pleural mesothelioma
❏ Laryngeal carcinoma
82.The major environmental source of lead absorbed in the human blood stream in adults is:
❏ Air
cjhernandez/djhorneja/ccorpuz
❏ Paint
❏ Water
❏ Food
83. Consequence of imbalance between water and salt content in the extra and intracellular
fluids due to excretion of salt through excessive sweating:
❏ Heat stroke → failure of thermoregulatory center and suppression of sweating;
CNS dysfunction
❏ Heat exhaustion → results from deficiency of water or salt leading to
circulatory insufficiency; feeling of fatigue but no CNS manifestations; rectal
temp <40 degrees
❏ Heat cramps → benign form; lactic acidosis
❏ All of the above
*Heat syncope → fainting; Standing erect and immobile in hot environment due to
pooling of blood in blood vessels of the skin and lower body parts
84. The triad of stomatitis, tremors and erethism is commonly associated with:
❏ Lead
❏ Hydrogen sulfide
❏ Mercury
❏ Arsenic
86. Decentralization of basic services including health to Filipinos was effected by:
❏ Health Sector Reform Agenda, 2000
❏ Universal Health Care Law 2019
cjhernandez/djhorneja/ccorpuz
❏ Local Government Code, 1991 → Local government units given the
responsibility for health service
❏ Cheaper Medicine Act, 2008
87. The health status of a community may be measured or expressed in terms of:
❏ Availability of health care services
❏ Availability of public utilities
❏ Its statistical indices of morbidity and mortality → triple burden of disease;
the top 10 causes of morbidity in the Phils are 9 out of 10 communicable.
No. 1 communicable disease is Acute Respiratory Infection; Hypertension is
the only noncommunicable disease among these.
❏ Utilization of health care services
*Why triple burden? 1st burden is morbidity, the communicable diseases; 2nd is
Mortality, non communicable diseases caused by 1) Diseases of Heart, 2) Vascular,
3) Malignancy and 3rd burden those caused by 1) Climate 2) Mental health disease
3)Physical injuries 4) Pandemic
*Top 1 leading cause of mortality is Cardiovascular disease
88. What program of the Department of Health focuses on the primary prevention of disease
under 12 months of age?
❏ Maternal and Child health
❏ Integrated Management of Childhood Illnesses
❏ Expanded Program of Immunization → BCG, HBV, DTaP, OPV, Measles
❏ Family Planning and Reproductive health
89. Which of the following diagnostic outcomes is prioritized by the DOTS (National Anti-TB)
Program of PHILHEALTH and DOH?
❏ Sputum negative but with history suggestive of TB
❏ Sputum negative cases but with possible X-ray findings
❏ Sputum positive cases
❏ Sputum negative but with clinical findings suggestive of TB
90. The agency that makes policies, strategies and programs on drug dependency prevention
and control:
❏ Philippine FDA
❏ Department of Health
❏ Dangerous Drugs Board → Dangerous Drugs Act of 1972
❏ Philhealth
cjhernandez/djhorneja/ccorpuz
❏ Is often greater when control over exposure is low
❏ Is often greater when control over exposure is high
❏ Tends to be less than the magnitude of the risk
92. In the Philippine health picture, which disease top lists mortality causes?
❏ Tuberculosis
❏ Accident
❏ Pneumonia
❏ Cardiovascular Disease
93. Almost half of the total health spending in the Philippines is paid by:
❏ Philippine Health Insurance Corporation → 20% of expenditure
❏ Private Health Insurance and Community-Based Health Financing Schemes
❏ Out-of-pocket Settlement of Individual Families → 50% personal account
❏ National and Local Government →what is left of the health spending after
out of pocket and PhilHealth
94. A person with AIDS refuses intubation for Pneumocystis carinii pneumonia and dies:
❏ Beneficence
❏ Autonomy
❏ Maleficence
❏ Justice
95. Refers to the reduction or limitation of the adverse impacts of hazards and related
disasters including measures, which encompass engineering techniques and
hazard-resistant construction as well as improved environmental policies and public
awareness:
❏ Capacity building
❏ Contingency planning
❏ Mitigation
❏ Disaster preparedness → aware of protocol during a disaster
96. What function of management involves selection of candidates for specific position in the
organization, training and professional development?
❏ Directing → from top level management to low level
❏ Planning
❏ Staffing → Human Resource for Health; HRM
❏ Evaluating
cjhernandez/djhorneja/ccorpuz
97. Which is a description of what the organization would look like to be in a preferred future
when it is fulfilling its purpose?
❏ Vision → dream, aspiration
❏ Policy
❏ Mission statement → to achieve the vision
❏ Program
98. In which of the following situations can PHILHEALTH benefit entitlement be availed of?
❏ Confinement in an accredited hospital for less than 24 hours due to an illness or
disease requiring hospitalisation
❏ Confinement falls outside the 45 days allowance for room and board but
complies with the rule/policy on single period of confinement.
❏ Confinements in non-accredited hospitals for an elective surgical procedure
❏ Payment of at least three (3) monthly contributions within the immediate six
month period prior to confinement
99. Which Primary health care characteristic assures utilization of essential health services in
available in a given community without prejudice to time, distance and ethos?
❏ Acceptability
❏ Affordability
❏ Accessibility
❏ Accountability
cjhernandez/djhorneja/ccorpuz
LEGAL MEDICINE MOCK BOARD EXAM
UST MBR 2020
1. It occurs when the pressure inside the container exceeds its structural strength. Ex. Air
pressure tanks, paint spray, water pressure tanks, air pump kerosene burner.
❏ Mechanical Explosion
❏ Electrical Explosion
❏ Nuclear Explosion → nuclear fusion of atoms that will create huge explosion
❏ A and B only
3. The presence of weapon which is tightly grasped by the hand of a victim of a shooting
incident is an example of:
❏ Rigor mortis → form of rigidity but occurs 3-6 hrs after death
❏ Death stiffening
❏ Cadaveric rigidity
❏ Defloration → “Deflowering”
❏ Gerontophilia
❏ Tribadism → lesbianism
❏ Pedophilia → children/infants
6. Algor Mortis can be defined as:
❏ The stiffening of the muscles and joints → Rigor mortis
❏ Postmortem decomposition
❏ Cold Stiffening → fats are solidified that’s why body becomes rigid
❏ Brain death
❏ Somatic death → cessation of all vital functions of brain, heart and lungs; no
resuscitation
10. Disease or injury that produces the physiological disruption in the body:
❏ Manner of death → how the cause of death came into being
(Natural/Violent death)
❏ Causes of death
❏ Virgo-intacta
cjhernandez/djhorneja/ccorpuz
❏ True Physical Virginity → hymen intact, opening is small and barely admits
finger of examiner
❏ Saponification
❏ Delusion of grandeur
❏ Tracing E.
20. Cutis Anserina is seen in:
❏ Drowning
❏ Immersion
❏ Hanging
❏ Strangulation
*Cutis Anserina - goose skin; contraction of erector pili muscle
21. Special Methods of preserving human tissue sections:
❏ Embalming → preserve whole body
❏ 10% Formalin
22. Insertion of foreign body to the mouth would lead to:
❏ Choking → mechanical occlusion of the air passage from inside
❏ Gagging
cjhernandez/djhorneja/ccorpuz
❏ Burking → form of traumatic asphyxiation where knees of assailant are
placed on chest, abdomen, back preventing person from moving and
respiration; usually accidental
23. Rulings based on laws that affect or deal with the practice of medicine are covered in:
❏ Legal medicine
❏ Forensic medicine
❏ Litigation Medicine
❏ Medical Jurisprudence
24. Medical knowledge for the purpose of law and administration of justice is studied in:
❏ Legal medicine
❏ Ethical medicine
❏ Medical jurisprudence
❏ Litigation Medicine
25. Points of identification, applicable to both the living and the dead,EXCEPT:
❏ Race
❏ Moles
❏ Scars
❏ Mugging → form of strangulation with the assailant standing at the back and
uses elbow around neck of victim
27. A 32-year-old farmer received abdominal trauma, after 2 hours the pulse reached
136/min, blood pressure 70/40. The cause is due to:
❏ Shock
❏ Internal hemorrhage
❏ Sympathetic shock
❏ Air embolism
28. Covering both nostrils and mouth with a pillow may cause death due to:
❏ Traumatic asphyxia → crucifixion, stampede
❏ Asphyxia by smothering
cjhernandez/djhorneja/ccorpuz
❏ Asphyxia choking
❏ Asphyxia by mugging
29. Asphyxial death occurs when the building collapse, landslide or stampede:
❏ Asphyxia by Smothering
❏ Traumatic Asphyxia
❏ Asphyxia by Burking
❏ Mugging
❏ Gagging
❏ Overlaying
*rest will produce abrasions on the mouth
31. An autopsy technique that involved the removal of internal organs in situ with en bloc:
❏ Virchow technique → systemized removal of organs
❏ Choking
❏ Smothering
34. Abortion which the law allows under some specific justifications:
❏ Therapeutic A.
cjhernandez/djhorneja/ccorpuz
❏ Criminal A.
❏ Stellate
❏ Both
❏ Neither
37. External examination of a dead body without incision being made:
❏ Autopsy → involves dissection
❏ Latin
❏ Spanish
❏ Chinese
39. An instrument used in examination of sexual abused like Rape:
❏ Electron Microscope
❏ Colonoscopy
❏ Colposcope
❏ Endoscope
40. The effect of physical violence may not always result to the production of this, but it is
always the effect of physical violence:
❏ Wound
❏ Vital Reaction
❏ Blunt Trauma
cjhernandez/djhorneja/ccorpuz
❏ Sharp Injuries
41. Refers to sensation of heat or rise in temperature:
❏ Calor
❏ Dolor
❏ Rubor
❏ Loss of Function
42. The presence of this differentiates an ante-mortem from post-mortem injury:
❏ Wound
❏ Vital Reaction
❏ Postmortem lividity
❏ Postmortem rigidity
43. Physical or mechanical stretching resulting to separation of skin or portion of tissues:
❏ Laceration
❏ Avulsion
❏ Tearing
❏ Splitting
44. Internal hemorrhage is due to Rupture of the blood vessel which may cause hemorrhage,
it may be due to the following, EXCEPT:
❏ Traumatic intracranial hemorrhage
❏ Stab Wound
❏ Punctured Wound
❏ Chop Wound
46. A general practitioner may confuse rigor mortis with:
❏ Cold stiffness
❏ Heat stiffness
❏ Cadaveric spasm
cjhernandez/djhorneja/ccorpuz
❏ Inevitable A.
❏ Missed A.
❏ Criminal A.
48. A physical injury located at the site and also opposite the site of application of force:
❏ Coup Injury → located at the site of application of force
❏ Locus Minoris Resistencia → injury not on site nor opposite site but
on areas of least resistance
49. The study of the changes in the body during decomposition and how it affects the
environment is:
❏ Forensic Entomology → using life cycle of insects to determine time of
death
❏ Forensic Pathology
❏ Forensic Taphonomy
50. The event which produced the fatality by initiating a natural and continuous sequence of
events unbroken by an efficient intervening cause and without which the end result would
not have occurred is called the:
❏ immediate cause of death
❏ The medical profession directly affects the very lives of the people, unlike other
careers which, for this reason, do not require regulation.
❏ Self-imposed limitation
❏ Law
cjhernandez/djhorneja/ccorpuz
❏ Medical Society
53. Pursuant to the Medical Act of 1959, a person shall be considered as engaged in the
practice of medicine:
❏ Pre-employment physical examination → not practice of medicine, unless
company doctor prescribes Medicine
❏ Physically examine any person, and diagnose, treat. operate or prescribe any
remedy for human disease, injury, deformity, physical, mental and psychical
condition.
❏ Who shall use the title of M.D. after his name. → illegal practice of medicine;
fine of 1-10, 000 pesos and imprisonment of 1-5 years
❏ A person who had treated and prescribed for certain from whom he received
monetary compensation without having previously obtained the proper
certificate of registration from the Board of Medical Examiners.
❏ A nurse that takes the blood pressure reading and gives advice and
prescribes treatment
cjhernandez/djhorneja/ccorpuz
57. Maldita’s daughter was suffering from an impacted tooth. She consulted Maldito who
told her to give paracetamol, a household remedy. Which describe appropriately Maldito’s
action?
❏ Illegal practice of medicine
❏ Diplomacy
❏ International right
❏ Insubordination
❏ Indirect contempt → committed outside the court, but you defy the
order of the court. Punishment: prison of up to 6 months and fine of
30,000 pesos
cjhernandez/djhorneja/ccorpuz
❏ Dereliction
62. It is the act or omission amounting to want of care on the part of the patient which,
concurring with the physician’s negligence, is the proximate cause of injury:
❏ Continuing negligence → If the MD, after a prolonged treatment of the
patient which normally produces alleviation of the condition, fails to
investigate non-response.
❏ Criminal liability
❏ Ethical liability
64. An anesthesiologist, by his negligence, faulty intubated a patient, which resulted to
permanent brain damage on the patient who was to undergo elective laparoscopic
cholecystectomy. Who may be held liable under the Captain of the ship Doctrine?
❏ The operating room nurse
❏ The surgeon → he will have to pay for the the acts of anesthesiologist
❏ The anesthesiologist
cjhernandez/djhorneja/ccorpuz
❏ Vicarious liability
❏ Medical certificate
❏ Medical records
69. Common defense of the hospital with regard to consultants’ liability to his private
patients;
❏ Independent Contractor
❏ Vicarious liability
70. Under this doctrine, a private hospital-owner may be liable for the mistakes of the house
residents:
❏ Borrowed servant doctrine → Employees of the hospital are sometimes
temporarily under the supervision and control of another.
❏ Criminal Liability
❏ Civil Liability
❏ No Liability
*PMA may file the case against the doctor at the PRC or doctor’s membership can
be terminated from PMA
72. If Dr. Mighty LiLing is charged with Homicide resulting from Gross Negligence/Reckless
Imprudence due to his patient’s death, he is most likely being investigated for what?
❏ Criminal Liability
❏ Administrative Liability
❏ Civil Liability
❏ No Liability
73. A post-graduate medical intern sutured a lacerated wound of a patient without the
supervision of a duly registered physician which caused tetanus infection and eventual
death. What are the liabilities that will be attached to the post-graduate medical intern?
❏ Civil and criminal liabilities
❏ Criminal Liability
❏ Civil Liability
❏ Ethical Liability
75. PD 169 punishes doctors who fail to report cases of less serious and serious physical
injuries with
❏ a fine of one to ten thousand pesos and 1 to 5 years imprisonment
cjhernandez/djhorneja/ccorpuz
❏ fine one to three thousand pesos and 1 to 5 years imprisonment
*PD 169 – requiring MD, hospitals clinics, etc. to report any persons of serious
and less serious physical injuries to the nearest police authorities; law specific for
doctors
*Failure to report may be fined with 1,000 – 3,000 and a prison term of between
1 – 3 years and revocation of license to practicE
76. The penalty for failure report child abuse case under RA 7610 is:
❏ fine of five thousand pesos
❏ 2/12/93
❏ 2/13/94
❏ 2/14/95
78. Duty to actively do good for patients:
❏ Nonmaleficence → Do no harm
❏ Fidelity
❏ Beneficence
❏ Veracity
79. Which of the following records requires disclosure as provided by law?
❏ Physician’s order sheet
❏ Medical certificate
❏ Medical records
80. True of the principle of Informed consent:
❏ The basic principle of informed consent is that, after having been properly
educated by the physician about the risks and benefits of the proposed
treatment, and about the viable treatment options, including the option of no
treatment, the patient has the right to choose freely whether to submit to
treatment.
❏ Informed consent is a legal condition whereby a person can be said to have given
consent based upon a clear appreciation and understanding of the facts,
implications and future consequences of an action.
cjhernandez/djhorneja/ccorpuz
❏ In order to give informed consent, the individual concerned must have adequate
reasoning faculties and be in possession of all relevant facts at the time consent
is given.
81. Some acts cannot legally take place because of a lack of informed consent. Which of the
following statement is true?
❏ In cases where an individual is considered able to give informed consent,
another person is generally authorized to give consent on their behalf.
❏ Both
❏ Neither
82. Elements of informed Consent:
❏ Sufficiency of the quality of disinformation → NOT disinformation
cjhernandez/djhorneja/ccorpuz
❏ Consensual
❏ Contractual
❏ Fiduciary
85. Except is an emergency, a physician may refuse to treat a person seeking his services:
❏ If other physicians are available
❏ When disclosure of information will serve both public and private health and
safety → can be done
❏ The colon was perforated during sigmoidoscopy and the surgeon decides on
laparotomy to undertake repair.
89. Under our Code of Ethics, we have the following duties except:
❏ Duties to patients
❏ Duties to colleagues
❏ Duties to community
cjhernandez/djhorneja/ccorpuz
❏ Duties owed to the hospital management
*Duty to Allied Medical Profession
90. The following statements is/are true in relation to Duties of Physician to Colleagues
pursuant to the Code of Ethics:
❏ The physician is obliged to respect the dignity and good name of his fellow
physicians.
❏ Both
2. Jose gave consent to surgically remove one of his kidneys which he intends to sell to a
9
patient in Japan with chronic renal disease. Which one of the following requisites was
violated as to make the consent invalid?
❏ Informed and enlightened
❏ Obtained voluntarily
❏ Enlightened
93. In which of the following is consent necessary:
❏ In out-patient basis consultation
❏ In extreme emergencies
❏ Refer the matter to the ethics committee as this involves the life of the
mother and the fetus
cjhernandez/djhorneja/ccorpuz
95. In the following statements, when is patient’s consent necessary?
❏ In an emergency, patient need tracheostomy
❏ The colon was perforated during sigmoidoscopy and the surgeon decides on
laparatomy to undertake repair.
6. If the patient is severely demented, the claim for medical fee shall be made from the
9
following persons in order:
❏ Patient, spouse, descendant, ascendant, brothers and sisters
❏ Ethical dilemma
❏ Moral uncertainty
❏ Moral principle
98. Considered Father of Medicine
❏ Leonardo da Vinci
❏ Hippocrates
❏ Pontius Pilate
❏ Aesculapius
99. The age required of a person to be able to give consent in the Philippines is:
❏ 21 years old
❏ 18 years old
❏ both
❏ neither
100. Which of the following statements is true regarding extension of surgery?
❏ If the condition does not immediate attention but will be favorable to the
patient
cjhernandez/djhorneja/ccorpuz
cjhernandez/djhorneja/ccorpuz